Sie sind auf Seite 1von 47

TOPNOTCH MEDICAL BOARD PREP PATHO SUPERTABLE by KEVIN ELOMINA, MD

For inquiries visit www.topnotchboardprep.com.ph or


https://www.facebook.com/topnotchmedicalboardprep/
Foreword: chromatin of a dying cell.
Pathology is very broad subject, and it takes discretion to Type of cell death, which is energy- Apoptosis
determine which information is essential for a general dependent, tightly regulated, and associated
practitioner or a medical graduate about to take the board with normal cellular functions; often
exam. physiologic; the cell undergoes shrinkage
I have compiled the most common, most high-yield, and and fragmentation.
most noteworthy concepts (according to my discretion, Pathway of apoptosis triggered by loss of Mitochondri
experience during boards, and my training as a Pathology survival signals, DNA damage and al/Intrinsic
resident) in general and systemic pathology; however, this accumulation of misfolded proteins; pathway
table is NOT to be used solely. inhibited by anti-apoptotic members of the
In order to practice integration with other disciplines, I Bcl family.
have omitted recurring themes in some chapters. It is up to you Initiator caspase for intrinsic pathway of Caspase 9
to maximize the utility of this handout by integrating related apoptosis.
concepts across different subjects. Also, I have included the Pathway of apoptosis responsible for Death
pages of some important tables in Robbins that you should elimination of self-reactive lymphocytes and receptor/
know by heart. damage by cytotoxic T lymphocytes; initiated Extrinsic
Should you need someone to talk to, with regard to this by TNF receptors. pathway
handout, the lecture presentations, Pathology in general, or Initiator caspases for extrinsic pathway of Caspase 8, 10
personal concerns that preclude you from performing apoptosis.
optimally during the review, do not hesitate contact me Executioner caspases. Caspase 3, 6
through Facebook or Instagram. I am willing to listen, and offer Regulated cell death that results in necrosis Necroptosis
some advice and counseling, if necessary. (morphologically); caspase-independent;
To the one who uses this, my prayers are with you. I activates RIP1 and RIP3 complexes that lead
fervently hope that this would help you get a good grade in to increased ROS and decreased
Pathology and in the boards. Study smart, and pray hard. God mitochondrial ATP production.
bless.
Type of apoptosis often seen in microbe- Pyroptosis
Kevin (Kei) Elomina, MD
infected cells; caspase 1 releases IL-1, and
Top 7, September 2016 PLE J
together with caspase 11, causes cell injury.
Facebook: Kevin Elomina
It involves sequestration of cellular Autophagy
Instagram: @keielomina
organelles into cytoplasmic autophagic

vacuoles that fuse with lysosomes and digest
PATHO SUPERTABLE enclosed material; used as a survival
mechanism under various stress conditions
PART I: GENERAL PATHOLOGY (i.e. nutrient deprivation).
CELLULAR RESPONSES TO STRESS AND TOXIC INSULTS: Marker for autophagy; involved in formation LC3
ADAPTATION, INJURY, AND DEATH of autophagosome.
QUESTION ANSWER A form of tissue necrosis in which the Coagulative
Increase in SIZE of cells resulting in Hypertrophy component cells are dead but the basic tissue necrosis
increased size of organ; cellular adaptation of architecture is preserved; affected tissues
non-dividing cells (i.e. myocardial fibers). take on a firm texture; often seen in infarcts
Increase in NUMBER of cells; examples: BPH, Hyperplasia of all solid organs (heart, spleen, kidney)
papilloma virus infection except the brain.
Reduction in the size of an organ or tissue Atrophy Characterized by digestion of dead cells, Liquefactive
due to decrease in cell size and number; resulting in transformation of the tissue into necrosis
example: Alzheimer’s disease a liquid viscous mass; often seen in infections
(pus) and in hypoxic death of cells within the
A reversible change in which one Metaplasia
CNS.
differentiated cell type is replaced by another
cell type; example: Barrett esophagus This term is usually applied to a limb, Gangrenous
generally the lower leg, that has lost its blood necrosis
The first manifestation of almost all forms of Cellular
supply and has undergone coagulative
injury to cells; due to influx of ions (and swelling
necrosis with superimposed liquefactive
water) due to failure of energy-dependent
necrosis involving multiple tissue layers.
ion pumps.
Friable, white appearance of necrosis; Caseous
A type of reversible injury, characterized by Fatty change
appears as a structureless collection of necrosis
appearance of lipid vacuoles in the (Steatosis)
fragmented or lysed cells and amorphous
cytoplasm; often seen in cells participating in
granular debris enclosed within a distinctive
fat metabolism (liver, heart).
inflammatory border (granuloma); “cheese-
Type of cell death, which results from a Necrosis
like”.
pathologic cell injury; undergoes cellular
Refers to focal areas of fat destruction, Fat necrosis
swelling and eventual pyknosis,
typically seen in acute pancreatitis; there is
karyorrhexis, and karyolysis.
release of activated pancreatic lipases into
It is the irreversible condensation of Pyknosis
the substance of the pancreas and the
chromatin in the nucleus of a cell undergoing
peritoneal cavity; the foci of necrosis contain
necrosis or apoptosis; there is nuclear
shadowy outlines of necrotic fat cells with
shrinkage with increased basophilia.
basophilic calcium deposits (saponification),
It is the destructive fragmentation of the Karyorrhexis
surrounded by an inflammatory reaction.
nucleus of a dying cell.
A special form of necrosis usually seen in Fibrinoid
It is the complete dissolution of the Karyolysis
immune reactions involving blood vessels; necrosis
TOPNOTCH MEDICAL BOARD PREP PATHO SUPERTABLE Page 1 of 47
For inquiries visit www.topnotchboardprep.com.ph or https://www.facebook.com/topnotchmedicalboardprep/
TOPNOTCH MEDICAL BOARD PREP PATHO SUPERTABLE by KEVIN ELOMINA, MD
For inquiries visit www.topnotchboardprep.com.ph or
https://www.facebook.com/topnotchmedicalboardprep/
deposits of immune complexes, together with vessel wall without increase in
fibrin that have leaked out of vessels, result vascular permeability.
in a bright pink and amorphous appearance An extravascular fluid with high Exudate
in H&E stains, called "fibrinoid" (fibrin-like) protein content; its presence
by pathologists. implies an increased vascular
Abnormal calcium deposition occurring in Dystrophic permeability, triggered by tissue
the absence of calcium metabolic calcification injury and ongoing inflammatory
derangements; examples: Psammoma bodies reaction.
in cancers with papillary architecture and Steps in leukocyte recruitment. A. Margination
meningioma B. Rolling
Calcium deposition in normal tissues Metastatic C. Adhesion
occurring in the presence of hypercalcemia; calcification D. Transmigration
example: calcinosis The process of leukocyte Margination
Effect of caloric restriction on longevity. Increased accumulation at the periphery of
due to blood vessels.
decreased Molecules in the endothelium P and E-selectins on
IGF-1 and leukocyte responsible for endothelium with
signaling rolling. Sialyl-Lewis-X on
leukocyte; Glycam-1,
CD34 on endothelium
with L-selectin on
Proteins that are produced in response to Sirtuins leukocyte
food deprivation; with actions that lead to Molecules responsible for ICAM-1 on endothelium
prolonged longevity (anti-apoptotic, anti- adhesion. with CD11/CD18
ROS). integrins (aka, LFA-1,
Mac-1) on leukocyte;
CHAPTER 2: INFLAMMATION AND REPAIR VCAM-1 on endothelium
QUESTION ANSWER with VLA-4 on leukocyte
INFLAMMATION The process of migration of the Transmigration or
Response of vascularized tissues Inflammation leukocytes through the diapedesis
to infections and damaged tissues endothelium.
that brings cells and molecules of Molecules in the endothelium PECAM-1/CD31
host defense from the circulation responsible for transmigration.
to the sites where they are Process of leukocyte migration Chemotaxis
needed, in order to eliminate the toward sites of infection or injury
offending agents. along a chemical gradient.
General steps of inflammatory A. Recruitment of Exogenous chemoattractants. Bacterial products (N-
response. leukocytes formylmethionine,
B. Regulation of lipids)
response Endogenous chemoattractants. IL-8, C5a, LTB4
C. Recognition of Type of inflammatory mediators Cell-derived mediators
injurious agent that are normally sequestered in
D. Removal of agent intracellular granules and can be
E. Resolution rapidly secreted by granule
Morphologic hallmarks of acute Vasodilation and exocytosis or are synthesized de
inflammation. accumulation of novo in response to a stimulus;
leukocytes and fluid in
examples: Histamine, Serotonin,
the extravascular tissue
Cytokines and Arachidonic acid
Components of acute 1. Dilation of small derivatives
inflammation. vessels Type of mediators that are Plasma-derived
2. Increased produced mainly in the liver and mediators
microvascular are present in the circulation as
permeability inactive precursors that must be
3. Emigration of activated by proteolytic
leukocytes and their
cleavages to acquire their
activation biologic properties; examples:
Most notable mediator that Histamine Complement, coagulation and
produces vasodilation. kinin systems
Most common mechanism of Endothelial cell Vasoactive amine that results in Serotonin
increased microvascular contraction vasoconstriction; sources include
permeability. platelets and neuroendocrine
Initial vascular response to Vasoconstriction cells.
injury. Most abundant complement. C3
An ultrafiltrate of blood, which Transudate Complement that acts as an C3b
contains little protein, little or no opsonin.
cellular material, and low specific Anaphylatoxins. C3a, C4a, C5a
gravity as a result of osmotic or Membrane attack complex. C5b, C6-C9
hydrostatic imbalance across the Anti-inflammatory cytokines for Lipoxin, TGF-β, IL-10
TOPNOTCH MEDICAL BOARD PREP PATHO SUPERTABLE Page 2 of 47
For inquiries visit www.topnotchboardprep.com.ph or https://www.facebook.com/topnotchmedicalboardprep/
TOPNOTCH MEDICAL BOARD PREP PATHO SUPERTABLE by KEVIN ELOMINA, MD
For inquiries visit www.topnotchboardprep.com.ph or
https://www.facebook.com/topnotchmedicalboardprep/
regulation of the inflammatory Accumulation of fluid in tissues or Edema (tissues) or
response. body cavities. effusion (body
Inflammatory response of Chronic inflammation cavities)
prolonged duration in which Four main mechanisms of edema A. Increased
inflammation, tissue injury, and formation. hydrostatic
attempts at repair coexist, in pressure
varying combinations. B. Decreased oncotic
Two kinds of cells seen in Epithelioid cells pressure
granulomas. (activated macrophages C. Increased vascular
with pink, granular permeability
cytoplasm, resembling Lymphatic
epithelial cell) and giant obstruction
cells General morphologic appearance Clearing and
REPAIR of edema. separation of ECM,
Type of tissue whose cells can Labile tissues and subtle cell
readily regenerate as long as the swelling
pool of stem cells is preserved; It is an active process resulting Hyperemia
examples: Bone Marrow, and from augmented blood flow due to
Vaginal Epithelium arteriolar dilation; affected tissue
Cells of theses tissues are Stable tissues is redder than normal, because of
quiescent and have only minimal engorgement with oxygenated
replicative activity in their blood.
normal state; capable of It is a passive process resulting Congestion
proliferating in response to from impaired venous return out
injury or loss of tissue mass; of a tissue; tissue has a blue-red
examples: Smooth muscles, color due to accumulation of
Endothelium, and Liver deoxygenated hemoglobin in the
Parenchyma affected tissue.
Cells of these tissues are Permanent tissues Characterized by alveolar Acute pulmonary
considered to be terminally capillaries engorged with blood, congestion
differentiated and with associated alveolar septal
nonproliferative in postnatal life; edema or focal minute intra-
examples: Neurons, Cardiac alveolar hemorrhage.
muscle Pulmonary septa are thickened Chronic pulmonary
Type of repair that happens in Regeneration and fibrotic, with hemosiderin- congestion
labile and stable tissues; laden macrophages in alveolar
influenced by growth factors. spaces.
Type of repair that happens in Connective tissue The central vein and sinusoids of Acute hepatic
chronic, severe inflammation, deposition the liver are distended with blood, congestion
and in permanent tissues. with central hepatocyte
degeneration; periportal
Steps in healing by connective A. Angiogenesis hepatocytes are better
tissue deposition. B. Formation of oxygenated.
granulation tissue The central regions of the hepatic Chronic hepatic
C. Remodeling of the lobules are grossly red-brown, congestion
scar slightly depressed, and are
Most notable growth factor in VEGF accentuated against the
angiogenesis. surrounding zones of uncongested
Most important cytokine for TGF-β tan, sometimes fatty liver (nutmeg
synthesis and deposition of liver); presence of centrilobular
connective tissue. necrosis with hepatocyte drop-
The pink, soft, granular tissue Granulation tissue out, hemorrhage, and
seen beneath the scab of a skin hemosiderin-laden macrophages.
wound; hallmark of tissue Pathologic form of hemostasis. Thrombosis
healing; appears by 3-5 days. Components of Virchow triad Endothelial injury,
Histologic findings of granulation Proliferation of (abnormalities that lead to Stasis,
tissue. fibroblasts and new, thrombus formation). Hypercoagulability
thin-walled, delicate It is a major contributor to the Turbulence or
capillaries development of arterial thrombi. endothelial injury
(angiogenesis) in a It is a major contributor to the Stasis
loose ECM, often with development of venous thrombi.
admixed inflammatory Any alteration of the coagulation Hypercoagulability
cells, mainly pathway that predisposes to
macrophages thrombosis; can be primary (e.g.
Factor V Leiden, Protein C and S
CHAPTER 3: HEMODYNAMIC DISORDERS, deficiency) or secondary (e.g.
THROMBOEMBOLIC DISEASE, AND SHOCK Cancer, atrial fibrillation, and
QUESTION ANSWER prolonged immobilization)
TOPNOTCH MEDICAL BOARD PREP PATHO SUPERTABLE Page 3 of 47
For inquiries visit www.topnotchboardprep.com.ph or https://www.facebook.com/topnotchmedicalboardprep/
TOPNOTCH MEDICAL BOARD PREP PATHO SUPERTABLE by KEVIN ELOMINA, MD
For inquiries visit www.topnotchboardprep.com.ph or
https://www.facebook.com/topnotchmedicalboardprep/
Laminations composed pale Lines of Zahn circulations, previously congested
platelet and fibrin deposits tissues, or when flow is
alternating with darker red cell– reestablished after an infarction
rich layers; signifies formation of (i.e. after angioplasty of
thrombus in flowing blood; obstructed artery); examples:
present in antemortem Pulmonary and Intestinal infarcts
thrombosis. Infarcts that tend to occur in solid White infarct
Most common site of arterial Coronary > cerebral organs with end-arterial
thrombosis. > femoral circulations; examples: Myocardial
Most common site of venous Superficial or deep and Splenic infarcts
thrombosis. veins of the leg (Note: State of systemic tissue Shock
thrombi in hypoperfusion due to reduced
superficial leg veins cardiac output and/or reduced
rarely embolize, effective circulating blood volume.
while thrombi in This type of shock results from Cardiogenic shock
deep leg veins are failure of the cardiac pump, which
the most common maybe caused by MI, ventricular
sources of venous arrhythmias, cardiac tamponade
emboli) or outflow obstruction.
A detached, intravascular solid, Embolus This type of shock results from Hypovolemic shock
liquid or gaseous mass that is loss blood or plasma volume.
carried by the blood distal to its This type of shock is caused by Septic shock
point of origin. microbial infection, caused by
Most common and most dreaded Pulmonary embolism Gram-negative and Gram-positive
sequelae of deep venous bacteria and fungi.
thrombosis. Type of shock that occurs in the Neurogenic shock
Embolus occluding a bifurcation in Saddle embolus setting of an anesthetic accident or
the pulmonary tree; associated spinal cord injury as a result of
with sudden death due to acute loss of vascular tone and
right-sided heart failure. peripheral pooling of blood.
A venous embolus, which entered Paradoxical embolus This type of shock represents Anaphylactic shock
the systemic circulation through systemic vasodilation and
an interarterial or interventricular increased vascular permeability
defect. caused by an IgE-mediated
Microscopic fat globules found in Fat embolism hypersensitivity reaction.
the circulation after fractures of
long bones or after soft-tissue CHAPTERS 4: GENETIC DISORDERS
trauma; can lead to pulmonary QUESTION ANSWER
insufficiency, neurologic GENETIC DISORDERS
symptoms, anemia, and Mendelian disorders
thrombocytopenia. One dominant allele is enough to produce Autosomal
Gas bubbles within the circulation Air embolism phenotype (heterozygous); diseases with dominant
obstructing vascular flow and mutations in key structural
causes distal ischemic injury; proteins/receptors are usually inherited in
example: Decompression sickness this manner; examples: Huntington disease,
(bends, chokes) NF, Myotonic dystrophy, TSC, ADPKD,
Entry of amniotic fluid into the Amniotic fluid Familial polyposis coli, HS, vWD, Marfan
maternal circulation through a embolism syndrome, EDS (some types), OI,
tear in the placental membranes Achondroplasia, Familial
and rupture of uterine veins; hypercholesterolemia, AIP
presence of marked pulmonary Two recessive alleles produce phenotype; Autosomal
edema, diffuse alveolar damage, enzyme deficiencies are usually inherited in recessive
and presence of squamous cells in this manner: examples: CF, PKU,
the pulmonary circulation shed Galactosemia, Homocystinuria, Lysosomal
from fetal skin, lanugo hair, fat, storage diseases, α-1-antitrypsin deficiency,
and mucin. Wilson disease, Hemochromatosis, Glycogen
Area of ischemic necrosis caused Infarct storage diseases, SCA, Thalassemias, CAH,
by occlusion of the vascular supply EDS (some types), Alkaptonuria, Neurogenic
to the affected tissue. muscular atrophies, Freidrich ataxia, Spinal
muscular atrophy
Most common lysosomal storage disorder; Gaucher
most common form is chronic, non- disease
neuronopathic form (Type I); acute
neuronopathic form (Type II) most severe;
microscopically, distended phagocytic cells
in spleen, liver, BM, LNs, tonsils, thymus and
Infarcts that tend to occur in loose Red infarct Peyer patches, with fibrillary cytoplasm
tissues and in those with dual (“crumpled tissue paper” appearance).
TOPNOTCH MEDICAL BOARD PREP PATHO SUPERTABLE Page 4 of 47
For inquiries visit www.topnotchboardprep.com.ph or https://www.facebook.com/topnotchmedicalboardprep/
TOPNOTCH MEDICAL BOARD PREP PATHO SUPERTABLE by KEVIN ELOMINA, MD
For inquiries visit www.topnotchboardprep.com.ph or
https://www.facebook.com/topnotchmedicalboardprep/
CoA, etc.
Type of hermaphrodism with presence of True
both ovarian and testicular tissue. hermaphrodis
m
Type of hermaphrodism characterized by Pseudoherma
disagreement between phenotypic and phrodism
gonadal sex; examples: MRKH syndrome and
androgenital syndromes
In this pattern of inheritance, males are X-linked Trinucleotide repeat disorders
usually affected due to hemizygosity of the recessive Property of trinucleotide repeat disorders Anticipation
allele, but females may express phenotype disorders wherein the disease worsens with each
due to random activation of one X successive generation.
chromosome; examples: DMD, Hemophilia A Most common cause of MR following Fragile X-
and B, CGD, G6PD deficiency, Trisomy 21; involves CGG expansion in syndrome
Agammaglobulinemia, Wiskott-Aldrich FMR1 gene in X-chromosome; most
syndrome, DI, Lesch-Nyhan syndrome, distinctive feature is macro-orchidism.
Fragile X syndrome
In this pattern of inheritance, females in X-linked
general are more affected than males (in dominant Autosomal dominant neurodegenerative Huntington
any scenario), because one allele in a female disorders disorder; CAG expansion on HTT gene on disease
is enough to manifest the trait; examples: Ch4p16.3; relentlessly progressive and
Alport syndrome and Vitamin D-resistant uniformly fatal.
rickets Diseases featuring maternal inheritance; Mitochondrial
Chromosomal disorders usually, fundamental defect is abnormality disorders
Most common of the chromosomal Trisomy 21 in the electron transport chain; examples:
disorders; leading cause of MR; most (Down MELAS, Leber hereditary optic neuropathy
common cause: nondisjunction of Ch21 syndrome) (LHON)
during meiosis; diagnostic clinical features: Disorders of genomic imprinting
flat facial profile, oblique palpebral fissures, Deletion in paternally derived Chromosome Prader-Willi
and epicanthal folds. 15q11.2q13; clinical features include: syndrome
Prominent occiput, mental retardation, Trisomy 18 mental retardation, hypotonia, profound
micrognathia, low-set ears, short neck, (Edward hyperphagia, obesity, small hands and feet,
overlapping fingers, CHDs, renal syndrome) and hypogonadism.
malformations, limited hip abduction, and Deletion in maternally derived Angelman
rocker-bottom feet are features of this Chromosome 15q12; clinical features syndrome
syndrome. include: mental retardation, ataxic gait,
Microcephaly and mental retardation, Trisomy 13 seizure, and inappropriate laughter “happy
microphathalmia, polydactyly, cardiac (Patau puppet”.
defects, umbilical hernias, renal defects, and syndrome)
rocker-bottom feet are features of this CHAPTER 5: DISEASES OF THE IMMUNE SYSTEM
syndrome. QUESTION ANSWER
Components of Chromosome 22q11.2 DiGeorge Gell and Coombs classification of hypersensitivity
deletion syndrome. syndrome Results from activation of TH2 CD4+ Type I
(CATCH22) helper T cells by environmental Hypersensitivity
and antigens, leading to the production (Immediate)
velocardiofaci of IgE antibodies, which become
al syndrome attached to mast cells; examples:
Patients with Ch22q11.2 deletion syndrome Schizophrenia Anaphylaxis, Allergies, Bronchial
are at high risk of these disorders. and bipolar asthma
disorders Caused by antibodies that bind to Type II
Condition characterized by ≥ 2 X Klinefelter fixed tissue and cell antigens and Hypersensitivity
chromosomes and ≥ 1 Y chromosomes; syndrome promote phagocytosis and (Antibody-mediated)
most common karyotype is 47XXY (90%); destruction of the coated cells or
important cause of reduced trigger pathologic inflammation in
spermatogenesis and male infertility; tissues; examples: Goodpasture
clinical features include: eunuchoid body syndrome, Autoimmune hemolytic
habitus, testicular atrophy, micropenis, anemia, Immune thrombocytopenia,
absence of secondary sex characteristics, Pemphigus vulgaris, ANCA
etc. vasculitides, Acute rheumatic fever,
Complete or partial monosomy of X Turner Graves disease, Myasthenia gravis,
chromosome; female hypogonadism in syndrome Insulin-resistant diabetes, Pernicious
phenotypic females; Most common anemia, hyperacute and acute
abnormality is lack of entire X chromosome humoral rejection
(45XO) (57%); single most important cause Caused by antibodies binding to Type III
of primary amenorrhea; clinical features antigens to form complexes that Hypersensitivity
include: ovarian streaks, absence of circulate and may deposit in (Immune-complex
secondary sex characteristics, short stature, vascular beds and stimulate mediated)
cystic hygromas of head neck, preductal inflammation secondary to
TOPNOTCH MEDICAL BOARD PREP PATHO SUPERTABLE Page 5 of 47
For inquiries visit www.topnotchboardprep.com.ph or https://www.facebook.com/topnotchmedicalboardprep/
TOPNOTCH MEDICAL BOARD PREP PATHO SUPERTABLE by KEVIN ELOMINA, MD
For inquiries visit www.topnotchboardprep.com.ph or
https://www.facebook.com/topnotchmedicalboardprep/
complement activation; examples: commonly encountered; IV (Diffuse)
SLE, serum sickness, Arthus reaction, involvement of half or more
PSAGN, PAN, Reactive arthritis glomeruli; with diffuse
Cell-mediated immune responses in Type IV hypercellularity and wire-looping
which T lymphocytes cause tissue Hypersensitivity (T- due to extnsive subendothelial
injury, either by producing cell- deposits.
cytokines that induce inflammation mediated/delayed) Widespread thickening of capillary Lupus nephritis Type
and activate macrophages, or by wall due to subepithelial immune V (Membranous)
directly killing cells; examples: complexes.
Allergic contact dermatitis, Multiple Complete sclerosis of greater than Lupus nephritis Type
sclerosis, Tuberculosis, PPD, RA, IBD, 90% of glomeruli; presents with VI (Advanced
Acute cellular and chronic rejection, end-stage renal disease. sclerosing)
Type 1 DM and Hashimoto A clinicopathologic entity with a Sjogren syndrome
thyroiditis (the last two have Type 2 triad of dry eyes, dry mouth and
components as well) other manifestations, such as
Autoimmune diseases arthritis.
32/F presents to the clinic with SLE Most common and most important Anti-Ro (SS-A) and
symptoms of fatigue, joint pains, autoantibody detected in Sjogren anti-La (SS-B) (90%)
and malar rash. CBC revealed syndrome.
anemia and low platelet count. Triad of 1. chronic inflammation Systemic sclerosis
What is the diagnosis? (autoimmunity); 2. widespread
Hallmark of SLE. Production of damage to small blood vessels; 3.
autoantibodies progressive interstitial and
Mechanisms of organ damage in Type III (most); Type perivascular fibrosis in the skin and
SLE. II (opsonization, multiple organs.
hematologic Most common autoantibody Anti-DNA
manifestations) associated with diffuse cutaneous topoisomerase I (anti-
systemic sclerosis. Scl 70)
Most common autoantibody in SLE. ANA (98%) Common autoantibody associated Anti-centromere
Most specific autoantibodies for anti-Sm (Smith), anti- with limited scleroderma (limited antibodies
SLE. dsDNA (anti-dsDNA cutaneous systemic sclerosis).
correlates with Most common antibody associated Anti-RNA pol III (up
disease activity) with scleroderma renal crisis. to 50%)
Autoantibodies associated with Anti-Ro (SS-A), Anti- Most common extracutaneous Raynaud
congenital heart block in neonatal La (SS-B) manifestation of systemic sclerosis. phenomenon
lupus.

Histopathologic findings in SLE LM: Liquefactive
with skin involvement. degeneration of basal Most common cause of death in Interstitial lung
layer, edema at DEJ, patients with systemic sclerosis. disease and
mononuclear pulmonary arterial
infiltrates around hypertension
blood vessels and Mixture of the features of SLE, Mixed connective
skin appendages systemic sclerosis, and tissue disease (MCTD)
IF: Deposition of Ig polymyositis; associated
and complement at autoantibody: anti-RNP particle-
DEJ containing U1 RNP.
Type of LE that associated with Drug-induced lupus
Hydralazine, INH, Procainamide, D- Transplantation Immunology
Penicillamine intake; rarely 48/M patient underwent kidney Hyperacute rejection
involves kidneys and brain; rarely transplant suddenly developed
associated with anti-dsDNA; bloody urine few hours after the
associated with anti-histone procedure. Nephrectomy was done
antibodies. and revealed a cyanotic, mottled,
Morphology of Lupus nephritides and flaccid kidney and necrotic
Immune complexes in mesangium Lupus nephritis Type kidney cortex. There is
only; no alterations detectable by I (Minimal mesangial) neutrophilic accumulation in the
light microscopy. arterioles, glomeruli, and
Immune complexes in mesangium Lupus nephritis Type peritubular capillaries. Glomeruli
with mild to moderate increase in II (Mesangial undergo thrombotic occlusion of
mesangial matrix and cellularity. proliferative) capillaries and fibrinoid necrosis in
Lesions visualized in fewer than Lupus nephritis Type arterial walls.
half of the glomeruli, segmentally or III (Focal) 45/F presented with signs of Acute cellular (T-cell
globally distributed within each kidney failure 2 months after her mediated) rejection
glomerulus; with cell proliferation, renal transplant.
swelling, and infiltration of Immunohistochemical staining
neutrophils and fibrinoid deposits. reveals both CD4+ and CD8+
Most serious form and most Lupus nephritis Type lymphocytes. Morphologic findings
TOPNOTCH MEDICAL BOARD PREP PATHO SUPERTABLE Page 6 of 47
For inquiries visit www.topnotchboardprep.com.ph or https://www.facebook.com/topnotchmedicalboardprep/
TOPNOTCH MEDICAL BOARD PREP PATHO SUPERTABLE by KEVIN ELOMINA, MD
For inquiries visit www.topnotchboardprep.com.ph or
https://www.facebook.com/topnotchmedicalboardprep/
showed an extensive interstitial characterized by syndrome
mononuclear infiltrate with edema thrombocytopenia, eczema, and
and mild interstitial hemorrhage. marked vulnerability to recurrent
30/M developed oliguria and Acute antibody- infection ending in early death.
subsequent renal failure 3 weeks mediated rejection Hereditary angioedema is C1 inhibitor
after an uneventful kidney secondary to: deficiency
transplant. Renal findings showed Deficiency of the following C3
necrotizing vasculitis with complement protein increases
endothelial cell necrosis, susceptibility to infections
neutrophilic infiltration, deposition involving pyogenic bacteria.
of complement, antibody and fibrin, Deficiency of the following C1q, C2 and C4
and thrombosis. complement proteins increases
24/F diagnosed with renal failure Chronic rejection susceptibility to immune complex-
underwent renal transplant. Four mediated disease.
years later, increasing levels of Deficiency of the following C5-C9
creatinine was noted, and biopsy complement proteins increases
showed an interstitial fibrosis and susceptibility to Neisseria
tubular atrophy, glomerulopathy infections.
with duplication of basement Secondary immunodeficiencies
membrane, and multilayering of Most common secondary Acquired immune
peritubular capillary basement immunodeficiency; caused by HIV deficiency syndrome
membranes. Interstitial infection, characterized by (AIDS)
mononuclear cell infiltrates, NK profound immunosuppression that
cells and plasma cells were also leads to opportunistic infections,
noted. secondary neoplasms, and
Complication of hematopoietic stem Graft vs. host disease neurologic manifestations; the virus
cell transplantation, wherein targets CD4+ T cells
immunologically competent cells The hallmark of AIDS: Variable loss of T-cell
attack the tissues of the mediated immunity
immunocompromised host. Cells initially infected by HIV in Dendritic cells
Primary and secondary immune deficiencies sexual transmission.
Defects in adaptive immunity Glycoprotein present in HIV for gp120
A constellation of genetically Severe combined attachment.
distinct syndromes with common immunodeficiency Glycoprotein present in HIV for gp41
feature of defects in both humoral (SCID) fusion.
and cell-mediated immune Most common fungal infection in Candidiasis
responses, making affected infants patients with AIDS.
susceptible to severe recurrent Most common type of lymphoma B-cell lymphomas
infections by bacteria, viruses, associated with HIV infection.
fungi, protozoans, and A disorder of protein misfolding, Amyloidosis
opportunistic infections. causing extracellular deposition of
Two common patterns of X-linked: common pink or red colored deposits stained
inheritance of SCID, and the gamma chain with Congo red, with apple-green
associated defects. (receptors of ILs, birefringence in polarized light.
especially IL-7) Most common organ affected in Kidney
AR: Adenosine amyloidosis and also with the most
deaminase deficiency serious involvement.

Primary immune deficiency X-linked
characterized by absent or agammaglobulinemia
markedly decreased numbers of B- (Bruton disease)
cells in the circulation, with
depressed serum levels of all
classes of immunoglobulin.
Cardinal features of Thymic Cardiac defect (TOF),
hypoplasia (DiGeorge syndrome). Abnormal facies,
Thymic aplasia, Cleft
palate, Hypocalcemia,
22q11.2 CHAPTER 6: NEOPLASIA
chromosomal Note: Specific cancers will be discussed in their respective
deletion (CATCH 22) systems.
Disorder of lymphocyte activation, Hyper IgM syndrome QUESTION ANSWER
characterized by absence of CD40L Nomenclature
(CD145) that results in lack of class- An abnormal mass of tissue the Neoplasm
switching with consequent increase growth of which exceeds and is
in IgM and decrease in IgA, IgG and uncoordinated with that of the
IgE. normal tissues and persists in the
An X-linked recessive disease Wiskott-Aldrich same excessive manner after the
TOPNOTCH MEDICAL BOARD PREP PATHO SUPERTABLE Page 7 of 47
For inquiries visit www.topnotchboardprep.com.ph or https://www.facebook.com/topnotchmedicalboardprep/
TOPNOTCH MEDICAL BOARD PREP PATHO SUPERTABLE by KEVIN ELOMINA, MD
For inquiries visit www.topnotchboardprep.com.ph or
https://www.facebook.com/topnotchmedicalboardprep/
cessation of stimuli, which evoked Hallmarks of cancer. Self-sufficiency in
the change. growth signals;
Localized neoplasms; usually ends Benign neoplasms insensitivity to growth
with –oma, except lymphoma, inhibitory signals;
seminoma, dysgerminoma, and evasion of cell death;
hepatoma, and melanoma (these limitless replicative
are malignant neoplasms). potential;
Neoplasms that invade and Malignant neoplasms development of
destroy adjacent tissues. sustained
Malignant neoplasms of epithelial Carcinomas angiogenesis; and
origin; usually spread by ability to invade and
lymphatic route; example: metastasize
colorectal adenocarcinoma. Normal cellular genes whose Proto-oncogenes
Malignant neoplasms of Sarcomas products promote cell
mesenchymal origin; usually proliferation; examples: RAS (most
spread by hematogenous route; commonly mutated proto-oncogene
example: uterine leiomyosarcoma. in human cancers), and ABL (in
Benign, but disorganized Hamartoma CML)
appearance of tissue indigenous to Mutant or overexpressed versions Oncogenes
a particular organ; example: Peutz- of proto-oncogenes that function
Jegher polyp autonomously without a
Cytologically and architecturally Choristoma requirement for normal growth-
normal tissue in an ectopic promoting signals.
location; example: Ectopic gastric Genes whose products apply Tumor suppressor
tissue in Meckel diverticulum brakes to the cell proliferation; genes
Characteristics of malignancies loss of function mutations of such
Extent to which neoplastic cells Differentiation genes lead to carcinogenesis.
resemble their normal forebears “Governor of the cell cycle”; a Rb
morphologically and functionally. tumor suppressor gene that
Considered a hallmark of Anaplasia antiproliferative effects by
malignancy, which literally means controlling G1-S checkpoint in the
"to form backward"; term used to cell cycle; mutated in
describe cells with little or no Retinoblastoma; key element in
differentiation. HPV tumorigenesis.
Disorderly but non-neoplastic Dysplasia "Guardian of the genome"; a tumor p53
proliferation of cells; described as suppressor gene that regulates cell
a loss in uniformity of individual cycle progression, DNA repair,
cells and in their architectural cellular senescence, and apoptosis,
orientation. and the most frequently mutated
Fibrous tissue formation in Desmoplasia tumor-suppressor gene in human
response to neoplasm. cancers; mutated in Li-Fraumeni
Dysplastic changes, which involve Carcinoma in situ syndrome.
the entire thickness of the Metabolism unique to cancer cells; Warburg metabolism
epithelium. High glucose uptake and increased
Development of secondary Metastasis conversion of glucose to lactose
implants discontinuous with the (fermentation), which provides
primary tumor, in remote tissues; carbon moieties necessary for
more than any other attribute, this biosynthesis and growth of cancer
identifies a neoplasm as cells.
malignant. Most common mechanism of Interference in the
Next to metastasis, this is the most Local invasiveness evasion of apoptosis used by intrinsic
reliable feature that distinguishes cancer cells. (mitochondrial)
malignant from benign tumors. pathway of apoptosis
Cancer epidemiology
Top 3 common cancers in ALL > CNS tumors > The limitless replicative potential Inactivation of
children. Burkitt lymphoma of cancer cells are due to the senescence signals and
following mechanisms: reactivation of
Top 3 common cancers in males. Prostate > Lung >
telomerase
Colorectal
Mechanisms of angiogenesis in VEGF and oncogenes
Top 3 common cancers in females. Breast > Lung >
cancer cells. that stimulate
Colorectal
synthesis of VEGF
Top 3 common cancer mortalities Lung > Prostate >
(RAS, MYC and MAPK)
in males. Colorectal
Steps in invasion and metastases A. Loosening of cell-cell
Top 3 common cancer mortalities Lung > Breast >
of cancer cells. contacts
in females. Colorectal
B. Degradation of ECM

components

C. Attachment of novel

ECM components
Molecular biology of cancer
TOPNOTCH MEDICAL BOARD PREP PATHO SUPERTABLE Page 8 of 47
For inquiries visit www.topnotchboardprep.com.ph or https://www.facebook.com/topnotchmedicalboardprep/
TOPNOTCH MEDICAL BOARD PREP PATHO SUPERTABLE by KEVIN ELOMINA, MD
For inquiries visit www.topnotchboardprep.com.ph or
https://www.facebook.com/topnotchmedicalboardprep/
D. Migration of tumor infection (cirrhosis), and scarring
cells schistosoma (pipestem fibrosis).
VIRAL DISEASES
Clinical oncology Multinucleated giant cells with Warthin-Finkeldey
Term used to describe the degree Tumor grade eosinophilic nuclear and cytoplasmic cells
of differentiation based on inclusions, seen in measles.
histologic appearance of the Pink to purple intranuclear inclusion Cowdry Type A
tumor. bodies, seen in Herpes simplex virus inclusion bodies
(HSV) infections.
Infection with Varicella-Zoster virus Intraepithelial
Term used to describe the degree Tumor stage (VZV) produces this kind of lesion. vesicles
of localization/spread of the Large, atypical cells with "Owl's eye" Cytomegalovirus
tumor; usual criteria: location and nuclei are seen in this viral infection. (CMV)
size of the primary tumor, nodal There are no morphologic hallmarks Epstein-Barr virus
status, and presence of distant for this viral infection, but it induces (EBV)
metastases; has more prognostic lymphoid cell proliferation
value than tumor grade. (peripheral blood lymphocytosis);
Progressive loss of body fat and Cancer cachexia associated with nasopharyngeal
lean body mass, accompanied by carcinoma, Burkitt lymphoma, and
profound weakness, anorexia, and some forms of HL.
anemia in cancer patients; main BACTERIAL DISEASES
cytokine implicated is TNF. Common Gram-positive and Gram-negative infections
Symptom complexes that occur in Paraneoplastic Main difference between More extensive
patients with cancer that cannot syndromes staphylococcal and streptococcal tissue destruction in
be readily explained by local and infections. staphylococci
distant spread of the tumor of by Suppuration with grayish Corynebacterium
elaboration of hormones not pseudomembrane formation, is seen diphtheriae
indigenous to the tissue of origin in what bacterial infection? Clue:
of the tumor; examples: ectopic "Chinese characters" appearance of
ACTH production (Cushing organism
syndrome) in Small cell lung Gram-positive intracellular bacilli in Listeria
cancer; Hypercalcemia in CSF are diagnostic of what bacterial monocytogenes
squamous cell carcinoma (due to infection? Clue: diagnosis is
PTHrp expression) granulomatosis infantiseptica.
Condition secondary to release of Tumor lysis syndrome Suppuration with tissue necrosis and Bacillus anthracis
products of dying cancer cells hemorrhagic lesions are diagnostic
during chemotherapy; of what bacterial infection?
characterized by: hyperkalemia, Gram-negative intracellular Neisseria gonorrheae
hyperphosphatemia, diplococci are diagnostic of what
hyperuricemia, and hypocalcemia. bacterial infection?
Fleur-de-lis pattern of necrotizing Pseudomonas
CHAPTER 7: INFECTIOUS DISEASES pneumonia and perivascular aeruginosa
QUESTION ANSWER infiltration of organisms producing a
GENERAL PATTERN OF RESPONSE TO INFECTIONS perivascular "blue haze"?
Usual pattern of inflammation, Suppurative Buboes, pneumonia and neutrophilic Yersinia pestis
observed in infections with sepsis are clinical forms of what
extracellular Gram-positiv cocci, and bacterial infection?
Gram-negative rods (pyogenic Mycobacteria (Tuberculosis is discussed under Lungs)
organisms). Myobacterial infection that causes Mycobacterium
Usual pattern of inflammation Mononuclear disseminated disease in patients avium complex
observed in chronic infections, and with profound immunodeficiency (MAC)
acute viral and intracellular bacteria (AIDS and transplant patients);
and parasites. histologically, there is no granuloma
Usual pattern of inflammation Granulomatous formation; rather, macrophages with
observed in tuberculosis, fungal AFB are seen, due to profound loss of
infections and schistosome eggs; CMI.
response to infectious agents that Form of leprosy associated with Tuberculous
are not easily eliminated. asymmetric peripheral nerve (paucibacillary)
Usual response to viral infections Cytopathic/ involvement; formation of leprosy
that involves cytopathic changes in Cytoproliferative granulomas, and positive lepromin
cells (inclusion bodies and skin test, due to an intact cell-
multinucleated giant cells) or mediated immunity.
proliferation of host cells. Form of leprosy associated with Lepromatous
Usual response to clostridial Tissue necrosis symmetric peripheral nerve (multibacillary)
infections, E. histolytica, HBV in liver, involvement; formation of lipid- leprosy
and Herpesviruses in brain. laden macrophages (lepra cells) with
Usually a sequela of chronic Chronic globi (AFB), and a negative lepromin
inflammation, seen in chronic HBV inflammation and skin test, due to a depressed cell-
TOPNOTCH MEDICAL BOARD PREP PATHO SUPERTABLE Page 9 of 47
For inquiries visit www.topnotchboardprep.com.ph or https://www.facebook.com/topnotchmedicalboardprep/
TOPNOTCH MEDICAL BOARD PREP PATHO SUPERTABLE by KEVIN ELOMINA, MD
For inquiries visit www.topnotchboardprep.com.ph or
https://www.facebook.com/topnotchmedicalboardprep/
mediated immunity. Trypanosome related disease Chagas disease
Syphilis transmitted by Triatoma sp. (Trypanosoma cruzi)
Lesion in syphilis characterized by Syphilitic gumma (reduviid bug), primarily affects
central coagulation necrosis, rimmed skeletal, smooth, and cardiac muscle
by palisading macrophages, and (heart, esophagus and colon).
fibroblasts, plasma cell-rich Note: Helminths will be covered in Microbiology.
infiltrate, and few organisms.
Characteristic lesion in syphilitic Obliterative CHAPTER 8: ENVIRONMENTAL AND NUTRITIONAL
aortitis. endarteritis of the PATHOLOGY
vasa vasorum QUESTION ANSWER
Forms of neurosyphilis. Meningovascular, ENVIRONMENTAL PATHOLOGY
Paretic, and Tabes Air pollution
dorsalis Organ mostly affected in air Respiratory (lungs)
Anaerobic diseases pollution.
Myonecrosis with gas bubble α-toxin (lecithinase) Size of particles in soot that are more Fine or ultrafine
formation is caused by Clostridium deleterious to health. particles (<10 µm)
perfringens. What enzyme is Systemic asphyxiant; important CO
responsible for the tissue necrosis? cause of accidental and suicidal
Pseudomembranous colitis, Toxin A: chemokine death; non-irritating colorless and
characterized by mucopurulent Toxin B: cytotoxin odorless gas; also present in
exudate reminiscent of a volcano, is cigarette smoke; clinically
associated with C. difficile. What are characterized by cherry-red
the toxins elaborated by this discoloration of skin and mucous
organism? membranes.
Chlamydial diseases CO saturation where systemic 20-30%
Most common sexually transmitted Chlamydia hypoxia occurs.
bacterial disease in the world. trachomatis CO saturation where coma and death 60-70%
occurs.
Heavy metal poisoning
Main difference between gonococcal Absence of Microcytic hypochromic anemia, Lead
and chlamydial urethritis. organisms in involvement of CNS (more common
chlamydial in children), and PNS (more common
urethritis in adults), and skeletal system are
Fungal diseases manifestations of poisoning with this
Gray-white, dirty-looking Candidiasis heavy metal.
pseudomembrane, with underlying Primary targets of mercury Kidney and CNS
mucosal hyperemia and poisoning.
inflammation, is characteristic of
what fungal infection? Forms of mercury that are more Metallic and organic
Infection with this fungus produces Cryptococcus associated with CNS damage due to
granulomas in immunocompetent neoformans their lipophilic properties.
hosts, and soap-bubble lesions in “Poison of kings”, “King of poisons”; Arsenic
CNS, in immunocompromised hosts. targets are CVS, GIT and CNS;
Fungus with septate hyphae, Aspergillus trivalent compounds are usually
branching at acute angles (40°). toxic; and can cause lung, bladder
Non-septate hyphae, branching at Mucor and skin cancers.
right angles. Heavy metal that is preferentially Cadmium
PARASITIC DISEASES toxic to kidneys; primary targets are
Causative agent of malaria, diagnosis Plasmodium lungs and bones; associated with
depends on demonstration of Itai-Itai disease.
asexual stages WITH hemozoin Occupational exposures
pigment. Most readily preventable cause of Smoking
Causative agent of babesiosis; Babesia death in humans.
diagnosis depends on demonstration Addictive substance in tobacco Nicotine
of parasites in peripheral blood smoke.
WITHOUT hemozoin, with the Most commonly abused substance. Alcohol
characteristic Maltese cross pattern. Organs systems most affected in CNS, GIT and liver
Etiologic agent of a parasitic disease Leishmania sp. acute alcohol intoxication.
transmitted by sandflies; has Thermal injuries
cutaneous and visceral forms; Most common thermal injury; most Thermal burns
parasites in macrophages are common causes are fire and scalding.
characteristic. Common causes of death in burn Shock, sepsis and
Etiologic agent of a parasitic disease Trypanosoma brucei patients. respiratory
transmitted by Glossina sp. (Tse-Tse insufficiency
fly); has hemolymphatic and cerebral Most common hyperthermia Heat exhaustion
phases; diagnosis relies on syndrome.
demonstration of trypanosomes on Electrical injury
blood smears.
TOPNOTCH MEDICAL BOARD PREP PATHO SUPERTABLE Page 10 of 47
For inquiries visit www.topnotchboardprep.com.ph or https://www.facebook.com/topnotchmedicalboardprep/
TOPNOTCH MEDICAL BOARD PREP PATHO SUPERTABLE by KEVIN ELOMINA, MD
For inquiries visit www.topnotchboardprep.com.ph or
https://www.facebook.com/topnotchmedicalboardprep/
Two forms of injury caused by Cardiac/neurologic neuropathy.
electrical injury. (arrhythmias and Deficiency of this trace element leads Zinc
cardiac and to a condition, characterized by
respiratory failure), periorificial distribution of lesions,
and burns called acrodermatitis enteropathica.
Radiation injury
Clinical consequences of ionizing Cell death, CHAPTER 9: DISEASES OF INFANCY AND CHILDHOOD
radiation. teratogenesis, and QUESTION ANSWER
carcinogenesis These represent primary errors of Malformation
NUTRITIONAL DISORDERS morphogenesis due to an intrinsically s
Protein-energy malnutrition abnormal developmental process; example:
PEM caused by a decrease in both Marasmus Polydactyly
protein and calorie intake, and These result from secondary destruction of Disruptions
weight falls below 60% of normal for an organ or body region that was previously
sex, height and age. Serum albumin normal in development, due to an extrinsic
is normal or slightly reduced. disturbance in morphogenesis; example:
PEM, which occurs when protein Kwashiorkor Amniotic bands
deprivation is relatively greater than These are due to generalized compression Deformations
reduction in total calories; of the growing fetus by abnormal
associated with generalized edema biomechanical forces; example: Uterine
(due to hypoalbuminemia), flaky constraint
paint appearance of the skin, This refers to multiple congenital anomalies Sequence
alternating bands of pale and dark that result from secondary effects of a single
hair, enlarged fatty liver, apathy, localized aberration in organogenesis. The
listlessness and loss of appetite; initiating event may be a malformation,
there is also mucosal atrophy and deformation or disruption; example:
loss of villi in the small bowel Oligohydramnios (Potter) sequence
(Making them lactose intolerant Complete absence of an organ or its anlage. Agenesis
initially). Incomplete development of an organ. Aplasia
Self-induced starvation, resulting in Anorexia nervosa Underdevelopment of an organ. Hypoplasia
marked weight loss; has the highest Absence of an opening, usually of a hollow Atresia
death rate of any psychiatric visceral organ.
disorder. Most common cause of neonatal mortality. Congenital
A condition wherein a patient binges Bulimia anomalies
on food and then induces vomiting; Most common cause of congenital Unknown (40-
more common than anorexia malformation in humans. 60%)
nervosa. Most common known cause of congenital Multifactorial
Deficiency of this vitamin causes Vitamin A malformations in humans. (20-25%)
night blindness, xerophthalmia, Most common genetic cause of congenital Chromosomal
keratomalacia, Bitot spots, and malformations in humans. aberrations
eventually, corneal ulceration. Elements of the TORCH complex. Toxoplasma,
Others
(Treponema
pallidum),
Rubella,
Cytomegalovir
us,
Herpesvirus
Condition in children as result of Rickets Second most common cause of neonatal Prematurity
vitamin D deficiency causing the mortality.
following signs and symptoms: Histologically characterized by eosinophilic Respiratory
Craniotabes (parietal bones buckle membranes line the respiratory distress
inward with pressure), frontal bronchioles, alveolar ducts and random syndrome of
bossing of the head, rachitic rosary, alveoli, which contain necrotic epithelial the newborn
pigeon breast deformity, lumbar cells, admixed with extravasated plasma (Hyaline
lordosis, and bow legs. proteins (Diffuse alveolar damage). membrane
Deficieny of this vitamin causes Vitamin C disease)
impaired collagen formation, leading Main component of pulmonary surfactant; Dipalmitoylph
to gum bleeding, easy bruisability, which is insufficient in hyaline membrane osphatidylcho
and impaired wound healing. disease. line (DPPC)
Deficiency of this vitamin leads to Vitamin D A complication of hyperoxic therapy in Retinopathy
rickets and osteomalacia. patients with RDS, resulting in of
Symptoms of cheilosis, stomatitis, Vitamin B2 neovasculatization of retinal vessels, prematurity
glossitis, dermatitis, and corneal deficiency leading to blindness. (ROP)/Retrol
vascularization results from ental
deficiency of this vitamin. fibroplasia
Symptoms of cheilosis, glossitis, Vitamin B6 A complication of hyperoxic injury in Bronchopulm
dermatitis, and peripheral deficiency patients with RDS, resulting in alveolar onary
TOPNOTCH MEDICAL BOARD PREP PATHO SUPERTABLE Page 11 of 47
For inquiries visit www.topnotchboardprep.com.ph or https://www.facebook.com/topnotchmedicalboardprep/
TOPNOTCH MEDICAL BOARD PREP PATHO SUPERTABLE by KEVIN ELOMINA, MD
For inquiries visit www.topnotchboardprep.com.ph or
https://www.facebook.com/topnotchmedicalboardprep/
hypoplasia or a decrease in the number of dysplasia CHAPTER 10: BLOOD VESSELS
mature alveoli; morphologically, there is (BPD) QUESTION ANSWER
large, simplified alveolar structures, and a Congenital anomalies
dysmorphic capillary configuration. These are small spherical Developmental/berry
This condition occurs more commonly in Necrotizing dilatations, typically in the circle of aneurysms
very-low-birth-weight infants, as a result of enterocolitis Willis.
intestinal ischemia, bacterial colonization of (NEC) These are abnormal, typically small, Arteriovenous
gut and formula milk feeding; histologically, direct connections between arteries fistulas
there is presence of submucosal gas and veins that bypass the
bubbles, transmural coagulative necrosis, intervening capillaries.
ulceration, and bacterial colonization. Focal, irregular thickening of the Fibromuscular
Results from antibody-induced hemolytic Immune walls of medium and large dysplasia
disease in the newborn that is caused by hydrops muscular arteries; segments of the
blood group incompatibility between vessel wall are focally thickened by
mother and fetus, leading to edema fluid combination of irregular medial
accumulation; examples: ABO and Rh and intimal hyperplasia and
incompatibility fibrosis, causing luminal stenosis.
Hydrops caused by cardiovascular defects, Nonimmune Hypertension
chromosomal anomalies, and fetal anemia. hydrops Consists of a homogenous pink Hyaline
Inherited disorder of ion transport that Cystic fibrosis hyaline thickening of the walls of arteriolosclerosis
affects fluid secretion in exocrine glands (CF) arterioles with loss of underlying
and in the epithelial lining of the structural detail and with
respiratory, gastrointestinal, and narrowing of the lumen; major
reproductive tracts; most common lethal morphologic characteristic in
genetic disease affecting Caucasian benign nephrosclerosis.
populations; involved gene is CFTR-1 in Characteristic of malignant Hyperplastic
Ch7q31.2. hypertension, associated with artieriolosclerosis
Defined as the sudden death of an infant Sudden infant "onion-skin" concentric, laminated,
under 1 year of age which remains death thickening of the walls of arterioles
unexplained after a thorough investigation; syndrome with luminal narrowing; these
characteristic findings include: multiple (SIDS) laminations consist of smooth
petechiae of the thymus, visceral and muscle cells and thickened
parietal pleura and epicardium, congested duplicated basement membrane;
lungs with vascular engorgement with or associated with necrotizing
without pulmonary edema; there is also arteriolitis.
hypoplasia of the arcuate nucleus. Atherosclerosis
Tumors arising from neural crest cells; Neuroblastom Characterized by intimal lesions Atherosclerosis
composed of small, primitive-appearing a (Classically called atheromas that protrude into
cells with dark nuclei, scant cytoplasm, and described as a vascular lumina.
poorly defined cell borders growing in solid mass crossing Composed of lipid-filled Fatty streaks
sheets within a finely fibrillar matrix the midline, macrophages and smooth muscle
(neuropil: eosinophilic fibrillary material vs. Wilms cells (foam cells) but are not
that corresponds to neuritic processes of tumor that significantly raised and thus do not
neuroblasts); associated with Homer- does not cross cause any disturbance in blood
Wright pseudorosettes. the midline) flow; can appear as early as 1 year,
Most common primary tumor of the kidney Wilms tumor and present in virtually all children
in children; second most common primary older than 10 years old.
tumor of the kidney overall (next to RCC); Three principal components of an 1. Cells (SM cells,
presents grossly as a large, solitary, well- atheromatous plaque. macrophages, T cells)
circumscribed mass; microscopically, a 2. Extracellular
combination of blastemal, stromal and matrix (collagen,
epithelial cell types is observed (Triphasic elastic fibers,
combination); blastemal components are proteoglycans); and
described as sheets of small blue cells with 3. Intracellular and
few distinctive features; stromal cells are extracellular lipid
fibrocytic or myxoid in nature; epithelial Most common site of Abdominal aorta
cells take the form of abortive tubules or atherosclerosis.
glomeruli. Plaque that has a thick fibrous cap, Stable plaque
minimal lipid core, and minimal
inflammation.
Plaque that has a thin fibrous cap, Vulnerable plaque
large lipid core, and greater
inflammation.
Fates of an atheromatous plaque
The luminal surface exposes the Rupture, ulceration
bloodstream to highly or erosion
thrombogenic substances and
PART II: SYSTEMIC PATHOLOGY induces thrombus formation.
TOPNOTCH MEDICAL BOARD PREP PATHO SUPERTABLE Page 12 of 47
For inquiries visit www.topnotchboardprep.com.ph or https://www.facebook.com/topnotchmedicalboardprep/
TOPNOTCH MEDICAL BOARD PREP PATHO SUPERTABLE by KEVIN ELOMINA, MD
For inquiries visit www.topnotchboardprep.com.ph or
https://www.facebook.com/topnotchmedicalboardprep/
Rupture of the overlying fibrous Hemorrhage granulomatous inflammation of the
cap or the thin-walled vessels in the internal elastic lamina with
areas of neovascularization, results infiltrate of T cells and
in: macrophages. The most likely
Discharge of debris into the Atheroembolism diagnosis is:
bloodstream, producing Patient initially present with Takayasu arteritis
microemboli composed of plaque fatigue, weight loss and fever and (Large arteries)
contents. then reduced blood pressure and
weak pulses in the carotids and
Increased pressure or ischemic Aneurysm formation upper extremities, ocular
atrophy of the underlying media, disturbances and neurologic
with loss of elastic tissue, leading to deficits. The most likely diagnosis
weakness of the vessel wall. is:
Critical stenosis in coronary 70% Note: Both vasculitides affect large arteries and exhibit
arteries granulomatous type of inflammation. In fact, they have the
Aneurysms and Dissection same histologic appearance. It depends on the clinical data to
It is a localized abnormal dilation of Aneurysm differentiate both diseases.
a blood vessel or heart.
Aneurysm that involves all three True aneurysm
layers of the arterial wall, or the This disease presents with rapidly Polyarteritis nodosa
attenuated wall of the heart. accelerating hypertension, (Small to medium-
A breach in the vascular wall False aneurysm abdominal pain, bloody stool, sized arteries)
leading to an extravascular myalgia and peripheral neuritis;
hematoma that freely histologically, there is segmental Remember: PAN can
communicates with the transmural necrotizing be renal or visceral,
intravascular space. inflammation that spares the but never pulmonary
Two most important causes of Atherosclerosis (AAA) pulmonary circulation with
aortic aneurysms. and hypertension extensive fibrinoid necrosis and
(Ascending aortic temporal heterogeneity of lesions.
aneurysm) The most likely diagnosis is:
This disease can more commonly Abdominal aortic 4/F presented with fever and Kawasaki disease
affects men >50 years old. Lesion aneurysm (AAA) maculopapular rash on her trunk (Medium-sized)
usually positioned below the renal and extremities. This is associated
arteries and above the aortic with conjunctival erythema, edema
bifurcation; can be saccular of the hands and feet, and cervical
(dilation of one portion) or fusiform lymph node enlargement; on
(circumferential dilation). biopsy, there is segmental
Small blood vessels and vasa Syphilitic aortitis transmural necrotizing
vasorum show luminal narrowing inflammation with less fibrinoid
and obliteration (Obliterative necrosis. The most likely diagnosis
endarteritis) scarring of the vessel is:
wall and a dense surrounding rim Note: Both vasculitides that affect medium-sized arteries have
of lymphocytes and plasma cells the same histologic appearance; only Kawasaki disease has
that may extend into the media; less fibrinoid necrosis. It depends on the clinical data to
characteristic of the tertiary stage differentiate both diseases.
of Syphilis. A necrotizing vasculitis that Microscopic
Arises when blood enters the wall Arterial dissection generally affects capillaries, polyangiitis (Small
of an artery, as a hematoma arterioles and venules, with few or arteries)
dissecting between its layers; often, no immune deposits; necrotizing
but not always aneurysmal in glomerulonephritis and pulmonary Memory device: PAN
origin. capillaritis are common; associated in small arteries that
Most frequent pre-existing Cystic medial with p-ANCA (MPO-ANCA); also are temporally
histologically detectable lesion in degeneration exhibits leukocytoclasia homogenous (lesions
aortic dissection; characterized by (fragmented PMNs in post-capillary are of same age)
elastic tissue fragmentation and venules); thus also the term
separation of the elastic and leukocytoclastic angiitis.
smooth muscle cell elements of the Eosinophil-rich and granulomatous Churg-Strauss
media by cystic spaces filled with inflammation involving the syndrome (Small
amorphous proteoglycan-rich respiratory tract and necrotizing arteries)
extracellular matrix. vasculitis affecting small vessels;
Vasculitides ssociated with asthma and blood
38/M presents with severe Giant cell arteritis eosinophilia; associated with p-
headache and diplopia associated (Large arteries) ANCA (MPO-ANCA).
with fever and fatigue. Area along Causes necrotizing granulomatous Wegener
the course of the superficial vasculitis with fibroblastic granulomatosis
temporal artery was painful to proliferation; Involves small vessels
palpation. Biopsy done revealed most notable in the upper and
intimal thickening of the artery, lower r; can cause crescentic
TOPNOTCH MEDICAL BOARD PREP PATHO SUPERTABLE Page 13 of 47
For inquiries visit www.topnotchboardprep.com.ph or https://www.facebook.com/topnotchmedicalboardprep/
TOPNOTCH MEDICAL BOARD PREP PATHO SUPERTABLE by KEVIN ELOMINA, MD
For inquiries visit www.topnotchboardprep.com.ph or
https://www.facebook.com/topnotchmedicalboardprep/
glomerulonephritis; associated with Tumors
c-ANCA (PR3-ANCA). Benign and tumor-like conditions
Characterized by sharply segmental Thromboangiitis Most common form of vascular Nevus flammeus
acute and chronic vasculitis of obliterans (Buerger ectasia. (birthmark)
medium sized and small arteries, disease) Radial, often pulsatile arrays of Spider telangiectasia
predominantly of the extremities; dilated subcutaneous arteries about
there is acute and chronic Note: Do not confuse a central core that blanch with
inflammation accompanied by with Berger disease pressure; associated with
luminal thrombosis, containing (IgA nephropathy) hyperestrinism (pregnancy and
microabscesses composed of liver cirrhosis).
neutrophils surrounded by 3/F with a large "port wine stain" A (Diagnosis is
granulomatous inflammation; on the right side of her face has Sturge-Weber
strong relationship with cigarette occasional seizures. Further syndrome or
smoking. examination showed mental encephalotrigeminal
Disorders of blood vessel hyperreactivity developmental delay. She may also angiomatosis)
Results from an exaggerated Raynaud have:
vasoconstriction of digital arteries Phenomenon A. More extensive vascular
and arterioles, inducing paroxysmal malformations
pallor or cyanosis of the digits of B. Multiple colonic polyps
the hands and feet; can be primary C. Color blindness
or secondary. D. Alpha thalassemia
An autosomal dominant disorder Hereditary
caused by mutations in genes that hemorrhagic
encode components of TGF-β telengiectasia/Osler-
signaling pathway. Presents with Weber-Rendu disease
telengiectasia that are widely
distributed over the skin and mucus
membranes.
These are bright red to blue lesions, Capillary
Disorders of veins and lymphatics that vary from a few millimeters to hemangiomas
Abnormally dilated, tortuous veins Varicose veins several centimeters in diameter; blood filled and lined with
produced by prolonged increase in Unencapsulated aggregates of flattened enfothelium
intraluminal pressure and loss of closely packed, thin-walled
vessel wall support; veins show capillaries, usually blood-filled and
wall thinning at the points of lined by flattened endothelium;
maximal dilation with smooth vessels are separated by scant
muscle hypertrophy and intimal connective tissue stroma; can also
fibrosis; focal thrombosis and be filled with lymph
venous valve deformities are (lymphangioma)
common; most common sites are Appears as red-blue, soft, spongy Cavernous
superficial veins of the upper and masses 1-2 cm in diameter, which hemangioma
lower leg. can affect large subcutaneous areas
Three sites of varices produced in GEJ (Esophageal of the face, extremities, and other
the presence of portal varices), Rectum body regions; mass is sharply
hypertension. (Hemorrhoids), defined but not encapsulated,
Periumbillical veins composed of large, cavernous,
(Caput medusae) blood-filled spaces; can also be
Usually arises in the setting of Superior vena cava filled with lymph (lymphangioma).
bronchogenic carcinoma, syndrome Borderline malignant lesions
mediastinal lymphoma, and other Common in patients with AIDS, Kaposi sarcoma
SOLs (aneurysms); usually presents caused by HHV-8, causing skin
with dilation of the veins of the lesions that progress from patches,
head, neck, and arms with cyanosis; to plaques, to nodules; there are
respiratory distress occurs with usually plump spindle cells with
pulmonary vessel involvement. extravasated erythrocytes and
Usually arises in the setting of Inferior vena cava hemosiderin-laden magrophages in
tumors that invade IVC syndrome the lesions
(hepatocellular and renal), and Malignant tumors
venous thrombosis; usually present These are malignant endothelial Angiosarcoma
with Lower extremity edema, neoplasms, with varying histology
superficial abdominal vein from plump, anaplastic but
distention, massive proteinuria recognizable endothelial cells
with renal vein involvement. producing vascular channels to
Most common cause of GABHS widely undifferentiated tumors
lymphangitis (usually with having solid, spindle cell
accompanying lymphadenitis). appearance and producing
Note: Venous thrombosis discussed in Hemodynamic nondefinite blood vessels;
Disorders. endothelial cell markers include
TOPNOTCH MEDICAL BOARD PREP PATHO SUPERTABLE Page 14 of 47
For inquiries visit www.topnotchboardprep.com.ph or https://www.facebook.com/topnotchmedicalboardprep/
TOPNOTCH MEDICAL BOARD PREP PATHO SUPERTABLE by KEVIN ELOMINA, MD
For inquiries visit www.topnotchboardprep.com.ph or
https://www.facebook.com/topnotchmedicalboardprep/
CD31 and vWF. close
Main determinant of clinical Size (<5mm: clinically
CHAPTER 11: THE HEART outcome in isolated VSDs. asymptomatic;
QUESTION ANSWER >10mm: clinically
Heart failure with failure to thrive
The left ventricle is hypertrophied Left-sided heart and repeated
and dilated, with secondary left failure infections)
atrial dilation; the lungs are heavy 5/w infant presents with Patent ductus
and boggy, with perivascular and tachypnea, diaphoresis, and arteriosus (PDA)
interstitial transudate, alveolar difficulty feeding; a harsh,
septal edema, and intra-alveolar continuous machinery-like murmur
edema; hemosiderin-laden was noted upon auscultation. The
macrophages (heart failure cells) most likely diagnosis is:
are present; most common cause of The reason why PDA in preterm PDA in term lacks
right-sided heart failure. infants has a greater chance of mucoid endothelium
Isolated right-sided HF occurring in Cor Pulmonale spontaenous closure than in term and muscular media
patients with intrinsic lung disease infants.
that result in chronic pulmonary Drug that can maintain patency of PGE2
hypertension. ductus arteriosus; vital in patients
Long standing severe right-sided Cardiac cirrhosis with duct-dependent lesions.
HF leads to fibrosis of centrilobular Cyanotic CHDs
areas, creating this condition. The most common cause of Tetralogy of Fallot
Congenital heart diseases cyanotic congenital heart disease. (TOF)
Most CHDs arise in this period. 3-8 weeks AOG Heart is large and "boot-shaped" as
Most common genetic cause of Trisomy 21 a result of right ventricular
CHDs. hypertrophy; components are: PS,
Acyanotic CHDs RVH, VSD; and aorta overriding the
Common radiographic feature of Increased pulmonary VSD.
acyanotic CHDs. blood flow Main determinant of clinical Degree of PS
Feature that will distinguish PDA Prominent aortic outcome in TOF patients.
from VSD radiographically. knob (because both It is a discordant connection of the Transposition of the
show LVH/BVH) ventricles to their vascular outflow; Great Arteries (TGA)
Chamber that enlarges in ASD. Right ventricle defect is an abnormal formation of
Smooth-walled defect near the Ostium secundum the truncal and aortopulmonary
foramen ovale, usually without atrial septal defect septa; common in infants of
associated cardiac abnormalities; diabetic mothers and in males;
comprises 90% of ASDs; regardless radiographically, "egg on the side"
of type, most common congenital appearance
heart disease in adults. Absence of direct connection Tricuspid atresia
Natural history of ASDs. Spontaneous closure between RA and RV, due to unequal
for small to division of the AV canal.
moderate-sized ASDs Obstructive lesions
Characterized by circumferential Preductal "infantile"
narrowing of the aortic segment Coarctation of the
between the left subclavian artery aorta
and the ductus arteriosus; DA is
usually patent and is the main
source of blood to the distal aorta;
RV is hypertrophied and dilated;
Incomplete closure of the Ventricular septal pulmonary trunk is also dilated;
ventricular septum leading to left- defect (VSD) clinically presents as lower
to-right shunting; the right extremity cyanosis.
ventricle is hypertrophied and
often dilated; diameter of
pulmonary artery is increased
because of the increased volume by
the right ventricle; most common
congenital heart disease overall;
most common type is
perimembranous (90%). Aorta is sharply constricted by a Postductal "adult"
Natural history of VSDs. Small defects usually ridge of tissue at or just distal to the Coarctation of the
close spontaneously nonpatent ligamentum arteriosum; aorta
(common: 1st 2 YOL); constricted segment is made of
Vast majority of smooth muscle and elastic fibers
lesions that close do that are continuous with the aortic
so before age 4; media, and lined by thickened
moderate to large intima; DA is closed; proximally, the
VSDs are less likely to aortic arch and its vessels are
TOPNOTCH MEDICAL BOARD PREP PATHO SUPERTABLE Page 15 of 47
For inquiries visit www.topnotchboardprep.com.ph or https://www.facebook.com/topnotchmedicalboardprep/
TOPNOTCH MEDICAL BOARD PREP PATHO SUPERTABLE by KEVIN ELOMINA, MD
For inquiries visit www.topnotchboardprep.com.ph or
https://www.facebook.com/topnotchmedicalboardprep/
dilated; LV is hypertrophic; there is
upper extremity hypertension, due Cause of acute MR post MI. Papillary muscle
to poor perfusion of the kidneys, dysfunction (more
but weak pulses and low blood common); papillary
pressure in the lower extremities; muscle rupture
claudication and coldness of the This refers to progressive cardiac Chronic ischemic
lower extremities also present; decompensation (heart failure) heart disease
enlarged intercostal and internal following myocardial infaction.
mammary arteries due to collateral Typical morphology of chronic IHD. Left ventricular
circulation, seen as rib "notching" dilation and
on X-ray. hypertrophy (Gross)
Ischemic heart disease Myocardial
A condition wherein ischemia Angina pectoris hypertrophy, diffuse
causes pain but is insufficient to subendocardial
lead to death of myocardium; can vacuolization and
be stable (secondary to increased fibrosis (Microscopic)
demand), unstable (crescendo; This can result from a lethal Sudden cardiac death
secondary to decreased supply), or arrhythmia following myocardial (SCD)
Prinzmetal (secondary to coronary ischemia; most common cause is
vasospasm). CAD (80%); most common
A condition wherein the severity or Acute myocardial mechanism is a lethal arrhythmia
duration of ischemia is enough to infarction (AMI) (asystole or ventricular fibrillation).
cause cardiac muscle death. Hypertensive heart disease
Pattern of infarction caused by Transmural Typical morphology of systemic Left ventricular
occlusion of an epicardial vessel, infarctions hypertensive heart disease. hypertrophy
through a combination of chronic Earliest change seen in left-sided Increased transverse
atherosclerosis and acute HHD. diameter of myocytes
thrombosis; also referred to as Typical morphology of acute cor Right ventricular
STEMI or Q wave MI; infarct pulmonale (pulmonary dilation
involving ≥50% of the myocardial hypertensive heart disease).
wall thickness. Typical morphology of chronic cor Right ventricular
Pattern of infarction caused by Subendocardial pulmonale (pulmonary (and often right
plaque disruption or hypotension infarction hypertensive heart disease). atrial) hypertrophy
leading to transient decreases in Valvular heart disease
oxygen delivery, causing The hallmark of this disease is Calcific aortic
circumferential myocardial heaped-up calcified masses on the degeneration/calcific
damage; also referred to as NSTEMI outflow side of the cusps, which aortic stenosis (most
or non-Q wave MI. protrude into the sinuses of common VHD)
Heart dominance is determined by: Vessel that gives rise Valsalva and mechanically impede
to the posterior IV valve opening; cusps may become
(descending) branch; secondarily fibrosed and thickened;
either RCA (right most common cause of aortic
dominant) or LCX stenosis.
(left dominant); most This is characterized by ballooning Myxomatous
common: RCA (90%) or hooding of the mitral leaflets; degeneration of the
Most common vessel involved in Left anterior affected leaflets are enlarged, mitral valve
AMI: descending artery redundant, thick and rubbery; the
(LAD) (40-50%) tendinous cords are elongated,
Note: Table 10-3 in Robbins Basic Pathology, 9th ed. p. 380 or thinned and occasionally ruptured;
Table 12-5 in Robbins and Cotran Pathologic Basis of Disease, histologically, there is thinning of
8th ed. p. 550, or Table 12-5 in Robbins and Cotran Pathologic the fibrosa layer of the valve,
Basis of Disease, 9th ed. p. 544, is a high-yield table. Make sure accompanied by expansion of the
you know this by heart. middle spongiosa layer with
Stain used for infarcts more ~2-3 Triphenyltetrazolium increased deposition of mucoid
hours old for visualization. chloride material; usual finding in MVP.
Most sensitive and specific cardiac Cardiac troponins Rheumatic fever and Rheumatic heart disease
biomarkers: (Troponin I and T) Rationale behind autoimmunity in Molecular mimicry
Injury to infarcts mediated in part Reperfusion injury rheumatic fever. (Streptococcal M
by oxygen free radicals generated proteins and cardiac
by increased number of infiltrating self-antigens)
leukocytes facilitated by Gell and Coombs classification of Type II
reperfusion; CK-MB is usually used ARF.
as cardiac biomarker for diagnosis. Required criteria in diagnosing Evidence of previous
Consequence of MI rheumatic fever. GABHS infection (ASO
Most common site of myocardial Ventricular free wall in pharyngitis, and
rupture. (clinically, cardiac anti DNAse B in
tamponade) pyoderma)
Fibrinous pericarditis post-MI. Dressler syndrome Jones major criteria in diagnosing Carditis
TOPNOTCH MEDICAL BOARD PREP PATHO SUPERTABLE Page 16 of 47
For inquiries visit www.topnotchboardprep.com.ph or https://www.facebook.com/topnotchmedicalboardprep/
TOPNOTCH MEDICAL BOARD PREP PATHO SUPERTABLE by KEVIN ELOMINA, MD
For inquiries visit www.topnotchboardprep.com.ph or
https://www.facebook.com/topnotchmedicalboardprep/
rheumatic fever. Migratory endocardium.
polyarthritis The lesions of this disease are Carcinoid heart
Subcutaneous distinctive, glistening white intimal disease
nodules thickenings on the endocardial
Erythema surfaces of the cardiac chambers
marginatum and valve leaflets; lesions are
Syndenham chorea composed of smooth muscle cells
and sparse collagen fibers
embedded in an acid
mucopolysaccharide-rich matrix;
usually right-sided (causing
Jones minor criteria in diagnosing Fever tricuspid insufficiency and
rheumatic fever. Arthralgia pulmonic stenosis).
Elevated acute phase
reactants (e.g. CRP)
Pathognomonic sign for rheumatic Aschoff bodies
fever; myocardial inflammatory
lesions composed of plump
activated macrophages
(Anitschkow cells), plasma cells,
and lymphocytes; can be seen in all
three layers of the heart in ARF Cardiomyopathies
(pancarditis); rarely seen in chronic Cardiomyopathy characterized by Dilated
RHD (due to replacement by fibrous progressive cardiac dilation and cardiomyopathy
tissue). systolic dysfunction; heart is
Characterized by organization and Chronic Rheumatic characteristically enlarged and
subsequent scarring, as a sequelae Heart Disease flabby, with dilation of all
of rheumatic fever; mitral valve chambers; the ventricular thickness
most commonly involved with may be less than, equal to or
leaflet thickening, commissural greater than normal; characterized
fission and shortening, thickening by systolic dysfunction; implicated
and fusion of the chordae tendinae; agents and diseases include:
fibrous bridging across valvular alcohol, myocarditis, doxorubicin,
commissures create "fish mouth" or and hemochromatosis.
" buttonhole" deformity Most common mutated gene in Titin (20%)
Serious infection characterized by Infective endocarditis cases of DCM.
microbial invasion of heart valves (IE) Type of DCM secondary to an Arrhythmogenic right
or mural endocardium, often with autosomal dominant genetic defect ventricular
destruction of the underlying on desmosomes that result in right cardiomyopathy
cardiac tissues; the heart valves are ventricular failure with arrhythmia (ARVC)
invaded by friable, bulky and and sudden death.
potentially destructive lesions Characterized by myocardial Hypertrophic
called vegetations that are usually hypertrophy, abnormal diastolic cardiomyopathy
in the form of large, irregular filling and ventricular outflow
masses on the valve cusps that may obstruction; the heart is thick-
extend into chordae. walled, heavy and
Endocarditis of previously normal Acute bacterial hypercontracting; there is an
valves, the most common causative endocarditis asymmetrical septal hypertrophy
agent is S. aureus. described as "banana-like";
Endocarditis affecting previously Subacute bacterial histologically, there is severe
damaged or abnormal valves, endocarditis myocyte hypertrophy and disarray
commonly caused by viridans with interstitial fibrosis; a common
streptococci. cause of sudden death in young
Clinical criteria used in diagnosing Duke criteria athletes.
infective endocarditis. Most common mutated gene in β-myosin heavy chain
Characterized by deposition of Nonbacterial cases of HCM.
sterile, non-inflammatory, thrombotic The ventricles are of approximately Restrictive
nondestructive and small (1mm) endocarditis (NBTE) normal size or slightly enlarged, the cardiomyopathy
masses of fibrin, platelets and other cavities not dilated, and the
blood components on cardiac myocardium is firm; biatrial
valves, along the line of closure of dilation is common;
leaflets or cusps. microscopically, there is interstitial
Sterile vegetations that can develop Libman-Sacks fibrosis, varying from minimal to
on the valves of patients with SLE; endocarditis (LSE) patchy to extensive and diffuse;
small, granular, pinkish vegetations characterized by diastolic
1-4mm in diameter and can be dysfunction.
located on the undersurface of AV Myocarditis
valves, on the cords or Most common cause of myocarditis. Viral (Coxsackie A, B,
TOPNOTCH MEDICAL BOARD PREP PATHO SUPERTABLE Page 17 of 47
For inquiries visit www.topnotchboardprep.com.ph or https://www.facebook.com/topnotchmedicalboardprep/
TOPNOTCH MEDICAL BOARD PREP PATHO SUPERTABLE by KEVIN ELOMINA, MD
For inquiries visit www.topnotchboardprep.com.ph or
https://www.facebook.com/topnotchmedicalboardprep/
enteroviruses) commonly seen in advanced HIV and
Parasitic etiologies of myocarditis. T. cruzi (Chagas other diseases.
disease), Trichinella Clinically significant neutropenia Agranulocytosis
spiralis (most (<500/mm3); highly susceptible to
common helminthic) infections (Candida and Aspergillus);
Immune causes of myocarditis. Hypersensitivity, most common cause: drug toxicity
giant-cell myocarditis An increase in the number of white cells Reactive
Pericardial diseases in the blood in a variety of inflammatory leukocytosis,
Type of pericarditis found in Fibrinous pericarditis states caused by microbial and Leukemoid
patients with uremia or viral nonmicrobial stimuli that may mimic reaction (high
infection; the exudate imparts an leukemia. leukocyte
irregular appearance to the alkaline
pericardial surface (bread and phosphatase, a
butter pericarditis). product of
Bacterial pericarditis manifests Fibrinopurulent normal WBCs;
with this type of exudate. pericarditis used to
Heart is completely encased by Constrictive differentiate it
dense fibrosis that it cannot expand pericarditis from leukemias)
normally during diastole; sequela of Enlargement of a lymph node as immune Reactive
chronic pericarditis. response to foreign antigens; histology lymphadenitis
Abnormal fluid accumulation in the Pericardial effusion usually nonspecific; depends on duration
pericardial sac; can be serous (from of disease and type of offending agent.
CHF or hypoalbuminemia); Neoplastic white cell disorders
serosanguinous (from blunt chest Lymphoid neoplasms
trauma, malignancy, ruptured MI, Most common type of cancer in children; Acute
or aortic dissection), or chylous highly aggressive tumors manifesting lymphoblastic
(from mediastinal lymph node with signs and symptoms of bone leukemia (ALL)
obstruction). marrow failure, marrow expansion,
dissemination of leukemic cells, and CNS
manifestations; lymphoblasts with
irregular nuclear contours, condensed
Tumors chromatin, small nucleoli and scant
Most common tumor of the heart. Metastatic tumor agranular cytoplasm on BMA; blasts
Most common primary tumor of the Myxoma compose >25% of marrow cellularity;
adult heart; major clinical TdT(+) in 95% of cases; most responsive
manifestations of this cardiac to chemotherapy (Asparaginase).
tumor are due to valvular "ball-
valve" obstruction, embolization or
a syndrome of constitutional
symptoms.
The most frequent primary tumor Rhabdomyoma
of the heart in infants and children; Clinical differences between B- and T-cell B-cell ALL
generally small gray-white ALL. typically occurs
myocardial masses composed of a in younger
mixed population of cells, the most b-all children with BM failure children
t-all adolescent male presenting with
characteristic of which are large,
rounded or polygonal cells BM failure
containing numerous glycogen- T-cell ALL
laden vacuoles separated by typically occurs
strands of cytoplasm, so-called in adolescent
spider cells. males
Cardiac transplantation presenting with
Most common indications for DCM and IHD thymic masses
cardiac transplantation. Good prognostic factors in ALL. Children 2-10
Primary problem requiring Allograft rejeciton years old;
surveillance. t(12;21) and
Single most important long-term Allograft arteriopathy hyperdiploidy
limitation for cardiac Poor prognostic factors in ALL. Male gender;
transplantation. (The genetic abnormalities are seen in age younger
the given age group) than 2 or older
CHAPTER 12: DISEASES OF THE WHITE BLOOD CELLS, than 10 years; a
LYMPH NODES, SPLEEN, AND THYMUS high leukocyte
QUESTION ANSWER count at
diagnosis; and
Non-neoplastic white cell disorders
molecular
Decreased number of circulating Leukopenia
evidence of
leukocytes; most commonly neutrophils
persistent
(neutropenia); deficiency of lymphocytes
disease on day
(lymphopenia) is less common, and is
TOPNOTCH MEDICAL BOARD PREP PATHO SUPERTABLE Page 18 of 47
For inquiries visit www.topnotchboardprep.com.ph or https://www.facebook.com/topnotchmedicalboardprep/
TOPNOTCH MEDICAL BOARD PREP PATHO SUPERTABLE by KEVIN ELOMINA, MD
For inquiries visit www.topnotchboardprep.com.ph or
https://www.facebook.com/topnotchmedicalboardprep/
28 of treatment, hyperviscosity syndrome (Waldenstrom
t(9;22) and MLL macroglobulinemia (IgM)): visual
rearrangements changes, neurologic problems, bleeding
Most common leukemia of adults in Chronic diathesis and cryoglobulinemia; no bone
Western world; chronic leukemia lymphocytic manifestations; renal manifestations rare.
associated with BCL2, an antiapoptotic leukemia/Small Lymphomas of memory B-cell origin; Marginal zone
molecule; patient presents with increased lymphocytic usually arises on tissues on chronic lymphoma
susceptibility to infections due to lymphoma inflammation; regresses when inciting
hypogammaglobulinemia; CBC showed (CLL/SLL) agent is removed, but once it progresses
>5000 lymphocytes/mm3; histologically, to DLBCL, regression may not be
foci of mitotically active cells SLL: <5000 possible; examples: salivary glands:
(proliferation centers) are present; also, lymphocytes/m Sjogren syndrome, thyroid gland:
smudge cells (due to fragility of m 3 Hashimoto thyroiditis, stomach: H. pylori
circulating tumor cells) are also evident; gastritis
can transform into DLBCL (Richter These tumors of neoplastic CD4+ T cells Mycosis
syndrome). home to the skin; usually manifests as a fungoides
Most common indolent lymphoma of Follicular nonspecific erythrodermic rash that (Cutaneous T-
adults; frequent small "cleaved" cells lymphoma progresses over time to a plaque phase cell lymphoma)
mixed with large cells, growth pattern is and then to a tumor phase; histologically,
nodular, centroblasts present; occurs in neoplastic T cells, often with a
older adults, usually involves nodes, cerebriform appearance produced by
marrow, spleen; associated with t(14;18) marked infolding of the nuclear
that results in overexpression of cyclin membranes, infiltrate the epidermis and
D1. upper dermis.
most common lymphoma of adults; most Diffuse Large B- Hodgkin lymphomas
common form of NHL; tumor cells have cell lymphoma Diagnostic cells in HL; large, multiple Reed-Sternberg
large nuclei with open chromatin and (DLBCL) nuclei or single with multiple lobes; each (RS) cells
prominent nucleoli; most important type with nucleolus about a size of a small
of lymphoma in adults, accounting to lymphocyte, CD15(+), CD30(+).
~50% of adult NHLs. Classical HL types; CD15 and CD30(+). NS, MC, LR,
17/M presented with a short history of Burkitt LDHL
fever, tonsillitis and unilateral enlarged lymphoma Most common variant of HL; with lacunar Nodular
cervical lymph nodes. PE revealed Reed-Sternberg (RS) cells; with sclerosis HL
enlargement of right cervical lymph node, deposition of collage bands that divide (NSHL)
3 cm in diameter, hard, and pharyngeal lymph node into nodules; not associated
hyperemia. Histologically, there was with EBV; excellent prognosis; usually
intermediate-sized round lymphoid cells diagnosed at early stage (Stage I/II) with
with 2-5 prominent nucleoli. High rates of frequent mediastinal involvement.
proliferation and apoptosis are Mononuclear and diagnostic RS cells with Mixed
characteristic. Nuclear remnants heterogenous cellular infiltrate; cellularity HL
phagocytosed by interspersed associated with EBV in 70% of cases; (MCHL)
macrophages with abundant clear >50% of cases diagnosed at Stage III/IV .
cytoplasm, "starry sky pattern". Also, it is Mononuclear and diagnostic RS cells with Lymphocyte-
associated with cMYC oncogene [t(8;14)]. lymphocytic infiltrate; not associated rich HL (LRHL)
The most likely diagnosis is? Clue: it is with EBV; excellent prognosis.
also the fastest growing human tumor. Lymphocytes are scarce with relative Lymphocyte-
abundance of diagnostic RS cells; depleted HL
associated with EBV in 90% of cases; (LRHL)
associated with PLHIV; worst prognosis.
Lymphohistiocytic "popcorn" RS cells Lymphocyte-
with nodular infiltrate of small predominant
Plasma cell neoplasms lymphocytes admixed with macrophages; (LPHL)
Disease that presents as multifocal Multiple excellent prognosis; CD15 and 30 (-),
destructive bone lesions seen as myeloma CD20(+).
punched-out defects on imaging. Renal Staging system used for HL. Ann-Arbor
involvement is also prominent, causing classification
production of proteinaceous casts in the
DCT and collecting ducts (that can cause
renal insufficiency); can also present with
immunodeficiency due to impaired
normal plasma cell function.
Most common M protein in myeloma IgG Myeloid neoplasms
cells. BMA shows hypercellular marrow packed Acute myeloid
Excess light or heavy chains along with Bence-Jones with ≥20% myeloblasts (and azurophilic leukemia (AML)
complete Igs synthesized by neoplastic protein needle-like material called Auer rods
plasma cells. (faggot cells)); clinically presents with
Basically, CLL/SLL with plasmacytic Lymphoplasmac pancytopenia and bleeding; poor
differentiation; usually present with ytic lymphoma prognosis because it is difficult to treat.
TOPNOTCH MEDICAL BOARD PREP PATHO SUPERTABLE Page 19 of 47
For inquiries visit www.topnotchboardprep.com.ph or https://www.facebook.com/topnotchmedicalboardprep/
TOPNOTCH MEDICAL BOARD PREP PATHO SUPERTABLE by KEVIN ELOMINA, MD
For inquiries visit www.topnotchboardprep.com.ph or
https://www.facebook.com/topnotchmedicalboardprep/
Stains used to differentiate Myeloblasts Myeloblasts: transformation to AML.
from Monoblasts. Myeloperoxidas
e (MPO)(+)
Non-specific CHAPTER 13: RED BLOOD CELL AND BLEEDING
esterase (NSE)(- DISORDERS
) QUESTION ANSWER
Monoblasts: RED BLOOD CELL DISORDERS
MPO(-), NSE(+) Hemolytic anemias
Stains used to differentiate Myeloblasts Lymphoblasts: Characterized by defective red cell Hereditary
from lymphoblasts. MPO(-) PAS(+) membrane proteins spectrin and ankyrin; spherocytosis
Main differences between AML and ALL. AML occurs in on PBS, red cells are spherical which lack (Intrinsic;
adults, CNS central pallor, and they show increased extravascular)
spread is rare, osmotic fragility when placed in
and is more hypotonic salt solutions; anemia with
difficult to treat; characteristically increased MCHC.
ALL occurs in X-linked disorder in which red cells are Glucose-6-
children, CNS unusually susceptible to damage cause by phosphate
spread is oxidants. (Drugs that produce oxidants dehydrogenase
common and is include antimalarials, sulfonamides, deficiency
generally nitrofurantoin, phenacetin, and vitamin K (G6PD
responsive to derivatives); Heinz bodies (derived from deficiency)
chemotherapy hemoglobin) seen on PBS of (Intrinsic;
An AML type that usually presents with Acute splenectomized patients; Bite cells seen extravascular
DIC; associated with t(15;17) promyelocytic on PBS of patients with intact spleen and
translocations; highly responsive to all- leukemia function. intravascular)
trans retinoic acid. This results from substitution of valine Sickle cell
Disorder of defective hematopoietic Myelodysplastic for glutamic acid at the 6th position of the anemia
maturation that results in ineffective syndrome β-chain, producing HbS; bizarre, (Intrinsic;
hematopoiesis (cytopenias); more elongated, spindled or boat-shaped cells extravascular)
common in the elderly; clinically present on PBS (evident on low oxygen tension
as bone marrow failure; associated with states); clinically, prominent cheekbones
increased risk of transformation to AML; and changes in skull resembling a "crew-
poor prognosis. cut" skull x-ray, due to bone marrow
Chronic myeloproliferative disorders expansion; also with painful vaso-
Common features of chronic Increased occlusive crisis; spleen may be enlarged,
myeloproliferative disorders. proliferative but in chronic cases may be small and
drive in BM, fibrosed (autosplenectomy), rendering
extramedullary them susceptible to infections with
hematopoiesis, encapsulated bacteria; treatment is
Spent phase, Hydroxyurea that increases HbF.
and variable Microcytic, hypochromic anemias Thalassemia
transformation characterized by deficiency of either α- or (Intrinsic;
to AML β-globin chains (memory device: the extravascular)
Chronic leukemia associated with BCR- Chronic name is what it lacks).
ABL fusion gene (Philadelphia myelogenous Condition caused by deletion of 1 α- Silent carrier
chromosome, t(9;22)); clinically presents leukemia (CML) globin gene.
with nonspecific symptoms and Condition caused by deletion of 2 α- α-Thalassemia
splenomegaly; BMA shows hypercellular globin genes; symptomatic at birth
marrow packed with less than 10% because of deficiency in HbF.
myeloblasts with more mature forms; Disease caused by deletion of 3 alpha α- Hemoglobin H
CBC shows leukocytosis >100,000/mm3 genes. disease
with low leukocyte alkaline phosphatase; Condition caused by deletion of all four α- Hydrops fetalis
and scattered macrophages with globin genes.
abundant, wrinkled, green-blue In β-Thalassemia, an individual who β-Thalassemia
cytoplasm (sea-blue histiocytes); can inherits one abnormal allele (out of 2) minor
proceed to a blast crisis if neglected. has this asymptomatic to mildly
Increase in all cell lines, but erythroid Polycythemia symptomatic condition; Red cells with a
lines are more increased; associated with vera (PV) central, dark-red puddle due to collection
JAK2 mutations in most cases; 2% chance of hemoglobin (target cells) on PBS.
of transformation to AML. Individuals with β-Thalassemia who β-Thalassemia
Increase in megakaryotic lines; Essential inherit two abnormal alleles, with severe major (Cooley
associated with JAK2 mutations in 50% of thrombocytosis anemia requiring regular blood anemia)
case; transformation to AML is transfusions; usually symptomatic at
uncommon. around six months of age because HbF
Extensive deposition of collagen in Primary synthesis ceases at this time; marked
marrow by non-neoplastic fibroblasts; myelofibrosis hypochromia, poikilocytosis (variation in
associated with JAK2 mutations in 50- cell size) and anisocytosis (variation in
60% of cases; 5-20% chance of cell shape) with normoblasts (signifying
TOPNOTCH MEDICAL BOARD PREP PATHO SUPERTABLE Page 20 of 47
For inquiries visit www.topnotchboardprep.com.ph or https://www.facebook.com/topnotchmedicalboardprep/
TOPNOTCH MEDICAL BOARD PREP PATHO SUPERTABLE by KEVIN ELOMINA, MD
For inquiries visit www.topnotchboardprep.com.ph or
https://www.facebook.com/topnotchmedicalboardprep/
erythropoietic drive). thymoma.
A mutation in the gene pigA causes this Paroxysmal Anemia caused by replacement of the Myelophthisic
disease; there is deficiency of PIG nocturnal bone marrow by in infiltrative processes anemia
(phosphatidylinositol glycan) that hemoglobinuria such as metastatic carcinoma and
anchors CD55 and CD59 to the red cell (Intrinsic; granulomatous disease; leads to the
surface; without such, red cells become intravascular) appearance of early erythroid and
susceptible to complement-mediated granulocytic precursors
lysis. (leukoerythroblastosis) and teardrop-
Anemia caused by low-affinity IgM which Cold antibody shaped red cells (dacrocytes) in the
bind to red cell membranes only at temp immune- peripheral blood.
<30°C, commonly experienced by distal hemolytic Denotes an increase in red cells per unit Polycythemia
parts of the body; associated with anemia volume of peripheral blood, usually in
Mycoplasma pneumoniae, and EBV (Extrinsic; association with an increase in
infections. intravascular) hemoglobin concentration; maybe
Hemolysis caused by IgG or IgA Warm antibody relative (hemoconcentration) or
antibodies that are active at 37°C, which immune- absolute; absolute may be primary
results in opsonization of red cells by the hemolytic (polycythemia vera) or secondary
autoantibodies. anemia (increased erythropoietin, lung disease,
(Extrinsic; high altitude, endurance athletes or
intravascular) erythropoietin-secreting tumors).
Anemia observed in a variety of Microangiopathi
pathologic states, in which small vessels c hemolytic
become particularly obstructed (e.g. DIC, anemia
malignant HTN, SLE, etc.); schistocytes, (Extrinsic;
Burr cells, helmet cells, triangle cells are intravascular) BLEEDING DISORDERS
seen in this condition. Caused by a systemic activation of Disseminated
Anemias of diminished erythropoiesis coagulation pathways, leading to intravascular
Most common cause of anemia in both Iron deficiency formation of thrombi throughout the coagulation
developing and developed countries; anemia microcirculation, and subsequent (DIC)
microcytic, hypochromic with low bleeding due to consumption of
ferritin, low serum iron levels low coagulation factors; laboratory findings
transferrin saturation, and increased include: thrombocytopenia, increased
TIBC. bleeding time and prolonged PT and PTT.
Most common cause of anemia in Anemia of Disorders of platelet number
hospitalized patients; anemia chronic disease Caused by antiplatelet antibodies Immune
characterized by high levels of hepcidin, directed against glycoproteins IIb-IIIa, Ib- thrombocytope
responsible for blocking the transfer of IX (usually IgG); laboratory findings nia/Immune
iron to erythroid precursors, hence include: decreased platelet count, thrombocytope
causing anemia; normocytic, prolonged bleeding time, and normal PT nic purpura
normochromic but can be microcytic, and PTT. (ITP)
hypochromic; increased ferritin, low Form of ITP usually affecting adults and Chronic ITP
serum iron levels, low transferrin females more than males; associated with
saturation, and decreased TIBC. SLE, HIV, B-cell neoplasms (CLL);
Anemia characterized by hypercellular Megaloblastic glucocorticoids are indicated; other
marrow as a result of increased number anemia tretament modalities include,
of megaloblasts, which are large cells that splenectomy, rituximab (anti CD20) in B-
have delicate, finely reticulated nuclear cell neoplasms, and TPO-mimetic
chromatin and abundant basophilic (Romiplostim).
cytoplasm; can be caused by Folate Form of ITP usually affecting children; Acute ITP
(without neurologic symptoms) or common antecedent viral illness;
Cobalamin (with neurologic symptoms) glucocorticoids are indicated only when
deficiency; PBS findings include: severe; usually self-limited.
hypersegmented neutrophils, large, egg- Thrombocytopenia with thrombosis, in a Heparin-
shaped macro-ovalocytes. patient taking anticogulants raises induced
Bone marrow is markedly hypocellular, Aplastic anemia suspicion of the this disorder. thrombocytope
with >90% of the intertrabecular space nia (HIT)
being occupied by fat; normocytic, Most common hematologic manifestation Thrombocytope
normochromic; most commonly caused of HIV infection. nia
by chemicals (Benzene) and drugs Associated with pentad of fever, Thrombotic
(Chloramphenicol). thrombocytopenia, microangiopathic thrombocytope
Selective suppression of marrow Pure red cell hemolytic anemia, transient neurologic nic purpura
erythroid precursors; can be autoimmune aplasia (PRCA) deficits and renal failure; primary defect (TTP)
or infectious (Parvovirus) in etiology; is deficiency of ADAMTS13, a vWF
presents with normocytic, normochromic metalloproteinase; laboratory findings
anemia with virtuallyt absent include: decreased platelet count,
eryhtroblasts in bone marrow, and prolonged bleeding time, and normal PT
normal granulo- and thrombopoietic and PTT.
elements; paraneoplastic syndrome of Associated with childhood onset Hemolytic
TOPNOTCH MEDICAL BOARD PREP PATHO SUPERTABLE Page 21 of 47
For inquiries visit www.topnotchboardprep.com.ph or https://www.facebook.com/topnotchmedicalboardprep/
TOPNOTCH MEDICAL BOARD PREP PATHO SUPERTABLE by KEVIN ELOMINA, MD
For inquiries visit www.topnotchboardprep.com.ph or
https://www.facebook.com/topnotchmedicalboardprep/
microangiopathic hemolytic anemia, Uremic DISEASES OF THE LUNG
thrombocytopenia, and acute renal Syndrome (HUS) Loss of lung volume cause by Atelectasis
failure; no neurologic symptoms; often inadequate expansion of airspaces,
with a history of bloody diarrhea (E. coli resulting in shunting of inadequately
O157:H7 infection); with normal oxygenated blood from pulmonary
ADAMTS13 activity; laboratory findings arteries into veins; can be resorption
include: decreased platelet count, (obstruction that prevents air
prolonged bleeding time, and normal PT reaching distal air spaces),
and PTT. compression (pleural fluid or mass
Disorders of platelet function compressing the lung), and
Decreased gp Ib leads to defective Bernard-Soulier contraction (pleural fibrosis impeding
platelet adhesion; laboratory findings syndrome expansion of lungs).
include: decreased platelet count, Abrupt onset of significant hypoxemia Acute lung injury
prolonged bleeding time, and normal PT and bilateral infiltrates WITHOUT (ALI)
and PTT. heart failure.
Decreased gp IIb-IIIa leads to defective Glanzmann Severe manifestation of ALI; lungs are Acute respiratory
platelet aggregation, laboratory findings thrombasthenia dark red, firm, airless and heavy; distress syndrome
include: normal platelet count, prolonged there is capillary congestion, necrosis (ARDS)
bleeding time, and normal PT and PTT. of alveolar epithelial cells, interstitial
Coagulation disorders and intra-alveolar edema and
Most common inherited bleeding Von Willebrand hemorrhage and neutrophils in
disorder; most common presenting disease (vWD) capillaries; hyaline membrane is also
symptoms of this condition are characteristic, lining the alveolar
spontaneous bleeding from mucous ducts.
membranes, excessive bleeding from Most common causes of ARDS. Sepsis, diffuse
wounds, or menorrhagia; bleeding pulmonary
tendency often goes unnoticed until some infections, Gastric
hemostatic stress, such as surgery, aspiration
reveals its presence; laboratory findings Class A recommendation for Low Tidal Volume
include normal platelet count, prolonged management of ARDS. ventilation: 6cc/Kg
bleeding time, and normal PT and PREDICTED Body
prolonged PTT. weight
Early
neuromuscular
Patient with this disease has a tendency Hemophilia blockade:
toward easy bruising and massive (Hemophilia A – Cisatracurium
hemorrhage after trauma or operative Factor VIII besylate for 48
procedures, and recurrent bleeding into deficiency; hours
the joints; laboratory findings include Hemophilia B
normal platelet count, bleeding time, PT (Christmas
and prolonged PTT. disease) –
Factor IX
deficiency
Vitamin K dependent coagulation and Factors II, VII,
fibrinolytic factors, and laboratory IX, and X; Obstructive and Restrictive lung diseases
picture of Vitamin K deficiency. Protein C and S; Diffuse pulmonary disease Obstructive lung
normal platelet characterized by limitation of airflow, diseases
count and usually resulting from an increase in
bleeding time; resistance caused by partial or
prolonged PT complete obstruction at any level;
and PTT spirometry findings: markedly
Transfusion reactions decreased FEV1, decreased FVC,
Transfusion reaction to watch out for in Allergic decreased FEV1/FVC; examples:
patients with IgA deficiency. reactions Bronchial asthma, Emphysema,
IgM-mediated destruction of donor red Acute hemolytic Chronic bronchitis and Bronchiectasis
cells. transfusion Characterized by abnormal Emphysema (Pink
reaction (AHTR) permanent enlargement of the puffers)
IgG-mediated destruction of donor red Delayed airspaces distal to the terminal
cells in a previously-sensitized individual. hemolytic bronchioles accompanied by
transfusion destruction of their walls without
reaction (DHTR) obvious fibrosis; there is thinning of
ARDS-like complication in patients who Transfusion the alveolar walls and loss of elastic
underwent transfusion; due to activation related acute tissue; can be centriacinar
of PMNs in lung vasculature. lung injury (respiratory bronchioles only;
(TRALI) associated with smoking), panacinar
(associated with α-1-antitrypsin
CHAPTER 14: THE LUNG AND PLEURA deficiency, distal acinar (most
QUESTION ANSWER commonly seen in adults with
TOPNOTCH MEDICAL BOARD PREP PATHO SUPERTABLE Page 22 of 47
For inquiries visit www.topnotchboardprep.com.ph or https://www.facebook.com/topnotchmedicalboardprep/
TOPNOTCH MEDICAL BOARD PREP PATHO SUPERTABLE by KEVIN ELOMINA, MD
For inquiries visit www.topnotchboardprep.com.ph or
https://www.facebook.com/topnotchmedicalboardprep/
spontaneous pneumothorax), and and honeycomb lung; examples: Usual
irregular (most common). interstitial pneumonia (Idiopathic
Defined as persistent productive Chronic bronchitis pulmonary fibrosis), Pneumoconiosis,
cough for at least 3 consecutive (Blue bloaters) and Sarcoidosis.
months in at least 2 consecutive
years; morphologically, mucosal Pulmonary infections
lining of the larger airways is usually Pneumonia with a patchy distribution Bronchopneumonia
hyperemic and swollen with edema of inflammation involving more than
fluid, often covered by a layer of one lobe.
mucinous or mucopurulent Pattern of pneumonia observed in Lobar pneumonia
secretions; trachea and bronchi have pneumococcal pneumonia; has four
enlarged mucus-secreting glands. phases of evolution, from congestion,
Triad of: 1. Intermittent and Bronchial asthma red hepatization, gray hepatization
reversible airway obstruction; 2. obstruction and resolution.
inflammation
Chronic bronchial inflammation with hypertrophy
Inflammatory reaction is largely Atypical
eosinophils; and 3. Bronchial smooth confined within the walls of the pneumonia
muscle cell hypertrophy and alveoli; septa widened and inflammatory action on walls
widened septa
hyperreactivity; morphologically edematous, with mononuclear mononuclear infiltrates
histiocytes and plasma cells
characterized by thick, tenacious infiltrates of lymphocytes, histiocytes free of cellular infiltrates

mucus plugs with Curschmann spirals and plasma cells; alveolar spaces are
(whorls of shed epithelium found in free of cellular exudate. This
mucus plugs) and Charcot-Leyden morphology points more to:
crystals (collections of crystalloids A localized area of suppurative Lung abscess
made up of eosinophil protein). necrosis within the pulmonary
Criteria for reversibility of airway FEV1 increase of parenchyma, resulting in the
obstruction diagnostic of bronchial >12% or 200 mL formation of one or more large
asthma. post cavities; most commonly occurs in the
bronchodilator right side, particularly in the posterior
segment of the upper lobe and apical
Permanent dilation of bronchi and Bronchiectasis segments of the lower lobe; most
bronchioles caused by destruction of common isolates are anaerobic
the muscle and elastic supporting bronchial dilation
necrotizing infection (exudate, mixed flora) bacteria.
tissue, resulting from chronic fibrosis
Tuberculosis
necrotizing infections; Form of tuberculosis that develops in Primary
morphologically, there is intense a previously unexposed, unsensitized tuberculosis
acute and chronic inflammatory person.
exudate within the walls, with mixed Implantation of the inhaled M. Ghon focus
flora often cultured; there is also tuberculosis bacilli in the distal
peribronchiolar fibrosis in chronic airspaces of the lungs, causes the
cases. formation of this 1-1.5 cm area of
Thickening of airway wall; sub- Airway remodeling gray-white inflammatory
basement membrane fibrosis; consolidation.
increased vascularity in submucosa; Ghon focus with involvement of the Ghon complex
increase in size of the submucosal hilar lymph nodes is called:
glands and goblet Radiographically detectable calcified Ranke complex
cell metaplasia of the airway Ghon complex is called:
epithelium; hypertrophy and/or
Pattern of disease that arises in Secondary or
hyperplasia of the bronchial muscle;
previously sensitized host to M. reactivation TB
are collectively called:
tuberculosis.

Occurs when TB bacilli drain through Miliary

the lymphatics into lymphatic ducts, tuberculosis

which eventually empty into the right

side of the heart and into pulmonary

circulation; individual lesions are
Diffuse pulmonary disease Restrictive lung
small, visible foci of yellow-white
characterized by reduced expansion diseases
consolidation scattered through the
of lung parenchyma accompanied by reduced lung parenchyma, dec FEV1, dec in FVC
-> normal to increased. FEV1/VVC parenchyma.
decreased total lung capacity;
Type of necrosis seen in tuberculosis. Caseous
spirometry findings: decreased FEV1,
Type of inflammation seen in Chronic
markedly decreased FVC, normal or
tuberculosis. granulomatous
increased FEV1/FVC; examples:
Type of hypersensitivity reaction in Type IV (Cell-
Chronic interstitial lung diseases
tuberculosis (including TST). mediated/Delayed)
Heterogeneous group of disorders Chronic interstitial
Cytokine responsible for formation of IFN-γ
characterized predominantly by lung diseases
granulomas in tuberculosis.
inflammation and fibrosis of the

pulmonary interstitium; follows
restrictive lung disease pattern; Histologic appearance of tuberculosis Macrophages filled
complications include cor pulmonale infection in the absence of an intact with bacilli
cell-mediated immunity.
TOPNOTCH MEDICAL BOARD PREP PATHO SUPERTABLE Page 23 of 47
For inquiries visit www.topnotchboardprep.com.ph or https://www.facebook.com/topnotchmedicalboardprep/
TOPNOTCH MEDICAL BOARD PREP PATHO SUPERTABLE by KEVIN ELOMINA, MD
For inquiries visit www.topnotchboardprep.com.ph or
https://www.facebook.com/topnotchmedicalboardprep/
Most common form of Tuberculous Main consequences of gingivitis. Plaque and calculus
extrapulmonary TB. lymphadenitis Fibrous proliferative lesion usually Pyogenic
(scrofula) occurring in children, adults and granuloma
Most common affected segment in Ileum pregnant women (pregnancy tumor);
gastrointestinal TB (Note: GITB is histologically, characterized by
rare, due to decrease in consumption organizing granulation tissue (highly
of infected milk). vascular).
Tumors Most common oral fungal infection. Candidiasis
One of the two most common Squamous cell Characterized by vesicle formation; Herpetic
histologic types of lung cancer carcinoma histologically characterized by gingivostomatitis
associated with smoking; centrally ballooned cells with large eosinophilic
located in major bronchi; may range intranuclear inclusions (Cowdry Type
from poorly differentiated to well- A inclusion bodies).
differentiated with keratin pearl A white/red plaque that cannot be Leukoplakia/
formation; associated in general with scraped off; considered precancerous Erythroplakia
hypercalcemia, due to production of unless proven otherwise; the red
PTH related peptide (PTHrp). plaque is more associated with
Immunostains: p63, p40. malignancy than the white plaque;
The most common histologic type of Adenocarcinoma most common risk factor is cigarette
lung cancer; most commonly seen in smoking.
nonsmokers; peripherally located; Neoplasms
histologically, may show acinar Most common histologic type of oral Squamous cell
(glandular), papillary, mucinous, and cavity cancer. carcinoma
solid types. Immunostains: TTF-1 and Most common benign tumor of the Pleomorphic
napsin A. salivary gland; histologically adenoma
Most common histologic type of lung Small cell (Oat cell) characterized by epithelial elements
cancer associated with smoking; carcinoma arranged in ducts, acini, irregular
centrally located; with early nodal tubules, strands, or even sheets, in a
involvement; histologically, small, mesenchyme-like background of loose
round to fusiform cells with scant myxoid tissue containing islands of
cytoplasm and finely granular chondroid and rarely, foci of bone;
chromatin with numerous mitotic locally invasive.
figures are present; most associated The most common primary malignant Mucoepidermoid
with paraneoplastic syndromes, such tumor of salivary glands composed of carcinoma
as Cushing syndrome (ectopic ACTH), mucous and squamous cells.
Lambert-Eaton myasthenic syndrome, Most common epithelial odontogenic Ameloblastoma
and SIADH, among others; only type tumor that arises from odontogenic
not amenable to surgery; epithelium that does not demonstrate
Immunostains: chromogranin, extomesenchymal differentiation;
synaptophysin, CD57 and BCL2. locally invasive.
Lung carcinoma that typically has Large cell DISEASES OF THE UPPER AIRWAYS
large, prominent nucleoli, and a carcinoma Rare neoplasm with strong links to Nasopharyngeal
moderate amount of cytoplasm, with EBV and high frequency among the carcinoma
minimal glandular or squamous Chinese population. Characterized by
differentiation. Diagnosed when large epithelial cells having indistinct (Note: some types
immunostains for known histologic borders (syncytial growth) and of HL, and Burkitt
types of lung cancer are negative. prominent eosinophilic nucleoli. are also associated
DISEASES OF THE PLEURA with EBV)
A rare cancer seen in patients who Malignant A benign neoplasm usually on the true Laryngeal
work in shipyards, miners and mesothelioma vocal cords that forms a soft, papilloma
insulators; presents as lung raspberry-like excresence rarely more
ensheathed by a yellow-white, firm or (Note: Malignant then 1 cm in diameter; consists of
gelatinous layer of tumor that mesothelioma is multiple, slender, finger-like
obliterates the pleural space; lesion is commonly projections.
preceeded by extensive pleural associated with Most common symptom of laryngeal Persistent
fibrosis and plaque formation; may be asbestos, but the carcinoma. hoarseness
epithelial, sarcomatoid or biphasic in most common Common etiologic agents implicated Smoking, alcohol,
morphology. cancer asbestos in laryngeal carcinoma asbestos, radiation,
Immunostains: Epithelioid: Keratin, causes is still lung HPV
calretinin, WT-1, CK5,6, D2-40; cancer) Most common histologic type of Squamous cell
Sarcomatoid: usually Keratin(+) only. laryngeal carcinoma carcinoma (95%)
Most common location of laryngeal Glottic (60-75%)
CHAPTER 15: HEAD AND NECK carcinoma
QUESTION ANSWER EARS
ORAL CAVITY Most common cause of otitis media. Viral
Non-neoplastic conditions Most common primary bacterial S. pneumoniae, non-
Main cause of tooth loss before age Dental caries causes of acute otitis media. typeable H.
35. influenzae, M.
TOPNOTCH MEDICAL BOARD PREP PATHO SUPERTABLE Page 24 of 47
For inquiries visit www.topnotchboardprep.com.ph or https://www.facebook.com/topnotchmedicalboardprep/
TOPNOTCH MEDICAL BOARD PREP PATHO SUPERTABLE by KEVIN ELOMINA, MD
For inquiries visit www.topnotchboardprep.com.ph or
https://www.facebook.com/topnotchmedicalboardprep/
catarrhalis metaplasia within the esophageal
Most common causes of chronic otitis P. aeruginosa, S. squamous mucosa; increased risk
media. aureus, fungus, of esophageal adenocarcinoma.
mixed Neoplasms
Complications of otitis media. Brain abscess and Esophageal carcinoma associated Adenocarcinoma
cholesteatoma with chronic GERD; usually occurs
NECK in the distal third of the esophagus;
True cyst arising from remnant of Branchial cleft cyst histologically characterized by
second branchial arch (most gland formation and mucin
common); clinically presents as upper production; premalignant lesion is
lateral neck mass along SCM. Barrett esophagus.
More common of the Squamous cell
histologicpatterns of esophageal carcinoma
cancer; associated with smoking,
alcohol, caustic injury, and frequent
consumption of very hot beverages;
Most common congenital anomaly of Thyroglossal duct usually occurs in the middle third
thyroid gland; arising from remnants cyst of the esophagus; premalignant
of thyroid gland descent; clinically lesion is squamous dysplasia.
present as a midline mass along the
track of descent of the thyroid gland;
treatment of choice is Sistrunk
procedure.
Neuroendocrine neoplasm associated Paranganglioma
with ANS; basically, an extra-adrenal
pheochromocytoma (histologically);
can be located paravertebral STOMACH
(sympathetic), or in aorticopulmonary Gastritides
chain (parasympathetic). Most common form of chronic H. pylori gastritis
SALIVARY GLANDS gastritis; most common cause of
Most common type of inflammatory Mucocele multifocal atrophic gastritis;
salivary gland lesion and the most characterized by the following:
common salivary gland lesion. predominantly antral, neutrophilic
Most common form of viral Mumps infiltrate with subepithelial plasma
sialadenitis; most common gland cells, serum gastrin normal to
involved: parotid. decreased; associated with
Most common gland involved in Submandibular hyperplastic/inflammatory polyps;
sialolithiasis. (+) for H. pylori antibodies;
Most common site of tumors Parotid gland sequelae includes gastric
regardless of behavior. lymphoma.
Most common tumor of the salivary Pleomorphic Most common form of gastritis in Autoimmune gastritis
gland; also the most common benign adenoma patients without H. pylori infection;
tumor. most common cause of diffuse
Most common primary malignant Mucoepidermoid atrophic gastritis; characterized by
tumor of the salivary gland; and the carcinoma the following: predominantly at the
second most common tumor. body, lymphocytic infiltrate with
Second most common benign tumor Warthin tumor macrophages, serum gastrin
of the salivary gland, with high (Papillary increased; associated with
association with smoking. cystadenoma neuroendocrine hyperplasia; (+)
lymphomatosum) for antibodies against parietal cells
Highly aggressive malignancy of the Adenoid cystic (proton pump and intrinsic factor);
salivary gland, with high propensity carcinoma clinically, patients present with
for perinueral invasion, and high achlorhydia (due to destruction of
occurrence of distant metastasis. parietal cells), and megaloblastic
anemia (due to Vitamin B12

deficiency).
CHAPTER 16: THE GASTROINTESTINAL TRACT
Peptic ulcer disease
ESOPHAGUS
Most common form of PUD. Antral or duodenal (H.
Non-neoplastic conditions
pylori)
Typical findings in significant Eosinophilic infiltrate
Histologic findings in PUD. NIGS: N: necrosis, I:
GERD. in mucosa with
acute inflammation,
neutrophils (if severe)
G: granulation tissue,
and Basal zone
S: fibrous scar (from
hyperplasia
superficial to deep)
Portocaval anastomoses involved Esophageal branch of
Neoplasms
in esophageal varices. left gastric vein and
azygos vein Most common gastric polyp. Inflammatory/
Hyperplastic (75%)
Complication of long-standing Barrett esophagus
GERD; characterized by intestinal The most common malignancy of Gastric
TOPNOTCH MEDICAL BOARD PREP PATHO SUPERTABLE Page 25 of 47
For inquiries visit www.topnotchboardprep.com.ph or https://www.facebook.com/topnotchmedicalboardprep/
TOPNOTCH MEDICAL BOARD PREP PATHO SUPERTABLE by KEVIN ELOMINA, MD
For inquiries visit www.topnotchboardprep.com.ph or
https://www.facebook.com/topnotchmedicalboardprep/
the stomach (90%); has two types: adenocarcinoma disease. penetrated by blood
intestinal (bulky tumors with vessels)
gland-like structures Type of diverticulum in sigmoid False diverticulum
histologically); and diffuse (intense .diverticulosis.
desmoplasia leading to leather Neoplasms
bottle stomach (linitis plastic) with Polyps common in the left colon; Hyperplastic polyp
signet ring cells containing mucin characterized by serrated
histologically. (Lauren architecture limited to the upper
classification of intestinal and third of the crypt; no cytologic
diffuse is used); most common site: atypia and lateral growth; do not
Antrum, on the lesser curvature. have malignant potential.
Malignancy associated with H. Lymphoma/MALToma Polyps common in the right colon; Sessile serrated
pylori infection; most common site characterized by serrated adenoma
of occurrence is the stomach; architecture throughout the whole
eradication of H. pylori leads to length of the gland; no cytologic
tumor regression, except when it atypia but exhibits lateral growth
transforms to DLBCL. (elephant-feet glands); has
Tumors of neuroendocrine origin; Carcinoid malignant potential like
common site is small intestine; may conventional adenomas.
elaborate hormones that can lead Autosomal dominant syndrome of Peutz-Jeghers
to ZES (gastrin) or carcinoid hamartomatous polyps and syndrome
syndrome (serotonin); most mucocutaneous
important prognostic factor is hyperpigmentation, associated
location i.e. midgut tumors are with increased risk of visceral
often multiple and aggressive; malignancies; histologically
immunostains: Synaptophysin and characterized by arborizing
Chromogranin A. networks composed of smooth
Most common mesenchymal tumor Gastrointestinal muscle intermixed with lamina
of the abdomen; more than half of stromal tumor (GIST) propria.
cases occur in the stomach; Morphologic variants of colonic Tubular,
cytogenetic origin is interstitial adenomas. Tubulovillous, Villous
cells of Cajal (ICC); histologically The single most important Size (>4cm) (others:
composed of elongated spindle characteristic of adenomas that architecture (villous),
cells or plump epithelioid cells; correlates with malignancy. and severity of
Immunostains: KIT. dysplasia
Familial syndrome characterized Familial adenomatous
SMALL AND LARGE INTESTINES by early onset development of polyposis (FAP)
Intestinal obstruction numerous colonic polyps; main
Most common cause of intestinal Hernias pathology is mutations on the APC
obstruction. gene (Ch5); criterion is the
Most common cause of intestinal Intussusception presence of at least 100 polyps.
obstruction in children <2 years
old.
Ischemic bowel disease
Most common regions susceptible Splenic flexure and
to ischemia. rectosigmoid
Inflammatory bowel disease Familial syndrome characterized Hereditary
Inflammatory bowel disease Crohn disease by early development of colon nonpolyposis colon
characterized by the following: skip cancer and other visceral cancer
lesions, transmural bowel malignancies; main pathology is (HNPCC)/Lynch
involvement, and rectal sparing; mutations in genes of DNA repair syndrome
associated with development of (MSH2 or MLH1) that leads to
colonic adenocarcinoma if with microsatellite instability and
colonic involvement. subsequent development of colonic
Inflammatory bowel disease Ulcerative colitis adenocarcinoma.
characterized by the following: Most common malignancy of the Colonic
continuous lesions usually limited gastrointestinal tract; two adenocarcinoma
up to submucosa, with rectal pathways are recognized; the
involvement but disease is limited adenoma carcinoma sequence
to the colon only; associated with involving APC among others; and
toxic megacolon and development the microsatellite instability
of colonic adenocarcinoma. pathway (sessile serrated
Note: Make sure to study Table 14-5 in Robbins Basic adenoma-carcinoma sequence)
Pathology, 9th ed. p. 587 or Table 17-8 in Robbins and Cotran involving MSH2 or MLH1; clinically
Pathologic Basis of Disease 8th ed. p. 808 or Table 17-9 in characterized by different
Robbins and Cotran Pathologic Basis of Disease 9th ed. p. 797. symptomatologies depending on
Diverticular disease location: (Right colon: usually
Most common site of diverticular Sigmoid (in areas anemia from bleeding from the
TOPNOTCH MEDICAL BOARD PREP PATHO SUPERTABLE Page 26 of 47
For inquiries visit www.topnotchboardprep.com.ph or https://www.facebook.com/topnotchmedicalboardprep/
TOPNOTCH MEDICAL BOARD PREP PATHO SUPERTABLE by KEVIN ELOMINA, MD
For inquiries visit www.topnotchboardprep.com.ph or
https://www.facebook.com/topnotchmedicalboardprep/
bulky nonobstructive masses; Left liver disease and portal hypertension hypertension
colon: usually presents with change due to excessive pulmonary vascular
in bowel habits due to napkin-ring remodeling; Clinically presents with
configuration of the lesion). exertional dyspnea and clubbing of
Two important prognostic factors Invasion and lymph the fingers.
in colonic adenocarcinoma. node status Clinical entity that occurs when Jaundice
Most common site of metastases of Liver retention of bilirubin leads to serum
colonic adenocarcinoma. levels of at least 2.0-2.5 mg/dL.
APPENDIX Most common cause of conjugated Hepatitis and intra-
The most common cause of acute Lymphoid hyperplasia hyperbilirubinemia. and extrahepatic
appendicitis in children and adults, and Fecalith, obstruction of bile
respectively. respectively flow
The most common tumor of the Carcinoid Most common cause of unconjugated Hemolysis
appendix. hyperbilirubinemia.
Hepatitides
CHAPTER 17: LIVER AND GALLBLADDER Note: Please study Table 15-5 of Robbins Basic Pathology, 9th
QUESTION ANSWER ed p. 613, or Table 18-3 of Robbins and Cotran Pathologic
LIVER Basis of Disease, 9th ed p. 836, for the detailed morphologies of
Clinical syndromes acute and chronic hepatitides.
Condition wherein there is 80-90% Acute liver failure The main differences in morphology Acute hepatitides
liver parenchymal loss in patient of acute and chronic hepatitides. have less
without pre-existing liver disease; inflammation and
clinically presents with more hepatocyte
encephalopathy and coagulopathy; death than chronic
occurs <26 weeks post initial insult. hepatitis
A clinical entity caused by severe loss Hepatic Which virus between HBV and HCV is HCV (most cases of
of hepatoceullular function and encephalopathy more associated with hepatocellular acute HBV infection
shunting of blood from portal to carcinoma? present with
systemic circulation, causing buildup subclinical disease
of toxins that cause neuronal and all of them
dysfunction (in acute cases); recover)
manifestations may range from Morphologic appearance of chronic Ground glass
subtle behavioral changes to deep Hepatitis B infection. hepatocytes
coma and death. Morphologic appearance of chronic Lymphoid follicles,
Syndrome heralded by a drop in Hepatorenal Hepatitis C infection. bile duct injury and
urine output and rising BUN and syndrome steatosis
creatinine; appears in severe liver Hepatitis characterized by early Autoimmune
failure; characterized by renal failure development of scarring after the hepatitis
in the setting of liver disease without initial wave of hepatocyte injury
primary renal abnormalities (versus viral hepatitides where
Common causes of chronic liver Chronic HBV and scarring usually happens years after
failure. HCV, Alcoholic liver the initial insult); associated with
disease, and NAFLD confluent necrosis, marked
Diffuse transformation of the liver Cirrhosis inflammation, plasma cell infiltrate,
into regenerating parenchymal and burned-out cirrhosis; has two
nodules, surrounded by dense bands types: Type 1 (in adults; associated
of scar, with variable degrees of with ANA and anti-SMA); and Type 2
vascular shunting. (in children; associated ithe Anti-
Common causes of death in chronic Complications of LKM1).
liver failure. acute liver failure Most common drug causing ALF. Acetaminophen
and hepatocellular Most common substance causing CLF. Alcohol
carcinoma Alcoholic and Non-alcoholic fatty liver disease
Amount of alcohol associated with 80 g/d
development of ALD.
Liver enzyme pattern unique to ALD. AST > ALT
Three morphological forms of Hepatocellular
Increased resistance to portal blood Portal hypertension alcoholic liver disease. steatosis; Alcoholic
flow that may be caused by Steatohepatitis; and
prehepatic, intrahepatic and Steatofibrosis
posthepatic causes; can cause dilation Hepatocyte ballooning, eosinophilic Alcoholic
of the portocaval anastomoses and intarcytoplasmic inclusion bodies Steatohepatitis
produce varices. composed of intermediate filaments
Severe arterial hypoxemia with Hepatopulmonary (Mallory-Denk bodies) and
dyspnea and cyanosis in patients syndrome neutrophilic infiltration are
with liver disease due to abnormal characteristic of:
intrapulmonary vascular dilatation
and increased pulmonary blood flow. Most common metabolic liver NAFLD
Pulmonary hypertension arising in Portopulmonary disease; usually occurs in patients
TOPNOTCH MEDICAL BOARD PREP PATHO SUPERTABLE Page 27 of 47
For inquiries visit www.topnotchboardprep.com.ph or https://www.facebook.com/topnotchmedicalboardprep/
TOPNOTCH MEDICAL BOARD PREP PATHO SUPERTABLE by KEVIN ELOMINA, MD
For inquiries visit www.topnotchboardprep.com.ph or
https://www.facebook.com/topnotchmedicalboardprep/
consuming <20 g/week of ethanol; (histologically characterized by
associated with Type 2 DM, insulin multinucleated giant cells, and cannot
resistance, dyslipidemia, and be corrected, or even worsened with
hypertension. surgical intervention).
Most common outcome of NAFLD Isolated fatty liver
(>80%); most cases
do not progress to
NASH
Difference of non-alcoholic fatty liver Less prominent Complete or partial obstruction of the Extrahepatic biliary
disease from ALD. features of lumen of the extrahepatic biliary tree atresia
steatohepatitis within the first 3 months of life;
compared to ALD single most common cause of death
A rare disease characterized by Reye syndrome from liver disease in early childhood;
microvesicular fatty change in the most common is perinatal form;
liver and encephalopathy; treatment of choice is Kasai
microscopy of hepatocellular procedure.
mitochondria reveals pleomorphic Autoimmune cholangiopathies
enlargement and electron lucency of A chronic, progressive cholestatic Primary biliary
the matrices, with disruption of liver disease characterized by cirrhosis (PBC)
cristae and loss of dense bodies; nonsuppurative destruction of small
clinically suspected in a pediatric to medium-sized intrahepatic bile
patient who took Aspirin as ducts; associated with
antipyretic for a viral infection. antimitochondrial antibodies is 90%
Inherited metabolic diseases of cases; histologically, characterized
Triad of micronodular cirrhosis, Hereditary by destruction of interlobular bile
diabetes mellitus and abnormal skin Hemochromatosis ducts with lymphoplasmacytic
pigmentation; main pathology is infiltration with or without
extensive accumulation of body iron granulomas (florid duct lesions).
in the liver, pancreas and heart; most Chronic progressive fibrosis and Primary sclerosing
common form is autosomal recessive, destruction of extrahepatic and cholangitis (PSC)
involving mutations in HFE gene in intrahepatic bile ducts of all sizes;
Chromosome 6; associated with 200- patchy involvement gives rise to a
fold increased risk of HCC. beading appearance of ducts;
Extensive accumulation of toxic levels Wilson disease histologically characterized by onion-
of Copper in the liver, brain and eye; (Hepatolenticular skin lesion with an atrophic tubule on
(Kayser-Fleischer rings); autosomal degeneration) the center (small ducts) or acute on
recessive; main pathology is loss of chronic inflammation (large ducts);
function mutations in ATP7B gene in associated with an increased risk of
Chromosome 13 cholangiocarcinoma.
Autosomal recessive disorder α-1-antitrypsin Structural abnormalities of the biliary tree
characterized by panacinar deficiency Triad of choledochal cysts. Pain, jaundice,
emphysema and liver disease abdominal mass
secondary to accumulation of Choledochal cysts and fibropolycystic Cholangiocarcinoma
misfolded proteins; histologically diseases predisposes to this
marked by PAS-positive, diastase- particular kind of cancer.
resistant, cytoplasmic inclusions, Tumors
corresponding to mutant AAT; Most common benign tumor of the Cavernous
associated with increased risk of HCC. liver. hemangioma
Cholestatic liver diseases Most common liver tumor of early Hepatoblastoma
Most common cause of large bile duct Gallstones, biliary childhood.
obstruction in adults, and children, atresia A well-demarcated but poorly Focal nodular
respectively. encapsulated lesion, consisting of hyperplasia
Secondary bacterial infection of Ascending hyperplastic hepatocyte nodules with
biliary tree; clinically typified by cholangitis a central fibrous scar; carries no risk
Charcot triad: 1. fever, 2. jaundice, for malignancy.
and 3. RUQ pain. A benign tumor characterized by Hepatic adenoma
Intrehepatic gallstone formation Primary hepatocytes without portal tracts
(calcium bilirubinate) due to hepatolithiasis with prominent neovascularization;
repeated bouts of ascending harbors a risk for malignant
cholangitis and parenchymal transformation.
destruction; associated with Type of hepatocellular adenoma with β-catenin adenomas
increased risk of cholangiocarcinoma. the highest risk of malignant
transformation.
Prolonged conjugated Neonatal cholestasis Most common primary malignant Hepatocellular
hyperbilirubinemia in the neonate; tumor of the liver; commonly arises carcinoma
can either be caused by extrahepatic in the setting of chronic liver disease;
biliary atresia (can be corrected with may appear as unifocal, multifocal or
surgery), or neonatal hepatitis diffusely infiltrative; with strong
TOPNOTCH MEDICAL BOARD PREP PATHO SUPERTABLE Page 28 of 47
For inquiries visit www.topnotchboardprep.com.ph or https://www.facebook.com/topnotchmedicalboardprep/
TOPNOTCH MEDICAL BOARD PREP PATHO SUPERTABLE by KEVIN ELOMINA, MD
For inquiries visit www.topnotchboardprep.com.ph or
https://www.facebook.com/topnotchmedicalboardprep/
propensity for vascular invasion; morphological changes: 1.
histologically, well-differentiated microvascular leak and edema; 2.
lesions may elaborate bile appearing enzymatic fat necrosis; 3. acute
as globules; tumor marker is AFP. inflammation; 4. destruction of
Most common site of hematogenous Lung pancreatic parenchyma; and 5.
metastases of HCC. destruction of blood vessels and
Most common tumor involving the Metastases interstitial hemorrhage.
liver. Most common causes of acute Alcoholism and
GALLBLADDER pancreatitis. biliary tract disease
Gallstones (80%)
Most common type of gallstones; pale Cholesterol stones Prolonged inflammation of the Chronic
yellow in color; radiolucent. (80%) pancreas associated with irreversible pancreatitis
Pigment stones derived from sterile Black pigment destruction of exocrine parenchyma,
gallbladder bile; 50-70% are stones fibrosis, and, in the late stages, the
radiopaque. destruction of endocrine parenchyma.
Pigment stones derived from infected Brown pigment Most common cause of chronic Long-term alcohol
gallbladder bile; radiolucent. stones pancreatitis. abuse
More common form of cystic disease Pseudocysts
Complication of cholelithiasis, Gallstone ileus in the pancreas.
wherein a large stone erodes directly (Bouveret Neoplasms
into adjacent small bowel. syndrome) Most common site of pancreatic Head (presents as
Most common form of acute Acute calculous adenocarcinoma. obstructive
cholecystitis. cholecystitis (90%) jaundice)
The gallbladder may be contracted, of Chronic acalculous Most common mutated oncogene in KRAS
normal size, or enlarged; presence of cholecystitis pancreatic adenocarcinoma.
stones in the absence of inflammation Most common mutated tumor p16
is diagnostic. suppressor gene in pancreatic
Most common form of chronic Chronic calculous adenocarcinoma .
cholecystitis; characterized by cholecystitis Strongest environmental risk factor Smoking
mononuclears in GB wall, with (>90%) for developing pancreatic
Rochitansky-Aschoff sinus. adenocarcinoma.
Tumors Two distinct features of pancreatic Highly invasive and
Most common malignant tumor of the Gallbladder ductal adenocarcinoma. intense
biliary tract; maybe exophytic (with carcinoma desmoplasia
mass) or infiltrating (with thickening Tumor marker for pancreatic CA19-9
of the wall; more common); most adenocarcinoma.
frequent histology is The usual sites of metastases of Liver and lungs
adenocarcinoma. pancreatic adenocarcinoma.
Most important risk factor for the Gallstones Paraneoplastic syndrome associated Trosseau syndrome
development of gallbladder with pancreatic adenocarcinoma,
carcinoma. characterized by migratory superficial
Second most common primary Cholangiocarcinoma thrombophlebitis, secondary to
malignant tumor of the liver; expression of pro-coagulant factors
malignant tumor of cholangiocytes; from the tumor or its necrotic
more common in extrahepatic bile products.
ducts; may develop in the hilum
(Klatskin tumor); risk factors include: CHAPTER 19: THE KIDNEY
NAFLD, PSC, fibrocystic diseases of QUESTION ANSWER
the biliary tree (choledocal cysts), Glomerular diseases
and infestation of Clonorchis sinensis Nephritic syndrome: 1. Hematuria (with dysmorphic RBCs
and Opistorchis viverrini; most and red cell casts indicating glomerular pathology); 2.
frequent histology is Oliguria and azotemia; and 3. Hypertension
adenocarcinoma. Most common cause of nephritic Acute
syndrome in children Postinfectious
CHAPTER 18: THE PANCREAS LM: diffuse hypercellularity (almost, if (Poststreptococcal)
QUESTION ANSWER not all glomeruli) Glomerulonephritis
Congenital anomalies EM: subepithelial humps on GBM (PSAGN)
What is the most common clinically Pancreatic divisum IF: granular deposits of IgG and
significant congenital anomaly of the complement within the capillary walls
pancreas? mesangium
Pancreatitides Rapidly progressing glomerulonephritis: syndrome of
Reversible pancreatic parenchymal Acute pancreatitis progressive loss of renal function, characterized by
injury associated with inflammation; nephritic syndrome often with severe oliguria; histologic
pathology is inappropriate release hallmark is presence of “crescents”, thus crescentic GN
and activation of pancreatic enzymes, Type of RPGN associated with anti- RPGN Type I
which destroy pancreatic tissue and GBM antibodies; associated with
elicit an acute inflammatory reaction; Goodpasture syndrome (glomerular
characterized by the following and pulmonary involvement);
TOPNOTCH MEDICAL BOARD PREP PATHO SUPERTABLE Page 29 of 47
For inquiries visit www.topnotchboardprep.com.ph or https://www.facebook.com/topnotchmedicalboardprep/
TOPNOTCH MEDICAL BOARD PREP PATHO SUPERTABLE by KEVIN ELOMINA, MD
For inquiries visit www.topnotchboardprep.com.ph or
https://www.facebook.com/topnotchmedicalboardprep/
uninvolved segments shows no infected AV shunts tive
proliferation LM: Thickened split GBM (Tram track glomerulonephritis
EM: ruptures in the GBM appearance) (MPGN) Type I
IF: linear IgG and C3 deposits along EM: subendothelial electron-dense
the GBM deposits
Type of RPGN associated with RPGN Type II IF: Irregular granular C3 deposits,
immune complex deposition; usually a with IgG and early complement
secondary event of immune complex- components (C1q and C4)
mediated nephritides; uninvolved Formerly called MPGN Type II; Dense deposit
segment shows immune complex fundamental abnormality is excessive disease
deposition complement activation
EM: “lumpy bumpy” appearance of LM: Thickened split GBM (Tram track
GBM (due to deposits) appearance)
IF: granular deposition of Ig and EM: lamina densa and glomerular
complement in GBM capillary wall transformed into
Type of RPGN associated with ANCA; RPGN Type III irregular, ribbon-like, extremely
sometimes a component of ANCA electron-dense structure
vasculitides (Microscopic polyangiitis IF: Irregular chunky and segmental
and Wegener granulomatosis); linear foci of C3 deposits in GBM and
uninvolved segments shows no mesangium, without IgG and early
proliferation complement components (C1q and
EM: no detectable deposits C4)
IF: negative for Ig and complement Isolated glomerular abnormalities
An important cause of ESRD; grossly, Chronic One of the most common causes of IgA nephropathy
kidneys are symmetrically contracted, glomerulonephritis recurrent microscopic or gross (Berger disease)
surfaces are red-brown and diffusely hematuria; most common glomerular Note: Berger
granular; histologically, glomeruli are disease revealed by renal biopsy disease and
obliterated with marked interstitial worldwide. Henoch-Schonlein
fibrosis. LM: mesangial widening and purpura have the
Nephrotic syndrome: 1. Massive proteinuria (≥3.5g/dL; 2. segmental inflammation sam renal
Hypoalbuminemia (<3g/dL); 3. Generalized edema; 4. EM: mesangial electron-dense manifestations and
Hyperlipidemia and lipiduria deposits morphology, but
Most common cause of nephrotic Minimal change IF: mesangial deposition of IgA, often HSP involves
syndrome in children; responsive to disease/Lipoid with C3 and properdin and smaller systemic deposition
corticosteroids. nephrosis amounts of IgG or IgM of IgA, has
LM: none extrarenal
EM: uniform and diffuse effacement of symptoms
foot processes of the podocytes Nephritis associated with hearing and Alport syndrome
Most common cause of nephrotic Focal segmental visual defects; defect in Type IV
syndrome in adults; involves only glomerulosclerosis collagen synthesis; X-linked dominant
some glomeruli (focal), and only a (FSGS) pattern of inheritance
part of glomerulus (segmental) is LM: glomerulosclerosis, vascular
affected; associated with HIV and sclerosis, tubular atrophy, and
heroin abuse. interstitial fibrosis
LM: increased mesangial matrix, EM: thin GBM (early); irregular foci of
obliterated capillary lumina, and GBM thickening or attenuation (late);
deposition effacement of foot “basket-weave” appearance
processes of hyaline masses
(hyalinosis) and lipid droplets. Diseases of the Tubules and Interstitium:
EM: Tubulointerstitial nephritides
IF: nonspecific trapping of Clinicopathologic entity characterized Acute tubular
immunoglobulins, usually IgM, and clinically by acute renal failure and injury/necrosis
complement in the areas of hyalinosis. often, but not invariably, morphologic
Associated with infections (HBV, Membranous evidence of tubular injury, in the form
Syphilis, Schistosomiasis, Malaria), nephropathy of necrosis of tubular epithelial cells;
Malignant solid tumors (lung and has two forms: ischemic and toxic,
colon), SLE, Gold and Mercury, and which vary in terms of affected
drugs (Penicillamine, Captopril, segments of the nephron and
NSAIDs); distribution of necrosis.
LM: diffuse thickening of the capillary Most common cause of acute kidney Acute
wall injury; a common suppurative pyelonephritis
EM: subepithelial deposits along the inflammation of the kidney and renal
GBM (spike and dome appearance); pelvis caused by bacterial infection,
effacement of foot processes either by hematogenous spread or
IF: granular deposits of through ascending infection (more
immunoglobulins and complement common and more important mode);
along the GBM principal causes are Gram-negative
Associated with HBV, HCV, SLE, and Membranoprolifera enteric rods; grossly, there is discrete,

TOPNOTCH MEDICAL BOARD PREP PATHO SUPERTABLE Page 30 of 47


For inquiries visit www.topnotchboardprep.com.ph or https://www.facebook.com/topnotchmedicalboardprep/
TOPNOTCH MEDICAL BOARD PREP PATHO SUPERTABLE by KEVIN ELOMINA, MD
For inquiries visit www.topnotchboardprep.com.ph or
https://www.facebook.com/topnotchmedicalboardprep/
yellowish, raised abscesses grossly Most common composition of kidney Calcium oxalate
apparent on the renal surface; stones. and/or Calcium
histologically, there is liquefactive phosphate (80%)
necrosis with abscess formation Most important cause of kidney stone Supersaturation
within the renal parenchyma. formation.
Hallmark of this disease is scarring Chronic Kidney stones occurring in patients Struvite
involving the pelvis or calyces, or pyelonephritis with alkaline urine due to UTI, (Magnesium
both, leading to papillary blunting and particularly Proteus vulgaris and ammonium
marked calyceal deformities; an staphylococci; most common phosphate);
important cause of chronic kidney composition of staghorn calculi.
disease; forms include: reflux Kidney stones seen in patients with Uric acid stones
nephropathy (more common), and gout and leukemias; urine pH is
chronic obstructive pyelonephritis. decreased; radiolucent.
Second most common cause of acute Drug-induced Kidney stones associated with a defect Cystine stones
kidney injury; T-cell mediated interstitial in the renal transport of certain amino
immune reaction of the kidneys to an nephritis acids; forms in acidic urine.
offending agent, characterized by Tumors
interstitial inflammation, with Tumors derived from renal tubular Renal cell
abundant eosinophils and edema. epithelium, located primarily at the carcinoma (RCC)
Vascular diseases cortex; with three common forms,
Form of nephrosclerosis associated Nephrosclerosis clear cell, papillary renal cell and
with essential hypertension; chromophobe carcinomas; most
histologically, hyaline common primary malignant tumor of
arteriolosclerosis; renal insufficiency the kidney.
is uncommon except in patients of Most important risk factor for renal Smoking
African descent, severe HPN, and cell carcinoma.
other diseases e.g. DM. Most common form (70-80%) of renal Clear cell RCC
Form of nephrosclerosis associated Malignant cell carcinoma and associated with
with malignant hypertension; nephrosclerosis homozygous loss of the VHL tumor
histologically, hyperplastic suppressor gene; thought to arise
nephrosclerosis and fibrinoid from proximal tubule cells; usually
necrosis. solitary and large, spherical masses
Most common causes of renal artery Atherosclerosis and reaching up to 15 cm in diameter; cut
stenosis. fibromuscular surface shows yellow orange to gray-
dysplasia white masses, with prominent areas
Nephron segments most susceptible Short; straight of cystic softening and hemorrhage;
to ischemia, and therefore, most segments of PT and cells appear vacuolated or may be
affected in ischemic ATN. AL LOH solid; often invades the renal vein.
Cystic diseases of the kidney Accounts for 10-15% of cases of RCC; Papillary RCC
Autosomal dominant; kidneys are Adult Polycystic throught to arise from distal tubule
enlarged, composed solely of cysts Kidney Disease cells; characterized by varying
without intervening parenchyma; (ADPKD) degrees of papilla formation with
cysts are filled with clear or turbid fibrovascular cores; cells have clear to
fluid; cysts may arise at any level of pink cytoplasm; associated with
the nephron, with variable, often increased activity of MET oncogene;
atrophic lining; pathology is defective tends to be bilateral.
gene PKD1, which codes for Accounts for 5% of cases of RCC; Chromophobe RCC
polycystin-1; a protein that is thought to arise from intercalated
involved in cell-cell or cell-matrix cells of collecting ducts; grossly tan-
adhesion; most common cause of brown; microscopically, cells have
death: CAD or HHD (40%). clear, flocculent cytoplasm with very
prominent, distinct cell membranes;
nuclei surrounded by halos of cleared
cytoplasm.
Most common site of metastases of Lungs (>50%),
RCC. followed by bones
(33%)

Autosomal recessive; numerous small Childhood
cysts in the cortex and medulla, giving Polycystic Kidney
the kidney a "sponge-like" Disease (ARPKD)
appearance; cysts have uniform
cuboidal epithelium; associated with
multiple cysts in the liver; pathology
is defective PKHD1, which codes for
fibrocystin; a protein found in cilia in
tubular epithelial cells.
Nephrolithiasis
TOPNOTCH MEDICAL BOARD PREP PATHO SUPERTABLE Page 31 of 47
For inquiries visit www.topnotchboardprep.com.ph or https://www.facebook.com/topnotchmedicalboardprep/
TOPNOTCH MEDICAL BOARD PREP PATHO SUPERTABLE by KEVIN ELOMINA, MD
For inquiries visit www.topnotchboardprep.com.ph or
https://www.facebook.com/topnotchmedicalboardprep/
CHAPTER 20: THE LOWER URINARY TRACT AND MALE Composed of sheets of large, uniform
GENITAL SYSTEM cells with distinct cell borders, clear, Note: Female
QUESTION ANSWER glycogen-rich cytoplasm, and round counterpart is
Penis nuclei with conspicuous nucleoli. The Dysgerminoma
An abnormal opening of the urethra Hypospadia (more cells are often arrayed in small
along the ventral or dorsal aspect of common); lobules with intervening fibrous septa
the penis, respectively. epispadia infiltrated with lymphocytes; 15%
Inflammation of the glans and of Balanitis; have increased hCG due to presence
overlying prepuce, respectively; cause balanoposthitis of syncytiotrophoblasts; tumors
by Gardnerella, Candida, anaerobic markers are CD117 (c-kit) and PLAP.
and pyogenic bacteria. Ill-defined invasive masses with foci Embryonal
Appears grossly as a solitary, plaque- Bowen of necrosis and hemorrhage; Large carcinoma
like lesion on the shaft of the penis; disease/Squamous and primitive-looking with indistinct
histologic examination reveals cell carcinoma in cell borders, large nuclei, and
morphologically malignant cells situ of the penis basophilic cytoplasm; negative for
throughout the epidermis with no tumor markers.
invasion of the underlying stroma; Most common primary testicular Yolk sac
has potential for malignant neoplasm in children younger than 3 tumor/Endodermal
transformation. years of age; histologically, low sinus
Occurs in young, sexually active Bowenoid papulosis cuboidal to columnar epithelial cells tumor/Infantile
males; histologically identical to forming microcysts, sheets, glands, embryonal
Bowen disease; presents with and papillae, often associated with carcinoma
multiple reddish brown papules on eosinophilic hyaline globules;
the glans and is most often transient; glomeruli-like structures (Schiller-
virtually never progresses to Duvall bodies) are present; tumor
carcinoma in immunocompetent marker is AFP.
patients. A highly aggressive tumor of the Choriocarcinoma
Appears as a gray, crusted, papular Squamous cell trophoblastic lineage; grossly
lesion, most commonly on the glans carcinoma of the presents as small, nonpalpable
penis or prepuce, which infiltrates the penis masses but with extensive systemic
underlying connective tissue to metastases; histologically
produce an indurated, ulcerated characterized by presence of
lesion with irregular margins; cytotrophoblasts (small cuboidal
histologically similar to Bowen cells) and syncytiotrophoblasts
disease but with infiltration of the (large, eosninophilic syncytial cells,
underlying stroma. containing multiple dark pleomorphic
Scrotum, Testis and Epididymis nuclei); without villus formation;
Represents failure of testicular Cryptorchidism tumor marker is HCG.
descent into the scrotum, which Tissues from all three germ-cell Teratoma
involves the right testis more layers with varying degrees of
commonly than the left; causes differentiation; can contain fully
increased risk of sterility and differentiated tissues from one or
development of testicular cancer; more germ cell layers (e.g., neural
most common phase of arrest: tissue, cartilage, adipose tissue, bone,
inguinoscrotal (4-7th month AOG). epithelium) in a haphazard array
Inflammatory disorders most Epididymis (mature); or immature somatic
commonly affect: testis or elements reminiscent of those in
epididymis? Clue: Gonorrhea and TB developing fetal tissue (immature).
usually affect this organ first. The most important predictor of Age (all teratomas
Tumors most commonly involve: Testis biologic behavior in testicular in postpubertal
testis or epididymis? Clue: Syphilis teratomas. males are regarded
usually affects this organ first. as malignant)
One of the few true urologic Testicular torsion Non-germ cell malignancy arising in a Teratoma with
emergencies, which involves twisting teratoma; malignant component is malignant
of spermatic cord, which can chemoresistant; usual cancers include transformation
ultimately lead to hemorrhagic SCCA, mucin-secreting adenoCA,
infarction of the testis; golden period sarcoma).
for intervention is 6 hours. Most common form of testicular Testicular
Most common cause of painless Testicular tumors neoplasm in men > 60 years of age; lymphoma
testicular enlargement. most common form is DLBCL; with
Testicular neoplasms higher rate of CNS involvement than
Main difference between biologic Germ cell tumors tumors arising in other areas.
behavior of testicular germ cell and are usually Prostate
sex cord-stromal tumors. aggressive; sex Most common form of prostatitis. Chronic abacterial
cord-stromal prostatitis
tumors are usually Composed of proliferating prostatic Nodular Prostatic
benign glandular elements and Hyperplasia/Benign
Most common germ cell tumor; Seminoma (Classic) fibromuscular stroma that form Prostatic
TOPNOTCH MEDICAL BOARD PREP PATHO SUPERTABLE Page 32 of 47
For inquiries visit www.topnotchboardprep.com.ph or https://www.facebook.com/topnotchmedicalboardprep/
TOPNOTCH MEDICAL BOARD PREP PATHO SUPERTABLE by KEVIN ELOMINA, MD
For inquiries visit www.topnotchboardprep.com.ph or
https://www.facebook.com/topnotchmedicalboardprep/
nodules through connective tissue Hyperplasia the epidermis, basal cell
septa; the glandular lumina often degeneration, hyperkeratosis,
contain inspissated, proteinaceous sclerotic changes of superficial
secretory material, termed corpora dermis, dermal lymphocytic
amylacea; stimulus for proliferation is infiltrate; not premalignant, but
increased circulating androgens symptomatic form has increased risk
(dihydrotestosterone); most of vulvar cancer.
commonly occurs is the transition Lesion characterized by acanthosis, Squamous
zone. hyperkeratosis, dermal lymphocytic hyperplasia
MC cancer in men; most commonly Prostatic infiltrate with evident mitosis; not
occurs in the peripheral zone; grossly, Adenocarcinoma considered premalignant.
may appear as firm gray-white Lesions of the anogenital area that Condyloma
lesions with ill-defined margins may be papillary and distinctly acuminata
invading the gland; histologically, elevated or may be somewhat flat
may form gland-like structures and rugose; characteristic cellular
without basal cell layer, appears morphology is the presence of
crowded, and without branching and cytoplasmic vacuolization with
papillary infoldings; tumor marker is nuclear angular polymorphism and
PSA that is used for adjunct for koilocytic atypia: hallmark of HPV
screening and surveillance. infection; not precancerous.
Grading systems used in prostatic Gleason and WHO HPV subtypes associated with HPV 6 and 11
adenocarcinoma. group grade system condyloma acuminata.
Ureter, Bladder and Urethra Most common histology of vulvar Squamous cell
Most common cause of Ureteropelvic carcinoma. carcinoma (Note:
hydronephrosis in children. junction most vulvar Cas are
obstruction non-HPV related
Most common and most serious Vesicoureteral (70%))
congenital anomaly of the urinary reflux
bladder, due to predisposition to
infection and scarring.
Bladder exstrophy, and urachal Bladder Clinically presents as eczematoid Vulvar Paget disease
anomalies predispose patients into adenocarcinoma lesion on the vulva; characterized by
what histologic type of bladder proliferation of malignant epithelial
cancer? cells within the epidermis; usually
Most common presentation of Painless hematuria not associated with underlying
bladder cancer. carcinoma, unlike its breast
Most common type of bladder cancer; Urothelial counterpart; PAS-positive (positive
has two premalignant lesions: 1. (Transitional cell) for mucin), unlike vulvar melanoma.
Noninvasive papillary tumor; and 2. carcinoma (90%) Vagina
Flat noninvasive carcinoma (CIS); Most common histology of vaginal Squamous cell
associated with cigarette smoking carcinoma. carcinoma
and occupational carcinogens. A soft polypoid mass, which is a rare Sarcoma botryoides
Current advancement in the Bacille-Calmette- form of primary vaginal cancer; (Embryonal
treatment of non-invasive urothelial Guerin (BCG) Usually encountered in infants and rhabdomyosarcoma)
carcinomas; also a vaccine that is part children less than 5 y/o.
of EPI, but is administered Condition characterized by Areas of Vaginal adenosis
intravesically. columnar mucinous epithelium
Histologic type of bladder cancer Squamous cell (endocervical-like); 35-90% of
associated with Schistosoma carcinoma patients with in utero exposure to
haematobium infection. DES; associated with clear cell CA of
Most common cause of urinary Nodular prostatic vagina.
bladder obstruction in males and hyperplasia, Cervix
females, respectively. cystocele Premalignant lesion of the cervix; Cervical
Changes that occur in the bladder Thickening of wall, can be low-grade (I; <1/3 of the intraepithelial
with obstruction. Trabeculations, and entire epithelium), moderate (II; up neoplasia (CIN)
Diverticula to 2/3 of the entire epithelium), and
severe (III or CIS; full-thickness of
CHAPTER 21: THE FEMALE GENITAL TRACT epithelium without invasion of the
QUESTION ANSWER basement membrane).
FEMALE GENITAL TRACT Another term for CIN I in the recent
Low-grade
Vulva two-tiered classification system; squamous
Cystic dilation of the Bartholin gland Bartholin cyst regresses in 60% of cases. intraepithelial
due to duct obstruction; may be lesion (LSIL)
infected and may lead to abscess Another term for CIN II and III in the High-grade
formation; occurrence in women >40 recent two-tiered classification squamous
years old requires biopsy to rule out system; persists in 60% of cases, intraepithelial
carcinoma. with 10% of cases progressing to lesion (HSIL)
Lesion characterized by thinning of Lichen sclerosus carcinoma within 10 years.
TOPNOTCH MEDICAL BOARD PREP PATHO SUPERTABLE Page 33 of 47
For inquiries visit www.topnotchboardprep.com.ph or https://www.facebook.com/topnotchmedicalboardprep/
TOPNOTCH MEDICAL BOARD PREP PATHO SUPERTABLE by KEVIN ELOMINA, MD
For inquiries visit www.topnotchboardprep.com.ph or
https://www.facebook.com/topnotchmedicalboardprep/
High-risk HPV implicated in the HPV 16, 18, 31 and Common gene mutated in MED12
development of cervical carcinoma. 33 leiomyomas and leiomyosarcomas.
Viral oncoprotein responsible for E5 Most common benign tumor in Leiomyoma
koilocytic atypia in HPV-infected females; grossly appears as sharply
cells. circumscribed firm gray white
HPV viral gene products that E6 (inactivates p53) masses with a characteristic whorled
promote tumorigenesis and the E7 (inactivates Rb) cut surface; histologically similar to
tumor suppressor genes they normal smooth muscle cells.
inactivate. Malignant counterpart of leiomyoma; Leiomyosarcoma
Most commonly develops in the Invasive cancer of grossly appears as soft, hemorrhagic,
transformation zone of the cervix; the cervix necrotic masses; diagnostic features
produces a "barrel cervix" if the include: tumor necrosis, cytologic
tumor encircles the cervix and atypia and mitotic activity; necrosis
invades the underlying stroma; most being the most important criterion
common histology is squamous cell among the three.
carcinoma (80%), followed by Ovaries
adenocarcinoma (15%) and Most common major type of ovarian Surface epithelial-
neuroendocrine carcinoma (5%); all tumors; 90% of which are malignant. stromal tumors (65-
of the types are caused by HPV. 70%)
Most common cause of death in Uremia Most common ovarian epithelial Serous tumors
cervical cancer. tumors; tumors with tubal-like
Body of Uterus epithelium; most are benign or
Most common cause of dysfunctional Anovulatory cycle borderline (70%).
uterine bleeding. Grossly appears as large, spherical Serous cystadenoma
Refers to the growth of the basal Adenomyosis structures with smooth glistening
layer of the endometrium down to serosal covering with less papillary
the myometrium; nests of projections; histologically, with a
endometrial stroma, glands or both single layer of tall columnar
are found in the myometrium, in epithelial cells without atypia and
between muscle bundles. invasion; usually with Psammoma
Characterized by the presence of Endometriosis bodies.
endometrial glands and stroma in a Serous cystadenoma with Bordeline serous
location outside the myometrium. micropapillary architecture and tumor
Exophytic masses that project into Endometrial polyp epithelial stratification, mild nuclear
endometrial cavity; may be clinically atypia but without invasion of
silent or may cause abnormal uterine stroma.
bleeding. Grossly appears as large, spherical Serous
structures with nodular serosal cystadenocarcinoma
covering with prominent papillar
projections; histologically, with
Tumor suppressor gene commonly PTEN layers of anaplastic epithelial cells
mutated in atypical endometriosis, and invasion; usually with
endometrial hyperplasia and Psammoma bodies; cancers are
endometrial carcinomas; also divided into low-grade (well-
mutated in endometrioid and clear differentiated) or high-grade
cell carcinomas of the ovary. (moderately-differentiated or
Precursor lesion of endometrial Endometrial poorly-differentiated), based on
carcinoma secondary to prolonged hyperplasia general architecture, nuclear atypia
and marked increased estrogen to and severity of stromal invasion.
progestin ratio; can be simple or
complex, with or without atypia. Mutations associated with low-grade KRAS, BRAF, ERBB2
Atypical hyperplasia is now also serous carcinomas.
called endometrial intraepithelial Mutation associated with high-grade TP53
neoplasia (EIN). serous carcinomas.
Most common malignancy of the Endometrial Mucin-containing cystic masses; can Mucinous tumors
female genital tract. carcinoma be benign or malignant, but most of
Most common histology of Endometrioid them are benign; tend to be larger,
endometrial carcinoma found in 80% carcinoma (Type I) multicystic and unilateral; can
of cases; associated with atypical rupture into the peritoneum and
hyperplasia; maybe exophytic or produce mucinous ascites
infiltrative; histologically resembles (pseudomyxoma peritonei).
endometrial glands to solid sheets, Most consistent mutation in KRAS
depending on differentiation; low- mucinous tumors.
grade nuclei is a usual feature. Tumors are usually malignant; 15- Endometrioid
Accounts for 15% of cases of Serous carcinoma 20% coexist with endometriosis; tumors
endometrial carcinoma; forms small (Type II) histologic hallmark is presence of
tufts and papillae with high nuclear tubular glands resembling benign or
grade. malignant endometrium.
TOPNOTCH MEDICAL BOARD PREP PATHO SUPERTABLE Page 34 of 47
For inquiries visit www.topnotchboardprep.com.ph or https://www.facebook.com/topnotchmedicalboardprep/
TOPNOTCH MEDICAL BOARD PREP PATHO SUPERTABLE by KEVIN ELOMINA, MD
For inquiries visit www.topnotchboardprep.com.ph or
https://www.facebook.com/topnotchmedicalboardprep/
Metastatic mucinous Krukenberg tumor with abundant granular, eosinophilic
adenocarcinoma of the ovary from a cytoplasm and small, round, deeply
gastrointestinal primary; often chromatic nuclei (apocrine metaplasia);
produces bilateral ovarian masses. almost always benign.
Cystic, lined by skin-like structures; Mature teratoma Proliferative breast diseases without atypia
usually occur in reproductive age Defined as having more than two layers Epithelial
women; histologically, derivatives of lining epithelium (normally, one hyperplasia
from more than one germ layer can layer for luminal, and another for
be seen; generally benign. myoepithelial cells).
Solid masses that usually occur in Immature teratoma Type of fibrocystic change Sclerosing
younger women and children; characterized by proliferation of adenosis
histologically composed of luminal spaces (adenosis) lined by
neuroepithelium; generally epithelial cells and myoepithelial cells
malignant. with massive stromal fibrosis;
A type of teratoma that produces Struma ovarii important as it appears like carcinoma.
hyperthyroidism due to presence of Clinically presents with bloody or Intraductal
mature thyroid tissue. serous discharge; tumors usually are papilloma
Ovarian tumor that may elaborate Granulosa cell tumor solitary and less than 1 cm in diameter,
large amounts of estrogen; two consisting of delicate, branching
thirds occur in post-menopausal growths within a dilated duct;
women; potentially malignant; microscopically, composed of multiple
histologically composed of mixture papillae, each having a connective
of cuboidal granulosa cells in cords, tissue core covered by double layer of
sheets, or strands and spindled or epithelial cells (important
plump lipid-laden theca cells; distinguishing point from carcinoma
granulosa elements may recapitulate counterpart).
ovarian follicle as Call-Exner bodies. These are stellate lesions characterized Complex
Sex cord tumor that recapitulates Sertoli-Leydig cell by a central nidus of entrapped glands sclerosing
development of testis with tubules or tumor in a hyalinized stroma; important as it lesion/Radial scar
cords and plump pink Sertoli cells; appears like carcinoma
masculinizing and rarely malignant. Proliferative breast diseases with atypia
Sex cord stromal tumor composed of Thecoma-fibroma Basically, DCIS without full duct Atypical ductal
solid gray fibrous cells to yellow involvement; involves monomorphic hyperplasia (ADH)
(lipid-laden) plump thecal cells; proliferations of cells (vs.
rarely malignant; associated with heterogeneous in epithelial
hydrothorax and ascites (Meig hyperplasia).
syndrome). Basically, LCIS but <50% of acini in a Atypical lobular
Diseases of Pregnancy lobule are involved; composed of hyperplasia (ALH)
A type of hydatidiform mole with the Complete monomorphic, loosely cohesive cells,
following features: diploid karyotype hydatidiform mole with absence of E-cadherin.
with diffuse villus edema and Carcinomas
trophoblast proliferation; markedly Most common location of breast Upper outer
elevated hCG with increased risk of carcinoma. quadrant (50%)
developing subsequent The two non-invasive breast Ductal CIS (DCIS)
choriocarcinoma. carcinomas; both of which arise from and Lobular CIS
A type of hydatidiform mole with the Incomplete (Partial) terminal duct lobular unit (TDLU). (LCIS)
following features: triploid hydatidiform mole Immunostain to determine presence of p63
karyotype with focal villus edema myoepithelial layer; an important
and trophoblast proliferation; less immunostain to distinguish in situ
elevated hCG with rare chance of disease from infiltrating carcinomas.
developing subsequent A distinct type of DCIS characterized by Comedocarcinoma
choriocarcinoma. highly pleomorphic nuclei of cells
Complete moles that are more Invasive mole (high-grade nuclei) with extensive
invasive locally but do not have the central necrosis; with calcifications
aggressive metastatic potential of a either from calcified necrotic debris or
choriocarcinoma. secretory material.
Main difference between ovarian and Ovarian chorioCAs Clinically presents as an eczematoid Paget disease of
gestational choriocarcinoma. are more difficult to lesion on the breast; associated with an the nipple
treat invasive carcinoma in most cases; cause
is an underlying DCIS that extended up
CHAPTER 22: THE BREAST to the lactiferous ducts and skin.
Non-proliferative breast changes CIS composed of monomorphic cells LCIS
Multifocal, bilateral blue-brown cysts Fibrocystic change with bland round nuclei (low-grade
("blue dome cysts") of the breast, nuclei) that occur in loosely cohesive
measuring 1-5 cm diameter, filled with clusters within the lobules; tend be
serous turbid fluid. Occurs normally in more commonly bilateral.
the menstrual cycle; histologically,
cysts are lined with large and polygonal
TOPNOTCH MEDICAL BOARD PREP PATHO SUPERTABLE Page 35 of 47
For inquiries visit www.topnotchboardprep.com.ph or https://www.facebook.com/topnotchmedicalboardprep/
TOPNOTCH MEDICAL BOARD PREP PATHO SUPERTABLE by KEVIN ELOMINA, MD
For inquiries visit www.topnotchboardprep.com.ph or
https://www.facebook.com/topnotchmedicalboardprep/


Most common of the infiltrating Invasive (ductal)
carcinomas; associated more with carcinoma (No
DCIS; characterized with intense special type)
desmoplasia that produces a hard
palpable mass; microscopically, ranges
from well-differentiated (tubule
formation) to poorly differentiated
(sheets of anaplastic cells); 50-65% are
ER(+), HER2 (+) (luminal).
The system used in grading invasive Nottingham
carcinomas of no special type. system (Tubule
formation,
Nuclear grade,
and Mitosis)
Infiltrating carcinoma with cells Invasive lobular
looking like those seen in LCIS; only in carcinoma
single-file conformation (“Indian
filing”); reflecting loss of E-cadherin
function, a molecule necessary for
cohesion of breast epithelial cells;
associated with adjacent LCIS in 2/3 of
cases.
Clinically presents as an enlarged, Inflammatory
swollen, erythematous breast with a carcinoma
palpable mass; histologically high-
grade; involves dermal lymphatic
spaces, which explains the
characteristic peau d’orange
appearance; poor prognosis.
Infiltrating carcinoma composed of Medullary
sheets large anaplastic cells, with carcinoma
increased mitosis, with well-
circumscribed pushing borders with a
pronounced lymphoplasmacytic
infliltrate; increased incidence in
patients with BRCA1 mutations; usually
triple-negative breast cancer; poor
prognosis.
Stromal tumors
The most common benign neoplasm of Fibroadenoma
the female breast; grossly, discrete,
usually solitary, freely movable nodule,
1 to 10 cm in diameter, easily "shelled
out" lesion of the breast; histologically
there is a loose fibroblastic stroma
containing duct-like, spaces lined by a
layer of epithelium that are regular and
have a well-defined, intact basement
membrane.
Biphasic tumor composed of highly Phyllodes tumor
cellular stromal elements forming leaf-
like projections and epithelial
elements; usually benign and localized;
ominous changes suggesting
malignancy include increased stromal
cellularity, anaplasia, high mitotic
activity, rapid increase in size, and
infiltrative margins.
Diseases of the Male Breast
Characterized by increase in connective Gynecomastia
tissue and epithelial hyperplasia of the
ducts; lobule formation is rare.
Reason why male breast cancer tend to Less tissue for the
have less favorable prognosis than tumor to invade in
female breast cancer. order to produce
metastases
TOPNOTCH MEDICAL BOARD PREP PATHO SUPERTABLE Page 36 of 47
For inquiries visit www.topnotchboardprep.com.ph or https://www.facebook.com/topnotchmedicalboardprep/
TOPNOTCH MEDICAL BOARD PREP PATHO SUPERTABLE by KEVIN ELOMINA, MD
For inquiries visit www.topnotchboardprep.com.ph or
https://www.facebook.com/topnotchmedicalboardprep/
CHAPTER 23: THE ENDOCRINE SYSTEM component.
QUESTION ANSWER Granulomatous inflammation of the Subacute
PITUITARY thyroid and viral infections; clinically Granulomatous (De
Main histologic feature of pituitary Monomorphic cells presents as painful thyroiditis. Quervain)
adenomas. without a thyroiditis
significant reticulin
network
Most common pituitary adenomas. Prolactin cell
adenoma; followed
by Somatotroph
(GH) cell adenoma Lymphocytic inflammation of the Subacute
The main pathology of Cushing Corticotroph cell thyroid (morphologically similar to Lymphocytic
disease (not syndrome). adenoma Hashimoto but without Hurthle cell thyroiditis
Associated with mass effects and Nonfunctioning change, fibrosis); commonly seen in
hypopituitarism (secondary to pituitary adenomas postpartum patients; clinically
destruction of the normal pituitary presents as painless thyroiditis.
parenchyma). Characterized by extensive fibrosis of Reidel thyroiditis
The only definitive criterion for Metastases the thyroid gland; associated with
diagnosis of pituitary carcinoma. primary retroperitoneal fibrosis
Atypical adenomas with metastases; Pituitary carcinoma (Ormond disease) and autommune
usually functional, with ACTH as the IgG4-related disease; pathology is
most common hormone produced, unknown, but postulated to be of
followed by prolactin. autoimmune type.
Posterior pituitary syndromes. Diabetes insipidus Graves diseases
and SIADH Triad of Graves disease. Thyrotoxicosis,
The origin of this tumor is from Craniopharyngioma ophthalmopathy
vestigial remnants of Rathke pouch; and dermopathy
with bimodal age incidence (5-15 Main pathology of Graves disease. Presence of thyroid
years; 65 years); has stimulating
adamantinomatous, and papillary immunoglobulin
types. (TSIs); binding to
THYROID TSH receptors
Hyperthyroidism and hypothyroidism causes activation of
Most common cause of primary Diffuse toxic follicular cell
hyperthyroidism. hyperplasia function
(Graves disease) Histologic features of Graves disease. Diffuse
(85%) hypertrophy and
Most common cause of congenital Iodine deficiency hyperplasia of the
hypothyroidism worldwide. follicles with
scalloped colloid
Most common cause of Hashimoto (moth-eaten)
hypothyroidism in iodine-sufficient thyroiditis (because of active
areas. reabsorption of
Hypothyroidism in infancy and Cretinism follicular cells)
childhood, impaired development of Diffuse and multinodular goiter; usually hypothyroid (and
CNS (Mental retardation) and skeletal compensatory TSH increase leads to proliferation of the
system (short stature), coarse facial gland)
features, protruding tongue, and Goiter that has 2 phases: 1. Colloid goiter
umbilical hernia. Hyperplastic phase (due to TSH
Hypothyroidism in late childhood and Myexedema influence), and 2. Colloid phase
adults; and slowing of physical and (involution of gland due to sufficient
mental activity; overweight, iodine intake or increased thyroid
hypercholesterolemia, nonpitting hormone demand)
edema, coarse facial features, Repeated hyperplastic and colloid Multinodular goiter
macroglossia and deepening of voice; phases results in irregular
accumulation of matrix substances, involvement of the thyroid gland,
such as glycosaminoglycans and leading to a condition known as:
hyaluronic acid, in skin, subcutaneous Multinodular toxic goiter Plummer syndrome
tissue, and a number of visceral sites. characterized by development of
Thyroiditides autonomously functioning nodules;
Histologically characterized by dense Hashimoto presents with hyperthyroidism not
lymphocytic infiltrate with germinal thyroiditis associated with ophthalmopathy and
centers; follicles are atrophic with dermopathy.
Hurthle cell change (cells with Thyroid neoplasms
eosinophilic granular cytoplasm); Clinical characteristics favoring Solitary, young,
associated with antibodies against malignancy in thyroid nodules. male, history of
thyroglobulin and thyroid peroxidase; radiation therapy,
Type IV hypersensitivity with Type II "cold" nodules
TOPNOTCH MEDICAL BOARD PREP PATHO SUPERTABLE Page 37 of 47
For inquiries visit www.topnotchboardprep.com.ph or https://www.facebook.com/topnotchmedicalboardprep/
TOPNOTCH MEDICAL BOARD PREP PATHO SUPERTABLE by KEVIN ELOMINA, MD
For inquiries visit www.topnotchboardprep.com.ph or
https://www.facebook.com/topnotchmedicalboardprep/
Normal looking follicular cells Follicular adenoma pathology; most common form of DM;
surrounded by an intact capsule; can histologically characterized by
also be composed of follicular cells amyloid replacement of islets.
that underwent Hurthle cell change The hallmark of diabetic Accelerated
(Hurthle cell adenoma); solitary; macrovascular disease, which atherosclerosis
Note: adenomatous nodule, which is a explains its association with coronary
part of multinodular goiter, is the artery disease and cerebrovascular
more appropriate diagnosis for disease.
multiple nodules showing Histologic characteristics of diabetic Diffuse thickening
adenomatous change. microangiopathy. of the basement
Main distinguishing feature of Capsular and membrane, with
follicular carcinoma from follicular vascular invasion more leaky
adenoma. capillaries
Nodular glomerulosclerosis Diabetic
(Kimmelsteil-Wilson lesion); Hyaline nephropathy
arteriolosclerosis; and necrotizing
papillitis from pyelonephritis are
lesions encountered in:
Retinal hemorrhages, Nonproliferative
microaneurysms, venous dilations, (background)
Most common form of thyroid cancer; Papillary edema, exudates and thickening of
histologically characterized by carcinoma retinal capillaries are lesions
papillary architecture with dense encountered in what form of diabetic
fibrovascular cores; nucleus has finely retinopathy?
dispersed chromatin (ground-glass or Neovascularization and fibrosis are Proliferative
Orphan Annie nuclei); with lesions encountered in what form of
Psammoma bodies; spreads by diabetic retinopathy?
lymphatic route (vs. follicular
carcinoma (hematogenous)).
Carcinoma arising from parafollicular Medullary Thyroid
C cells (secretes calcitonin); Cancer (MTC) Pancreatic neuroendocrine tumors
histologically characterized by Most common PanNET; associated Insulinoma
amyloid deposits; known for its with Whipple’s triad: 1. Hypoglycemia
multicentricity and association with <50 mg/dL; 2. Neuroglycopenic
MEN2 syndromes. symptoms; and 3. Relief with feeding
Highly lethal thyroid cancer Undifferentiated or parenteral administration of
(mortality rate of 100% in 6 months) carcinoma glucose; Biologically favorable
with a highly pleomorphic (Anaplastic behavior; characterized by deposition
morphology; clinically presents with carcinoma) of amyloid.
rapidly enlarging thyroid mass; Associated with Zollinger-Ellison Gastrinoma
immunostaining for thyroglobulin is syndrome: 1. Pancreatic islet cell
negative. tumor; 2. Hypersecretion of gastric
Parathyroid acid; and 3. Peptic ulceration; also
Most common cause of primary Parathyroid associated with MEN-1; biologically
hyperparathyroidism; almost always Adenoma (85-95%) aggressive behavior.
confined to one gland (compared to Adrenal cortex
hyperplasia that involves multiple Most common cause of Cushing Exogenous
glands); may sometimes show atypia. syndrome (hypercortisolism). administration of
Definitive criteria for diagnosis of Metastases and steroids
parathyroid carcinoma. local invasion Most common cause of endogenous ACTH-secreting
Most common cause of secondary Renal failure hypercortisolism. pituitary adenoma
hyperparathyroidism. (Cushing disease)
Most common cause of Iatrogenic Morphologic change in adrenal cortex Diffuse hyperplasia
hypoparathyroidism. of patients with ACTH-dependent
Endocrine pancreas Cushing syndrome.
Diabetes Mellitus (Clinical and physiologic aspects left to Morphologic change in adrenal cortex Cortical atrophy
clinical subjects) of patients with ACTH-independent
Absolute deficiency of insulin Type I DM Cushing syndrome. (Cortisol-
secondary of beta cell destruction by secreting adrenal adenoma).
an autoimmune process; accounts for Most common cause of primary Bilateral idiopathic
only 10% of cases of DM; Type IV hyperaldosteronism. hyperplasia (60%)
hypersensitivity with Type II The most common enzymatic defect 21-hydroxylase
component; histologically in congenital adrenal hyperplasia. deficiency
characterized by mononuclear Most common cause of primary Autoimmune
infiltration of the islets with reduction adrenal insufficiency (Addison adrenalitis (60-
of islet size and number (insulitis). disease). 70%)
Peripheral resistance to insulin and Type 2 DM The only criterion to distinguish Metastases
relative insulin deficiency is the main adrenocortical carcinoma from an (adenomas tend to
TOPNOTCH MEDICAL BOARD PREP PATHO SUPERTABLE Page 38 of 47
For inquiries visit www.topnotchboardprep.com.ph or https://www.facebook.com/topnotchmedicalboardprep/
TOPNOTCH MEDICAL BOARD PREP PATHO SUPERTABLE by KEVIN ELOMINA, MD
For inquiries visit www.topnotchboardprep.com.ph or
https://www.facebook.com/topnotchmedicalboardprep/
adrenocortical adenoma. have atypia as well) CHAPTER 24: THE SKIN
Adrenal medulla Note: Please familiarize yourselves with the dermatologic and
Tumor of chromaffin cells; Pheochromocytoma dermatopathologic lexicon before proceeding.
histologically composed of polygonal QUESTION ANSWER
to spindle-shaped cells with finely Disorders of pigmentation and melanocytes
granular cytoplasm, arranged in nests Disease of depigmentation secondary to Vitilligo
called zellballen; only criteria for destruction of melanocytes.
malignancy is metastases; 10% Disease of depigmentation secondary to Albinism
extraadrenal; 10% bilateral; 10% decreased or absent synthesis of
biologically malignant; 10% are not melanin.
associated with hypertension. These are nevi, usually <6 mm, benign- Melanocytic
Multiple endocrine neoplasia syndromes looking, clinically and histologically, and nevus
MEN with 3Ps: Parathyroid (Primary MEN 1 (Wermer are removed for cosmetic reasons; these
hyperparathyroidism either syndrome) lesions have RAS mutations but most
secondary to hyperplasia or never progress because of intact p16
adenoma), Pancreas (either (tumor-suppressor) activity.
Gastrinoma or Insulinoma), and These are large nevi (>5 mm) and may Dysplastic nevi
Pituitary (most common is occur as hundreds of lesions on the body
Prolactinoma, followed by surface; they are flat macules to slightly
Somatotroph adenoma); Autosomal raised plaques, with a "pebbly" surface;
dominant; associated with mutations considered as a marker of melanoma
in MEN1 gene on 11q13. risk.
MEN with MTC, Pheochromocytoma MEN 2A (Sipple The most deadly of all skin cancers; Melanoma
and primary hyperparathyroidism; syndrome) results from excessive sun exposure;
Autosomal dominant; associated with malignant cells have large nuclei with
gain of function mutations on RET irregular contours having chromatin
proto-oncogene in 10q11.2 characteristically clumped at the
MEN with MTC, Pheochromocytoma, MEN 2B periphery of the nuclear membrane and
ganglioneuromas and Marfanoid prominent eosinophilic nucleoli often
habitus; associated with different described as "cherry red"; has both
mutations on RET proto-oncogene. radial and vertical growth phases.
Immunohistochemical staining for S-100, and HMB-
melanoma. 45 (more
specific)
Benign and premalignant tumors
Clinically mimics melanoma; round, Seborrheic
exophytic, coin-like plaques varying in keratosis
diameter, with a velvety/granular
surface; tan to dark brown in color; it
has a stuck-on appearance often seen in
older individuals; the lesions consist of
an orderly proliferation of uniform,
monotonous sheets of small cells
(basaloid in appearance) with a
tendency to form keratin microcysts
(horn cysts).
This lesion is usually the result of Actinic keratosis
chronic exposure to sunlight; dermis
contains thickened, blue-gray elastic
fibers or "solar elastosis" which is the
result of chronic sun damage; grossly,
lesions are less than 1cm, tan-brown or
red in color, with sandpaper-like
surface; microscopically, there is
Atypical dyskeratotic cells in basal
epidermis with intercellular bridges,
solar elastosis, hyperparakeratosis and
dermal chronic inflammation.
Malignant epithelial tumors
A common tumor arising on sun- Squamous cell
exposed sites in older people, with carcinoma
higher incidence in women; may arise
from prior actinic keratoses, then when
advanced becomes nodular and may
ulcerate; characterized by anaplastic
(seen on all levels of the epidermis),
rounded cells with foci of necrosis and
only abortive, single-cell keratinization
TOPNOTCH MEDICAL BOARD PREP PATHO SUPERTABLE Page 39 of 47
For inquiries visit www.topnotchboardprep.com.ph or https://www.facebook.com/topnotchmedicalboardprep/
TOPNOTCH MEDICAL BOARD PREP PATHO SUPERTABLE by KEVIN ELOMINA, MD
For inquiries visit www.topnotchboardprep.com.ph or
https://www.facebook.com/topnotchmedicalboardprep/
(dyskeratosis). Blistering disorders
A rare autoimmune blistering disorder Pemphigus
resulting from loss of integrity of normal vulgaris
intercellular attachments within the
epidermis and mucosal epithelium;
caused by a Type II hypersensitivity
reaction (mainly IgG) against
desmoglein; histologically characterized
by acantholysis immediately above the
basal layer.
This is the most common human cancer, Basal cell Also an autoimmune (IgG-mediated Bullous
which is a slow-growing tumor that carcinoma against bullous pemphigoid antigens) pemphigoid
rarely metastasizes; tends to occur at blistering disease against the epidermal
sites subject to chronic sun exposure basement membrane components
and in lightly pigmented people; These (hemidesmosomes); however, it is
tumors present as pearly, smooth- characterized by subepidermal
surfaced papules, often containing nonacantholytic blisters.
prominent, dilated subepidermal blood
vessels (telangiectasia); the cells have
scant cytoplasm, small hyperchromatic
nuclei, and a peripheral palisade with Associated with celiac disease; clinically Dermatitis
clefting from the stroma. characterized by pruritic urticarial; herpetiformis
Acute inflammatory dermatoses histologically characterized by
Condition caused by local mast cell Urticaria subepidermal blisters that develop from
degranulation; histologically, there is coalescence of vacuolized
usually a sparse superficial perivenular microabscesses at the dermal papilla
infiltrate of mononuclear cells with tips.
superficial dermal edema. Infectious dermatoses
This term is the accumulation of edema Spongiosis A skin infection caused by either S. Impetigo
fluid within the epidermis; characterizes aureus (characterized by formation of
all forms of eczamatous dermatitis. bullae) or S. pyogenes (characterized by
An uncommon, usually self-limited Erytherma appearance of honey-colored crusts);
disorder that seems to be a multiforme histologically characterized by
hypersensitivity response to certain subcorneal neutrophilic infiltration.
infections and drugs; patients present A benign lesion cause by low-risk strains Verrucae
with an array of "multiform" lesions, of HPV (2, 4 and 7); characterized by
including macules, papules, vesicles, and koilocytic atypia (cytoplasmic
bullae, as well as the characteristic vacuolization).
targetoid lesion consisting of a red
macule or papule with a pale vesicular CHAPTER 25: BONES, JOINTS, AND SOFT TISSUE TUMORS
or eroded center; part of a spectrum QUESTION ANSWER
with SJS-TEN. BONES
Chronic inflammatory dermatoses Congenital disorders
A skin disorder whose main pathology is Psoriasis Most common skeletal dysplasia; Achondroplasia
increased epithelial cell turnover; there characterized by
is acanthosis and loss of the stratum disproportionate shortening of
granulosum with extensive overlying the proximal extremities, bowing
parakeratotic scale; there is also a of the legs, and a lordotic (sway-
regular downward elongation of rete backed) posture; pathology is
ridges (test tubes in a rack appearance); mutations on FGFR3; autosomal
associated with multiple punctate dominant.
hemorrhages upon removal of scales Most common lethal form of Thanatophoric
from the lesions (Auspitz sign) due to dwarfism; also caused by dysplasia
presence of dilated tortuous vessels. mutations in FGF3; usual cause of
Small aggregates of neutrophils within Psoriasis death is respiratory insufficiency
the parakeratotic stratum corneum (underveloped chest cavity).
(Munro microabscesses); and spongiotic This is a group of hereditary Osteogenesis imperfect
superficial epidermis (Pustules of Kogoj) disorders caused by defective (OI) "brittle bone
are findings associated with: synthesis of type I collagen; disease"
"Pruritic, purple, polygonal, planar Lichen planus classic finding of a "blue sclerae"
papules, and plaques" describes this is seen in one type of this
disorder of the skin and mucosa. Also disorder; has 4 types; Type II is
noted grossly are Wickham striae, which the most lethal; most commonly
are white lacelike markings over the autosomal dominant.
papules; pattern of inflammation of this A disease that manifests with Osteopetrosis (Albers-
disorder is characterized by angulated, dense but structurally unsound Schonberg disease)
zigzag contour ("sawtoothing") of the bone due to mutations that result "Marble bone disease"
dermoepidermal junction. to impaired osteoclast function;
TOPNOTCH MEDICAL BOARD PREP PATHO SUPERTABLE Page 40 of 47
For inquiries visit www.topnotchboardprep.com.ph or https://www.facebook.com/topnotchmedicalboardprep/
TOPNOTCH MEDICAL BOARD PREP PATHO SUPERTABLE by KEVIN ELOMINA, MD
For inquiries visit www.topnotchboardprep.com.ph or
https://www.facebook.com/topnotchmedicalboardprep/
also the first disease to be treated tuberculous osteomyelitis
with HSC transplantation. involving vertebral bodies; can
Acquired diseases cause vertebral deformity,
The hallmark of this disease is Osteoporosis collapse and posterior
loss of bone; cortices are thinned displacement leading to
and trabeculae are reduced in neurologic deficits.
thickness; osteoclastic activity is Tumors
present but not dramatically Bone-forming tumors
increased; mineral content of the Bone-forming tumors with Osteoid osteoma
bone tissue is normal. marked reactive bone (nidus) Note: osteoblastomas
Basic difference between Severe osteopenia formation; clinically, <2 cm are > 2 cm, and pain is
osteopenia and osteoporosis. enough to increase risk lesions; painful but relieved by not relieved by NSAIDs;
of fractures in NSAIDs. morphologically, less
osteoporosis nidus formation
Characterized by repetitive Paget disease of bone Basic difference between osteoid Presence of osteoblastic
episodes of frenzied, regional (Osteitis deformans) osteoma/osteoblastoma and rimming in osteoid
osteoclastic activity and bone fibrous dysplasia. osteoma/osteoblastoma
resorption, followed by A bone-producing malignant Osteosarcoma
exuberant bone formation, and mesenchymal tumor,
finally by an apparent exhaustion characterized as gritty, gray-
of cellular activity; white tumors, often exhibiting
pathognomonic feature is a hemorrhage and cystic
"mosaic pattern" of lamellar degeneration; the production of
bone; associated with secondary mineralized or unmineralized
osteosarcoma. bone (osteoid) by malignant cells
is essential for diagnosis;
associated with Codman triangle
and sunburst pattern,
radiographically; metaphyseal.
Most common benign bone Osteochondroma
Condition that results from Rickets (children)/ tumor, characterized by hyaline
Vitamin D deficiency; Osteomalacia (adults) cartilage-capped outgrowths
characterized by attached by a bony stalk to the
undermineralization of bone (as underlying skeleton;
opposed to osteoporosis where metaphyseal.
bone mineral content is normal),
rendering one susceptible to
fractures.
Basic difference between rickets Osteoporosis: normal
and osteomalacia, and mineralization,
osteoporosis. decreased bone mass Second most common malignant Chondrosarcoma
Rickets/Osteomalacia: matrix=producing tumor of bone; (Note: If you see
decreased mineral a malignant tumor of malignant tumor made
content of bone chrondrocytes; these lesion of chrondrocytes in a
Components of von Increased bone cell arises within the medullary child, think
Recklinghausen disease of bone. activity, peritrabecular cavity of the bone to form an chondroblastic
fibrosis, cystic brown expansile glistening mass that osteosarcoma first)
tumors often erodes the cortex; they
Term used for skeletal changes in Renal osteodystrophy exhibit malignant hyaline and
chronic renal failure (including myxoid cartilage; in its
dialysis). conventional form, it is almost
Osteomyelitis always never found in children.
Most common route of infection Hematogenous Second most common group of Ewing sarcoma
of pyogenic osteomyelitis. dissemination bone sarcomas in children;
Most common organism S. aureus primary malignant small round-
implicated in pyogenic cell tumor of bone and soft tissue,
osteomyelitis overall. characterized by sheets of small
Common organisms implicated in E. coli, Pseudomonas, round cells with scant, cleared
pyogenic osteomyelitis in IV drug Klebsiella cytoplasm, circled about a central
users and patients with GUT fibrillary space or "Homer-
infections. Wright rosettes"; basically a
Common organisms implicated in H. influenzae and GBS PNET of bone; associated with a
pyogenic osteomyelitis in (organisms involves in chromosomal translocation
neonates. sepsis) abnormality (t(11:22)).
Organism implicated in Salmonella A relatively uncommon benign Giant cell tumor of bone
osteomyelitis in patients with tumor usually arising in (Osteoclastoma)
sickle cell anemia. individuals in their 20s to 40s;
A clinically serious form of Pott disease these are large and red-brown
TOPNOTCH MEDICAL BOARD PREP PATHO SUPERTABLE Page 41 of 47
For inquiries visit www.topnotchboardprep.com.ph or https://www.facebook.com/topnotchmedicalboardprep/
TOPNOTCH MEDICAL BOARD PREP PATHO SUPERTABLE by KEVIN ELOMINA, MD
For inquiries visit www.topnotchboardprep.com.ph or
https://www.facebook.com/topnotchmedicalboardprep/
lesions with frequent cystic found in the cytoplasm of the
degeneration, composed of neutrophils as well as in small
uniform oval mononuclear cells clusters in the synovium.
with frequent mitoses, with Pathognomonic feature of gout; Tophi
scattered osteoclast-type giant formed by large aggregations of
cells containing 100 or more urate crystals surrounded by an
nuclei; locally invasive; intense inflammatory reaction of
epiphyseal. lymphocytes, macrophages, and
Most common skeletal Metastases foreign-body giant cells,
malignancy. attempting to engulf the masses
JOINTS of crystals.
Most common form of arthritis; Osteoarthritis Crystal-induced arthropathy due Pseudogout
most fundamental feature is to calcium pyrophosphate
degeneration of articular deposition; crystals are
cartilage; characterized by rhomboid, and positively-
fibrillation and cracking of the birefringent.
articular cartilage matrix, bone SOFT TISSUE
eburnation, and bony Most common soft tissue tumor Lipoma
outgrowths/spurs (osteophytes); of adulthood.
full-thickness portions of the Most common sarcoma of Liposarcoma
cartilage are lost, and the adulthood.
subchondral bone plate is Most common soft tissue Rhabdomyosarcoma
exposed. sarcoma of childhood and
Chronic inflammatory Rheumatoid arthritis adolescence.
autoimmune disease principally (RA) Immunostain for Myogenin
attacking the joints; mechanism rhabdomyoblastic differentiation.
of disease is Type IV Most common neoplasm in Leiomyoma
hypersensitivity; radiographic women: common site is the
hallmark of this joint disease are uterus.
joint effusions and juxta-articular Immunostains for smooth muscle Smooth muscle actin,
osteopenia with erosion and differentiation. desmin
narrowing of joint space and loss
of articular cartilage. CHAPTER 26: PERIPHERAL NERVES AND SKELETAL
Basic difference between OA and Joint fusion is absent in MUSCLES
RA. OA QUESTION ANSWER
Classic lesion of RA composed of Pannus Disorders of peripheral nerve
proliferating synovial lining cells Most common chronic acquired Chronic inflammatory
admixed with inflammatory cells, peripheral neuropathy; clinical demyelinating
granulation tissue, and fibrous feature is mixed sensorimotor polyradiculoneuropathy
connective tissue; the polyneuropathy ≥ 2 months. (CIDP)
overgrowth of this tissue is so Most common inherited Charcot-Marie Tooth
exuberant that the usually thin, peripheral neuropathy; has (CMT) disease
smooth synovial membrane is autosomal dominant (CMT1 and
transformed into lush, 2) and CMTX (X-linked) forms;
edematous, frondlike (villous) CMT2 is an axonal neuropathy
projections. that is phenotypically severe and
presents early in life.

Rapidly progressive acute Guillain-Barre

demyelinating disorder affecting syndrome

motor axons that results in
Examples of seronegative Ankylosing spondylitis ascending weakness that may
spondyloarthropathies. (Rheumatoid lead to death from failure of
spondylitis, Marie- respiratory muscles over a
Strumpell disease), period of only several days;
Reactive arthritis associated with C. jejuni infection.
(Reiter syndrome),
The most common form of Distal symmetric
Enteritis-associated
diabetic neuropathy (DM sensorimotor
arthritis, Psoriatic
neuropathy: most common cause neuropathy
arthritis
of peripheral neuropathy)
Crystal-induced arthropathy
Crystal-induced arthropathy due Gout
to excessive amounts of uric acid; Disorders of neuromuscular junction
histologically characterized by a
Disorder caused by Myasthenia gravis
dense neutrophilic infiltrate
autoantibodies that block the
permeating the synovium and
function of postsynaptic
synovial fluid; long, slender,
acetylcholine receptors at motor
needle-shaped monosodium
end plates, which results in the
urate crystals are frequently
degradation and depletion of the
TOPNOTCH MEDICAL BOARD PREP PATHO SUPERTABLE Page 42 of 47
For inquiries visit www.topnotchboardprep.com.ph or https://www.facebook.com/topnotchmedicalboardprep/
TOPNOTCH MEDICAL BOARD PREP PATHO SUPERTABLE by KEVIN ELOMINA, MD
For inquiries visit www.topnotchboardprep.com.ph or
https://www.facebook.com/topnotchmedicalboardprep/
receptors; clinically presents
with fatigable weakness.
Disorder caused by Lambert-Eaton
autoantibodies that inhibit the myasthenic syndrome Autosomal dominant disorder Neurofibromatosis Type
function of presynaptic calcium (LEMS) caused by mutations in the tumor 1
channels, which reduces the suppressor neurofibromin a
release of acetylcholine into the tumor suppressor of Ras
synaptic cleft; associated with oncoprotein, encoded on
SCLC as paraneoplastic Chromosome 17 (17q11.2);
syndrome. associated with development of
Acquired disorders of skeletal muscle neurofibromas, MPNSTs, optic
Most common in inflammatory Dermatomyositis nerve gliomas, other glial tumors
myopathy in children; associated and hamartomatous lesions,
with Gottron papules and pheochromocytomas, pigmented
heliotrope rash (see Pediatrics); iris nodules (Lisch nodules), and
microscopically, it is associated cutaneous hyperpigmented
with perivascular mononuclear macules (axillary freckling and
cell infiltrates, “dropout” of café au lait spots).
capillaries, the presence of so- Autosomal dominant disorder Neurofibromatosis Type
called tubuloreticular inclusions caused by mutations in the 2
in endothelial cells, and myofiber merlin gene (NF2), a tumor
damage in a paraseptal or suppressor that facilitates E-
perifascicular pattern. cadherin contact inhibition, in
Basic difference between Skin changes absent in Chromosome 22 (Ch22q12);
dermatomyositis and polymyositis hallmark is bilateral vestibular
polymyositis. schwannomas, meningiomas and
Most common inflammatory Inclusion body myositis ependymomas (commonly
myopathy in patients older than intraspinal).
65 years; clinically present with
quadriceps and distal upper CHAPTER 27: THE CENTRAL NERVOUS SYSTEM
extremity weakness, with Note: Congenital anomalies are omitted because of low-yield;
dysphagia. for completion, you may refer to the parent handout. Also,
some common CNS anomalies are discussed in anatomy (under
embryology). CNS infections are covered in Microbiology, and
Muscular dystrophies lumbar tap findings for meningitides are covered in IM and
Disorders caused by mutation in Duchenne Muscular Pediatrics.
dystrophin gene in X Dystrophy QUESTION ANSWER
chromosome (Xp21); patients (DMD)/Becker Cellular pathology of the CNS
present with weakness at around Muscular Dystrophy General response of neurons to injury. Acute: Intense
5 years of age; (BMD) (less severe) eosinophilia (red
pseudohypertrophy of the calf neuron)
muscles is an important initial Chronic: cell loss
physical finding; associated with General response of astrocytes and Astrocytes:
Gower sign. microglia to injury. Hyperplasia,
Differences in dystrophin DMD: Dystrophin(-) hypertrophy; with
immunohistochemistry between BMD: Dystrophin(+) but accumulation of
DMD and BMD. reduced GFAP
Peripheral nerve sheath tumors Microglia:
Mixture of cellular areas (Antoni Schwannoma/ proliferation
A) and hypocellular areas in Neurilemmoma Edema, herniation and hydrocephalus (focus on general
myxoid stroma (Antoni B) with concepts)
palisading of nuclei and Verocay It is the accumulation of excess fluid Cerebral edema
bodies; Immunohistochemical within the brain parenchyma. The
staining for S-100 is positive; brain is softer than normal and often
associated with NF type 2. appears to "overfill" the cranial vault.
Tumor composed of Bland Neurofibroma In generalized edema the gyri are
Schwann cells with stromal cells flattened, the intervening sulci are
(mast cells, perineurial cells, narrowed, and the ventricular cavities
CD34+ spindle cells and are compressed; can be vasogenic
fibroblasts) in a loose collagen (occurs when the integrity of the
stroma; associated with NF type normal blood-brain barrier is
1. disrupted; with increased vascular
Most common origin of Malignant permeability, fluid shifts from the
malignant peripheral nerve transformation of a vascular compartment into the
sheath tumors. plexiform neurofibroma intercellular spaces of the brain), or
cytotoxic (due to an increase in
intracellular fluid secondary to
neuronal, glial, or endothelial cell
TOPNOTCH MEDICAL BOARD PREP PATHO SUPERTABLE Page 43 of 47
For inquiries visit www.topnotchboardprep.com.ph or https://www.facebook.com/topnotchmedicalboardprep/
TOPNOTCH MEDICAL BOARD PREP PATHO SUPERTABLE by KEVIN ELOMINA, MD
For inquiries visit www.topnotchboardprep.com.ph or
https://www.facebook.com/topnotchmedicalboardprep/
membrane injury, as might be liquefactive necrosis of the brain
encountered in an individual with a parenchyma.
generalized hypoxic/ischemic insult or Injury to small perforating vessels in Lacunar infarcts
with exposure to some toxins). the form of sclerosis due to
hypertension; morphologically, small,
cavitary infarcts (lacunes).
Injury to small perforating vessels, in Slit hemorrhages
the form of rupture of small vessels
due to hypertension; morphologically,
tissue destruction, pigment-laden
macrophages and gliosis.
Bleeding secondary to rupture of small Hypertensive
intraparenchymal vessel; intraparenchymal
Refers to the accumulation of CSF Hydrocephalus nontraumatic; most common cause of hemorrhage
leading to dilation of the ventricular deep parenchymal hemorrhage is
system; can be noncommunicating hypertension; commonly affects the
(from an obstacle or disruption of flow basal ganglia (putamen), thalamus,
seen most commonly at the foramen of and pons.
Monroe or aqueduct of Sylvius), or Intraparenchymal hemorrhage Cerebral amyloid
communicating (due to reduced affecting the leptomeninges and angiopathy-
resorption of CSF; all of the ventricular cortex; due to deposition of amyloid in associated
system is enlarged). small to medium-sized leptomingeal intraparenchymal
Effect of increased intracranial Herniation and cortical vessels. hemorrhage
pressure from an increase in volume of Patients with this type of intracranial Subarachnoid
any intracranial component; may lead hemorrhage complains of having "the hemorrhage
to vascular compromise, infarction, worst headache I've ever had"/
additional swelling and herniation; has thunderclap headache; the most
several types including, subfalcine, frequent cause is rupture of saccular
transtentorial, and tonsillar. berry aneurysm; most common
CNS trauma (focus on traumatic vascular injuries) location is ACA-ACoA junction (40%).
This leads to accumulation of blood Epidural A vascular malformation wherein the Arteriovenous
between the dura and the skull; the hematoma involved vessels resemble a tangled malformation
expanding hematoma has a smooth network of wormlike vascular
inner contour that compresses the channels; microscopically, they are
brain surface; clinically, patients may enlarged blood vessels separated by
experience a lucid interval between gliotic tissue, often with evidence of
the moment of trauma and prior hemorrhage; common location
development of neurologic symptoms; vessels involved are of the
bleeding source usually arterial (from subarachnoid space and brain.
middle meningeal artery in pterion Demyelinating disorders
fractures); on imaging, typically shows Most common demyelinating disorder; Multiple sclerosis
a lentiform bleed. characterized by distinct episodes of
This leads to accumulation of blood in Subdural neurologic deficits, separated in time,
the subdural space; bleeding is usually hematoma attributable to white matter lesions
venous (from injury to bridging veins); that are separated in space; affected
usually observed in extremes of ages; areas show multiple, well-
can be chronic; on imaging, typically circumscribed, slightly depressed,
shows a crescent-shaped bleed. glassy, gray-tan, irregularly shaped
Cerebrovascular diseases lesions, termed "plaques"; common
In the setting of this condition, the Global cerebral initial manifestation is unilateral
brain is swollen, with wide gyri and ischemia involvement of the optic nerve (optic
narrowed sulci; the cut surface shows or retrobulbar neuritis).
poor demarcation between gray and CSF findings in Multiple sclerosis. Moderate
white matter; results from generalized pleocytosis, mildly
reduction of cerebral perfusion, elevated protein,
usually below systolic pressures of less increased IgG
than 50mmHg. (oligoclonal)
Neurons most sensitive to ischemia. Pyramidal layer of Clinically present with bilateral optic Neuromyelitis
hippocampus neuritis and spinal cord optica (NMO)
(CA1: Sommer demyelination; associated with
sector), cerebellar antibodies against aquaporin-4.
Purkinje cells, Demyelinating disorder associated Central pontine
pyramidal cells of with rapid correction of hyponatremia. myelinolysis
cerebral cortex (CPM)/Osmotic
Results from cerebral artery occlusion, Focal cerebral demyelination
either thrombotic or embolic; and ischemia syndrome (ODS)
hemorrhagic or nonhemorrhagic; Neurodegenerative diseases
nonhemorrhagic infarcts show It is the most common cause of Alzheimer disease
TOPNOTCH MEDICAL BOARD PREP PATHO SUPERTABLE Page 44 of 47
For inquiries visit www.topnotchboardprep.com.ph or https://www.facebook.com/topnotchmedicalboardprep/
TOPNOTCH MEDICAL BOARD PREP PATHO SUPERTABLE by KEVIN ELOMINA, MD
For inquiries visit www.topnotchboardprep.com.ph or
https://www.facebook.com/topnotchmedicalboardprep/
dementia in the older adults, (AD) capillaries with calcifications.
characterized by presence of plaques Common location of this tumor is near Ependymoma
composed of Aβ amyloid, and 4th ventricle during the first two
neurofibrillary tangles composed of decades of life, and intraspinal in
Tau proteins. adults; cytogenetic origin is ependymal
Loss of dopaminergic neurons in Parkinson disease cells; tend perivascular pseudorosettes
substantia nigra; triad of tremor, (PD) on histologic examination.
rigidity and bradykinesia; there are Poorly-differentiated tumors
single or multiple, intracytoplasmic, Often located midline in the Medulloblastoma
eosinophilic, round to elongated cerebellum in children, and lateral in
inclusions that often have a dense core adults; on histology, lesion is
surrounded by a pale halo or Lewy extremely cellular with sheets of
bodies, composed of α-synuclein anaplastic (small blue) cells; can also
aggreagates. form Homer-Wright rosettes;
Accumulation of Lewy bodies in basal Lewy body essentially a primitive
nucleus of Meynert leads to this dementia neuroectodermal tumor (PNET);
condition: radiosensitive.
This is the most common form of Amytropic lateral Other tumors
neurodegeneration affecting the motor sclerosis (ALS) Most common form of primary CNS DLBCL
system; characterized by muscle lymphoma.
atrophy and hypereflexia due to loss of Cytogenetic origin is arachnoid Meningioma
both upper and lower motor neurons. meningothelial cells; varied histologic
Acquired metabolic and toxic disturbances patterns include syncytial, fibroblastic,
Psychotic symptoms or Wernicke transitional, psammomatous and
ophthalmoplegia on a background of syndrome secretory (associated with
acute alcoholism; reversible with pseudopsammoma bodies); grades
thiamine administration; include: Typical (WHO I/IV), Atypical
morphologically, hemorrhage and (WHO II/IV); and Anaplastic
necrosis of the mamillary bodies and (Malignant) (WHO III/IV).
the walls of the third and fourth Sharply demarcated masses, often at Metastases
ventricles the junction of gray and white matter,
usually surrounded by a zone of
edema; usual sources are lung, breast,
Short-term memory disturbances and Korsakoff skin (melanoma), kidney, and GIT, and
confabulation on a background of syndrome choriocarcinomas.
chronic alcoholism; not reversible with
thiamine administration;
morphologically, cystic space with
hemosiderin-laden macrophages in
dorsomedial nucleus of the thalamus.
Degeneration of posterolateral cord Subacute
tracts due to abnormal myelin combined
formation associated with Vitamin B12 degeneration
deficiency; clinically, sensory
disturbance and ataxia in LEs that
leads to spastic paralysis and
paraplegia.
Tumors
Gliomas
Cytogenetic origin is astrocytes; forms Astrocytoma
include: Pilocytic (WHO I/IV)
(common in children), Well-
differentiated (WHO II/IV), Anaplastic
(WHO III/IV) and glioblastoma (WHO
IV/IV) (Note: the rest except pilocytic
astrocytoma are called infiltrating
astrocytomas, and are common in
adults).
Histologic criteria for the diagnosis of Necrosis and
glioblastoma. endothelial
proliferation
Cytogenetic origin is oligodendrocytes; Oligodendroglioma
forms include: Well-differentiated
(WHO II/IV), and Anaplastic (WHO
III/IV); histologically characterized by
round tumor cells with cytoplasmic
halos ("fried-egg appearance) supplied
a by network of anastomosing
TOPNOTCH MEDICAL BOARD PREP PATHO SUPERTABLE Page 45 of 47
For inquiries visit www.topnotchboardprep.com.ph or https://www.facebook.com/topnotchmedicalboardprep/
TOPNOTCH MEDICAL BOARD PREP PATHO SUPERTABLE by KEVIN ELOMINA, MD
For inquiries visit www.topnotchboardprep.com.ph or
https://www.facebook.com/topnotchmedicalboardprep/
CHAPTER 28: THE EYE findings typical of this condition, which
QUESTION ANSWER may occur in the presence or absence of
Orbit ischemia.
Forward displacement of the eye; has Proptosis Most common type of retinal lymphoma. DLBCL
two forms: axial and positional.
Most common primary tumor of the Vascular tumors
orbit. (Infancy and
early childhood:
capillary
hemangioma,
lymphangioma; Most common primary intraocular Retinoblastoma
adults: cavernous malignancy in children; clinical findings
hemangioma) include poor vision, strabismus, whitish
Eyelid hue when eyes are illuminated
Most common malignancy of the eyelid. Basal cell (luekocoria); histologically, composed of
carcinoma small, round cells with large
Conjunctiva hyperchromatic nuclei and scant
Etiologic agent of conjunctivitis that lead Chlamydia cytoplasm, with characteristic
to significant conjunctival scarring. trachomatis structures consisting of clusters of
Most common site of conjunctival Limbus cuboidal or short columnar cells
neoplasms. arranged around a central lumen; nuclei
Common histology of common Squamous cell are displaced away from the lumen,
malignancies. carcinomas, which appears to have a limiting
Melanomas membrane; associated with Flexner-
Anterior segment Wintersteiner rosettes and fleurettes.
Inflammation within the vitreous Endophthalmitis Optic nerve
humor. Swelling of the optic nerve head; can be Optic nerve
Inflammation involving the retina, uvea, Panophthalmitis unilateral or bilateral (papilledema); edema
sclera and orbit. unilateral usually caused by nerve
Opacification of the lens. Cataract compression; bilateral usually caused by
increased ICP.
Submucosal fibrovascular connective Pinguecula Loss of vision secondary to Optic neuritis
tissue that does not invade the cornea. demyelination of the optic nerve; most
important cause is multiple sclerosis.
Submucosal fibrovascular connective Pterygium
tissue that invades the cornea; Most common primary optic nerve Pilocytic
associated with visual problems tumors. astrocytomas and
(astigmatism). meningiomas
Uvea
References:
Inflammation of the tissues that Uveitis
comprise the uvea (iris, ciliary body and • Kumar V, Abbas AK, Aster JC. 2013. Robbins Basic
choroid). Pathology, 9th ed. Elsevier Saunders PA USA.
• Kumar V, Abbas AK, Fausto N, Aster JC. 2010. Robbins and
Most common intraocular malignancy. Metastases to Cotran Pathologic Basis of Disease. 9th ed. Elsevier
uvea Saunders PA USA.
Most common primary intraocular Melanoma • Schneider AS, Szanto PA. 2014. BRS Pathology. 5th ed.
malignancy in adults. Lippincott Williams & Wilkins PA USA.
Retina and vitreous • Topnotch Pathology handout
Unclean vitreous detachment tears the Rhegmatogenous • Topnotch Pathology Flashcards
retina; liquefied vitreous accumulates retinal -------
between RPE and neurosensory retina; detachment
involves full-thickness retinal defect. There verses are included in my guiding verses in life, and
during the boards. It speaks of the power of persistence, and
Any condition that damages the RPE and Non-
the providence of God to his children.
permits fluid to leak from the choroidal rhegmatogenous

circulation under the retina (choroidal
Luke 11:5-13 New King James Version (NKJV)
tumors, malignant hypertension) leads

to this type of retinal detachment; does
A Friend Comes at Midnight
not involve full-thickness retinal defect.
And He said to them, “Which of you shall have a friend, and go
Ophthalmologic emergency; usually Central retinal
to him at midnight and say to him, ‘Friend, lend me three
occurs with ischemia; histologically artery occlusion
loaves; for a friend of mine has come to me on his journey, and
characterized by fragments of (CRAO)
I have nothing to set before him’; and he will answer from
atherosclerotic plaque lodged in retinal
within and say, ‘Do not trouble me; the door is now shut, and
circulation (Hollenhorst plaques); and
my children are with me in bed; I cannot rise and give to you’? I
cherry-red spot in macula (fovea and
say to you, though he will not rise and give to him because he is
foveola) with retinal pallor.
his friend, yet because of his persistence he will rise and give
Retinal hemorrhages, dilated tortuous Central retinal
him as many as he needs.
retinal veins, cotton-wool spots, macular vein occlusion

edema, and optic disc edema, are (CRVO)
Keep Asking, Seeking, Knocking
TOPNOTCH MEDICAL BOARD PREP PATHO SUPERTABLE Page 46 of 47
For inquiries visit www.topnotchboardprep.com.ph or https://www.facebook.com/topnotchmedicalboardprep/
TOPNOTCH MEDICAL BOARD PREP PATHO SUPERTABLE by KEVIN ELOMINA, MD
For inquiries visit www.topnotchboardprep.com.ph or
https://www.facebook.com/topnotchmedicalboardprep/
“So I say to you, ask, and it will be given to you; seek, and you
will find; knock, and it will be opened to you. For everyone who
asks receives, and he who seeks finds, and to him who knocks it
will be opened. If a son asks for bread[a] from any father
among you, will he give him a stone? Or if he asks for a fish, will
he give him a serpent instead of a fish? Or if he asks for an egg,
will he offer him a scorpion? If you then, being evil, know how
to give good gifts to your children, how much more will your
heavenly Father give the Holy Spirit to those who ask Him!"


TOPNOTCH MEDICAL BOARD PREP PATHO SUPERTABLE Page 47 of 47


For inquiries visit www.topnotchboardprep.com.ph or https://www.facebook.com/topnotchmedicalboardprep/

Das könnte Ihnen auch gefallen